Book TB for content review

Ace your homework & exams now with Quizwiz!

10. A 15-year-old child is brought to the emergency department with symptoms of hyperglycemia and is subsequently diagnosed with diabetes. Based on the fact that the child's pancreatic beta cells are being destroyed, the patient would be diagnosed with what type of diabetes? A) Type 1 diabetes B) Type 2 diabetes C) Non-insulin-dependent diabetes D) Prediabetes

: A Feedback: Beta cell destruction is the hallmark of type 1 diabetes. Non-insulin-dependent diabetes is synonymous with type 2 diabetes, which involves insulin resistance and impaired insulin secretion, but not beta cell destruction. Prediabetes is characterized by normal glucose metabolism, but a previous history of hyperglycemia, often during illness or pregnancy.

37. The most recent blood work of a patient with a longstanding diagnosis of type 1 diabetes has shown the presence of microalbuminuria. What is the nurse's most appropriate action? A) Teach the patient about actions to slow the progression of nephropathy. B) Ensure that the patient receives a comprehensive assessment of liver function. C) Determine whether the patient has been using expired insulin. D) Administer a fluid challenge and have the test repeated.

: A Feedback: Clinical nephropathy eventually develops in more than 85% of people with microalbuminuria. As such, educational interventions addressing this microvascular complication are warranted. Expired insulin does not cause nephropathy, and the patient's liver function is not likely affected. There is no indication for the use of a fluid challenge.

2. A patient presents to the clinic complaining of symptoms that suggest diabetes. What criteria would support checking blood levels for the diagnosis of diabetes? A) Fasting plasma glucose greater than or equal to 126 mg/dL B) Random plasma glucose greater than 150 mg/dL C) Fasting plasma glucose greater than 116 mg/dL on 2 separate occasions D) Random plasma glucose greater than 126 mg/dL

: A Feedback: Criteria for the diagnosis of diabetes include symptoms of diabetes plus random plasma glucose greater than or equal to 200 mg/dL, or a fasting plasma glucose greater than or equal to 126 mg/dL.

26. A patient with a longstanding diagnosis of type 1 diabetes has a history of poor glycemic control. The nurse recognizes the need to assess the patient for signs and symptoms of peripheral neuropathy. Peripheral neuropathy constitutes a risk for what nursing diagnosis? A) Infection B) Acute pain C) Acute confusion D) Impaired urinary elimination

: A Feedback: Decreased sensations of pain and temperature place patients with neuropathy at increased risk for injury and undetected foot infections. The neurologic changes associated with peripheral neuropathy do not normally result in pain, confusion, or impairments in urinary function.

30. A nurse is conducting a class on how to self-manage insulin regimens. A patient asks how long a vial of insulin can be stored at room temperature before it "goes bad." What would be the nurse's best answer? A) "If you are going to use up the vial within 1 month it can be kept at room temperature." B) "If a vial of insulin will be used up within 21 days, it may be kept at room temperature." C) "If a vial of insulin will be used up within 2 weeks, it may be kept at room temperature." D) "If a vial of insulin will be used up within 1 week, it may be kept at room temperature."

: A Feedback: If a vial of insulin will be used up within 1 month, it may be kept at room temperature.

27. A patient has been brought to the emergency department by paramedics after being found unconscious. The patient's Medic Alert bracelet indicates that the patient has type 1 diabetes and the patient's blood glucose is 22 mg/dL (1.2 mmol/L). The nurse should anticipate what intervention? A) IV administration of 50% dextrose in water B) Subcutaneous administration of 10 units of Humalog C) Subcutaneous administration of 12 to 15 units of regular insulin D) IV bolus of 5% dextrose in 0.45% NaCl

: A Feedback: In hospitals and emergency departments, for patients who are unconscious or cannot swallow, 25 to 50 mL of 50% dextrose in water (D50W) may be administered IV for the treatment of hypoglycemia. Five percent dextrose would be inadequate and insulin would exacerbate the patient's condition.

34. A patient has been living with type 2 diabetes for several years, and the nurse realizes that the patient is likely to have minimal contact with the health care system. In order to ensure that the patient maintains adequate blood sugar control over the long term, the nurse should recommend which of the following? A) Participation in a support group for persons with diabetes B) Regular consultation of websites that address diabetes management C) Weekly telephone "check-ins" with an endocrinologist D) Participation in clinical trials relating to antihyperglycemics

: A Feedback: Participation in support groups is encouraged for patients who have had diabetes for many years as well as for those who are newly diagnosed. This is more interactive and instructive than simply consulting websites. Weekly telephone contact with an endocrinologist is not realistic in most cases. Participation in research trials may or may not be beneficial and appropriate, depending on patients' circumstances.

28. A diabetic nurse is working for the summer at a camp for adolescents with diabetes. When providing information on the prevention and management of hypoglycemia, what action should the nurse promote? A) Always carry a form of fast-acting sugar. B) Perform exercise prior to eating whenever possible. C) Eat a meal or snack every 8 hours. D) Check blood sugar at least every 24 hours.

: A Feedback: The following teaching points should be included in information provided to the patient on how to prevent hypoglycemia: Always carry a form of fast-acting sugar, increase food prior to exercise, eat a meal or snack every 4 to 5 hours, and check blood sugar regularly.

7. A diabetes nurse educator is teaching a group of patients with type 1 diabetes about "sick day rules." What guideline applies to periods of illness in a diabetic patient? A) Do not eliminate insulin when nauseated and vomiting. B) Report elevated glucose levels greater than 150 mg/dL. C) Eat three substantial meals a day, if possible. D) Reduce food intake and insulin doses in times of illness.

: A Feedback: The most important issue to teach patients with diabetes who become ill is not to eliminate insulin doses when nausea and vomiting occur. Rather, they should take their usual insulin or oral hypoglycemic agent dose, then attempt to consume frequent, small portions of carbohydrates. In general, blood sugar levels will rise but should be reported if they are greater than 300 mg/dL.

18. A diabetic educator is discussing "sick day rules" with a newly diagnosed type 1 diabetic. The educator is aware that the patient will require further teaching when the patient states what? A) "I will not take my insulin on the days when I am sick, but I will certainly check my blood sugar every 2 hours." B) "If I cannot eat a meal, I will eat a soft food such as soup, gelatin, or pudding six to eight times a day." C) "I will call the doctor if I am not able to keep liquids in my body due to vomiting or diarrhea." D) "I will call the doctor if my blood sugar is over 300 mg/dL or if I have ketones in my urine."

: A Feedback: The nurse must explanation the "sick day rules" again to the patient who plans to stop taking insulin when sick. The nurse should emphasize that the patient should take insulin agents as usual and test one's blood sugar and urine ketones every 3 to 4 hours. In fact, insulin-requiring patients may need supplemental doses of regular insulin every 3 to 4 hours. The patient should report elevated glucose levels (greater than 300 mg/dL or as otherwise instructed) or urine ketones to the physician. If the patient is not able to eat normally, the patient should be instructed to substitute soft foods such a gelatin, soup, and pudding. If vomiting, diarrhea, or fever persists, the patient should have an intake of liquids every 30 to 60 minutes to prevent dehydration.

22. A nurse is caring for a patient newly diagnosed with type 1 diabetes. The nurse is educating the patient about self-administration of insulin in the home setting. The nurse should teach the patient to do which of the following? A) Avoid using the same injection site more than once in 2 to 3 weeks. B) Avoid mixing more than one type of insulin in a syringe. C) Cleanse the injection site thoroughly with alcohol prior to injecting. D) Inject at a 45º angle.

: A Feedback: To prevent lipodystrophy, the patient should try not to use the same site more than once in 2 to 3 weeks. Mixing different types of insulin in a syringe is acceptable, within specific guidelines, and the needle is usually inserted at a 90º angle. Cleansing the injection site with alcohol is optional.

12. An occupational health nurse is screening a group of workers for diabetes. What statement should the nurse interpret as suggestive of diabetes? A) "I've always been a fan of sweet foods, but lately I'm turned off by them." B) "Lately, I drink and drink and can't seem to quench my thirst." C) "No matter how much sleep I get, it seems to take me hours to wake up." D) "When I went to the washroom the last few days, my urine smelled odd."

: B Feedback: Classic clinical manifestations of diabetes include the "three Ps": polyuria, polydipsia, and polyphagia. Lack of interest in sweet foods, fatigue, and foul-smelling urine are not suggestive of diabetes.

23. A patient with type 2 diabetes achieves adequate glycemic control through diet and exercise. Upon being admitted to the hospital for a cholecystectomy, however, the patient has required insulin injections on two occasions. The nurse would identify what likely cause for this short-term change in treatment? A) Alterations in bile metabolism and release have likely caused hyperglycemia. B) Stress has likely caused an increase in the patient's blood sugar levels. C) The patient has likely overestimated her ability to control her diabetes using nonpharmacologic measures. D) The patient's volatile fluid balance surrounding surgery has likely caused unstable blood sugars.

: B Feedback: During periods of physiologic stress, such as surgery, blood glucose levels tend to increase, because levels of stress hormones (epinephrine, norepinephrine, glucagon, cortisol, and growth hormone) increase. The patient's need for insulin is unrelated to the action of bile, the patient's overestimation of previous blood sugar control, or fluid imbalance.

39. A diabetic patient calls the clinic complaining of having a "flu bug." The nurse tells him to take his regular dose of insulin. What else should the nurse tell the patient? A) "Make sure to stick to your normal diet." B) "Try to eat small amounts of carbs, if possible." C) "Ensure that you check your blood glucose every hour." D) "For now, check your urine for ketones every 8 hours."

: B Feedback: For prevention of DKA related to illness, the patient should attempt to consume frequent small portions of carbohydrates (including foods usually avoided, such as juices, regular sodas, and gelatin). Drinking fluids every hour is important to prevent dehydration. Blood glucose and urine ketones must be assessed every 3 to 4 hours.

33. A patient with a history of type 1 diabetes has just been admitted to the critical care unit (CCU) for diabetic ketoacidosis. The CCU nurse should prioritize what assessment during the patient's initial phase of treatment? A) Monitoring the patient for dysrhythmias B) Maintaining and monitoring the patient's fluid balance C) Assessing the patient's level of consciousness D) Assessing the patient for signs and symptoms of venous thromboembolism

: B Feedback: In addition to treating hyperglycemia, management of DKA is aimed at correcting dehydration, electrolyte loss, and acidosis before correcting the hyperglycemia with insulin. The nurse should monitor the patient for dysrhythmias, decreased LOC and VTE, but restoration and maintenance of fluid balance is the highest priority.

8. The nurse is discussing macrovascular complications of diabetes with a patient. The nurse would address what topic during this dialogue? A) The need for frequent eye examinations for patients with diabetes B) The fact that patients with diabetes have an elevated risk of myocardial infarction C) The relationship between kidney function and blood glucose levels D) The need to monitor urine for the presence of albumin

: B Feedback: Myocardial infarction and stroke are considered macrovascular complications of diabetes, while the effects on vision and renal function are considered to be microvascular.

16. A nurse is caring for a patient with type 1 diabetes who is being discharged home tomorrow. What is the best way to assess the patient's ability to prepare and self-administer insulin? A) Ask the patient to describe the process in detail. B) Observe the patient drawing up and administering the insulin. C) Provide a health education session reviewing the main points of insulin delivery. D) Review the patient's first hemoglobin A1C result after discharge.

: B Feedback: Nurses should assess the patient's ability to perform diabetes related self-care as soon as possible during the hospitalization or office visit to determine whether the patient requires further diabetes teaching. While consulting a home care nurse is beneficial, an initial assessment should be performed during the hospitalization or office visit. Nurses should directly observe the patient performing the skills such as insulin preparation and infection, blood glucose monitoring, and foot care. Simply questioning the patient about these skills without actually observing performance of the skill is not sufficient. Further education does not guarantee learning.

5. A medical nurse is caring for a patient with type 1 diabetes. The patient's medication administration record includes the administration of regular insulin three times daily. Knowing that the patient's lunch tray will arrive at 11:45, when should the nurse administer the patient's insulin? A) 10:45 B) 11:15 C) 11:45 D) 11:50

: B Feedback: Regular insulin is usually administered 20-30 min before a meal. Earlier administration creates a risk for hypoglycemia; later administration creates a risk for hyperglycemia.

36. A diabetes nurse is assessing a patient's knowledge of self-care skills. What would be the most appropriate way for the educator to assess the patient's knowledge of nutritional therapy in diabetes? A) Ask the patient to describe an optimally healthy meal. B) Ask the patient to keep a food diary and review it with the nurse. C) Ask the patient's family what he typically eats. D) Ask the patient to describe a typical day's food intake.

: B Feedback: Reviewing the patient's actual food intake is the most accurate method of gauging the patient's diet.

6. A patient has just been diagnosed with type 2 diabetes. The physician has prescribed an oral antidiabetic agent that will inhibit the production of glucose by the liver and thereby aid in the control of blood glucose. What type of oral antidiabetic agent did the physician prescribe for this patient? A) A sulfonylurea B) A biguanide C) A thiazolidinedione D) An alpha glucosidase inhibitor

: B Feedback: Sulfonylureas exert their primary action by directly stimulating the pancreas to secrete insulin and therefore require a functioning pancreas to be effective. Biguanides inhibit the production of glucose by the liver and are in used in type 2 diabetes to control blood glucose levels. Thiazolidinediones enhance insulin action at the receptor site without increasing insulin secretion from the beta cells of the pancreas. Alpha glucosidase inhibitors work by delaying the absorption of glucose in the intestinal system, resulting in a lower postprandial blood glucose level.

17. An elderly patient comes to the clinic with her daughter. The patient is a diabetic and is concerned about foot care. The nurse goes over foot care with the patient and her daughter as the nurse realizes that foot care is extremely important. Why would the nurse feel that foot care is so important to this patient? A) An elderly patient with foot ulcers experiences severe foot pain due to the diabetic polyneuropathy. B) Avoiding foot ulcers may mean the difference between institutionalization and continued independent living. C) Hypoglycemia is linked with a risk for falls; this risk is elevated in older adults with diabetes. D) Oral antihyperglycemics have the possible adverse effect of decreased circulation to the lower extremities.

: B Feedback: The nurse recognizes that providing information on the long-term complications—especially foot and eye problems—associated with diabetes is important. Avoiding amputation through early detection of foot ulcers may mean the difference between institutionalization and continued independent living for the elderly person with diabetes. While the nurse recognizes that hypoglycemia is a dangerous situation and may lead to falls, hypoglycemia is not directly connected to the importance of foot care. Decrease in circulation is related to vascular changes and is not associated with drugs administered for diabetes.

40. A patient is brought to the emergency department by the paramedics. The patient is a type 2 diabetic and is experiencing HHS. The nurse should identify what components of HHS? Select all that apply. A) Leukocytosis B) Glycosuria C) Dehydration D) Hypernatremia E) Hyperglycemia

: B, C, D, E Feedback: In HHS, persistent hyperglycemia causes osmotic diuresis, which results in losses of water and electrolytes. To maintain osmotic equilibrium, water shifts from the intracellular fluid space to the extracellular fluid space. With glycosuria and dehydration, hypernatremia and increased osmolarity occur. Leukocytosis does not take place.

3. A patient newly diagnosed with type 2 diabetes is attending a nutrition class. What general guideline would be important to teach the patients at this class? A) Low fat generally indicates low sugar. B) Protein should constitute 30% to 40% of caloric intake. C) Most calories should be derived from carbohydrates. D) Animal fats should be eliminated from the diet.

: C Feedback: Currently, the ADA and the Academy of Nutrition and Dietetics (formerly the American Dietetic Association) recommend that for all levels of caloric intake, 50% to 60% of calories should be derived from carbohydrates, 20% to 30% from fat, and the remaining 10% to 20% from protein.Low fat does not automatically mean low sugar. Dietary animal fat does not need to be eliminated from the diet.

4. A nurse is providing health education to an adolescent newly diagnosed with type 1 diabetes mellitus and her family. The nurse teaches the patient and family that which of the following nonpharmacologic measures will decrease the body's need for insulin? A) Adequate sleep B) Low stimulation C) Exercise D) Low-fat diet

: C Feedback: Exercise lowers blood glucose, increases levels of HDLs, and decreases total cholesterol and triglyceride levels. Low fat intake and low levels of stimulation do not reduce a patient's need for insulin. Adequate sleep is beneficial in reducing stress, but does not have an effect that is pronounced as that of exercise.

35. A patient with type 1 diabetes mellitus is seeing the nurse to review foot care. What would be a priority instruction for the nurse to give the patient? A) Examine feet weekly for redness, blisters, and abrasions. B) Avoid the use of moisturizing lotions. C) Avoid hot-water bottles and heating pads. D) Dry feet vigorously after each bath.

: C Feedback: High-risk behaviors, such as walking barefoot, using heating pads on the feet, wearing open-toed shoes, soaking the feet, and shaving calluses, should be avoided. Socks should be worn for warmth. Feet should be examined each day for cuts, blisters, swelling, redness, tenderness, and abrasions. Lotion should be applied to dry feet but never between the toes. After a bath, the patient should gently, not vigorously, pat feet dry to avoid injury.

20. A 28-year-old pregnant woman is spilling sugar in her urine. The physician orders a glucose tolerance test, which reveals gestational diabetes. The patient is shocked by the diagnosis, stating that she is conscientious about her health, and asks the nurse what causes gestational diabetes. The nurse should explain that gestational diabetes is a result of what etiologic factor? A) Increased caloric intake during the first trimester B) Changes in osmolality and fluid balance C) The effects of hormonal changes during pregnancy D) Overconsumption of carbohydrates during the first two trimesters

: C Feedback: Hyperglycemia and eventual gestational diabetes develops during pregnancy because of the secretion of placental hormones, which causes insulin resistance. The disease is not the result of food intake or changes in osmolality.

32. A student with diabetes tells the school nurse that he is feeling nervous and hungry. The nurse assesses the child and finds he has tachycardia and is diaphoretic with a blood glucose level of 50 mg/dL (2.8 mmol/L). What should the school nurse administer? A) A combination of protein and carbohydrates, such as a small cup of yogurt B) Two teaspoons of sugar dissolved in a cup of apple juice C) Half of a cup of juice, followed by cheese and crackers D) Half a sandwich with a protein-based filling

: C Feedback: Initial treatment for hypoglycemia is 15 g concentrated carbohydrate, such as two or three glucose tablets, 1 tube glucose gel, or 0.5 cup juice. After initial treatment, the nurse should follow with a snack including starch and protein, such as cheese and crackers, milk and crackers, or half of a sandwich. It is unnecessary to add sugar to juice, even it if is labeled as unsweetened juice, because the fruit sugar in juice contains enough simple carbohydrate to raise the blood glucose level and additional sugar may result in a sharp rise in blood sugar that will last for several hours.

13. A diabetes educator is teaching a patient about type 2 diabetes. The educator recognizes that the patient understands the primary treatment for type 2 diabetes when the patient states what? A) "I read that a pancreas transplant will provide a cure for my diabetes." B) "I will take my oral antidiabetic agents when my morning blood sugar is high." C) "I will make sure to follow the weight loss plan designed by the dietitian." D) "I will make sure I call the diabetes educator when I have questions about my insulin."

: C Feedback: Insulin resistance is associated with obesity; thus the primary treatment of type 2 diabetes is weight loss. Oral antidiabetic agents may be added if diet and exercise are not successful in controlling blood glucose levels. If maximum doses of a single category of oral agents fail to reduce glucose levels to satisfactory levels, additional oral agents may be used. Some patients may require insulin on an ongoing basis or on a temporary basis during times of acute psychological stress, but it is not the central component of type 2 treatment. Pancreas transplantation is associated with type 1 diabetes.

1. A patient with type 1 diabetes has told the nurse that his most recent urine test for ketones was positive. What is the nurse's most plausible conclusion based on this assessment finding? A) The patient should withhold his next scheduled dose of insulin. B) The patient should promptly eat some protein and carbohydrates. C) The patient's insulin levels are inadequate. D) The patient would benefit from a dose of metformin (Glucophage).

: C Feedback: Ketones in the urine signal that there is a deficiency of insulin and that control of type 1 diabetes is deteriorating. Withholding insulin or eating food would exacerbate the patient's ketonuria. Metformin will not cause short-term resolution of hyperglycemia.

24. A physician has explained to a patient that he has developed diabetic neuropathy in his right foot. Later that day, the patient asks the nurse what causes diabetic neuropathy. What would be the nurse's best response? A) "Research has shown that diabetic neuropathy is caused by fluctuations in blood sugar that have gone on for years." B) "The cause is not known for sure but it is thought to have something to do with ketoacidosis." C) "The cause is not known for sure but it is thought to involve elevated blood glucose levels over a period of years." D) "Research has shown that diabetic neuropathy is caused by a combination of elevated glucose levels and elevated ketone levels."

: C Feedback: The etiology of neuropathy may involve elevated blood glucose levels over a period of years. High blood sugars (rather than fluctuations or variations in blood sugars) are thought to be responsible. Ketones and ketoacidosis are not direct causes of neuropathies.

19. Which of the following patients with type 1 diabetes is most likely to experience adequate glucose control? A) A patient who skips breakfast when his glucose reading is greater than 220 mg/dL B) A patient who never deviates from her prescribed dose of insulin C) A patient who adheres closely to a meal plan and meal schedule D) A patient who eliminates carbohydrates from his daily intake

: C Feedback: The therapeutic goal for diabetes management is to achieve normal blood glucose levels without hypoglycemia. Therefore, diabetes management involves constant assessment and modification of the treatment plan by health professionals and daily adjustments in therapy (possibly including insulin) by patients. For patients who require insulin to help control blood glucose levels, maintaining consistency in the amount of calories and carbohydrates ingested at meals is essential. In addition, consistency in the approximate time intervals between meals, and the snacks, help maintain overall glucose control. Skipping meals is never advisable for person with type 1 diabetes.

38. A nurse is assessing a patient who has diabetes for the presence of peripheral neuropathy. The nurse should question the patient about what sign or symptom that would suggest the possible development of peripheral neuropathy? A) Persistently cold feet B) Pain that does not respond to analgesia C) Acute pain, unrelieved by rest D) The presence of a tingling sensation

: D Feedback: Although approximately half of patients with diabetic neuropathy do not have symptoms, initial symptoms may include paresthesias (prickling, tingling, or heightened sensation) and burning sensations (especially at night). Cold and intense pain are atypical early signs of this complication.

31. A patient has received a diagnosis of type 2 diabetes. The diabetes nurse has made contact with the patient and will implement a program of health education. What is the nurse's priority action? A) Ensure that the patient understands the basic pathophysiology of diabetes. B) Identify the patient's body mass index. C) Teach the patient "survival skills" for diabetes. D) Assess the patient's readiness to learn.

: D Feedback: Before initiating diabetes education, the nurse assesses the patient's (and family's) readiness to learn. This must precede other physiologic assessments (such as BMI) and providing health education.

14. A diabetes nurse educator is presenting the American Diabetes Association (ADA) recommendations for levels of caloric intake. What do the ADA's recommendations include? A) 10% of calories from carbohydrates, 50% from fat, and the remaining 40% from protein B) 10% to 20% of calories from carbohydrates, 20% to 30% from fat, and the remaining 50% to 60% from protein C) 20% to 30% of calories from carbohydrates, 50% to 60% from fat, and the remaining 10% to 20% from protein D) 50% to 60% of calories from carbohydrates, 20% to 30% from fat, and the remaining 10% to 20% from protein

: D Feedback: Currently, the ADA and the Academy of Nutrition and Dietetics (formerly the American Dietetic Association) recommend that for all levels of caloric intake, 50% to 60% of calories come from carbohydrates, 20% to 30% from fat, and the remaining 10% to 20% from protein.

21. A medical nurse is aware of the need to screen specific patients for their risk of hyperglycemic hyperosmolar syndrome (HHS). In what patient population does hyperosmolar nonketotic syndrome most often occur? A) Patients who are obese and who have no known history of diabetes B) Patients with type 1 diabetes and poor dietary control C) Adolescents with type 2 diabetes and sporadic use of antihyperglycemics D) Middle-aged or older people with either type 2 diabetes or no known history of diabetes

: D Feedback: HHS occurs most often in older people (50 to 70 years of age) who have no known history of diabetes or who have type 2 diabetes.

29. A nurse is teaching basic "survival skills" to a patient newly diagnosed with type 1 diabetes. What topic should the nurse address? A) Signs and symptoms of diabetic nephropathy B) Management of diabetic ketoacidosis C) Effects of surgery and pregnancy on blood sugar levels D) Recognition of hypoglycemia and hyperglycemia

: D Feedback: It is imperative that newly diagnosed patients know the signs and symptoms and management of hypo- and hyperglycemia. The other listed topics are valid points for education, but are not components of the patient's immediate "survival skills" following a new diagnosis.

25. A patient with type 2 diabetes has been managing his blood glucose levels using diet and metformin (Glucophage). Following an ordered increase in the patient's daily dose of metformin, the nurse should prioritize which of the following assessments? A) Monitoring the patient's neutrophil levels B) Assessing the patient for signs of impaired liver function C) Monitoring the patient's level of consciousness and behavior D) Reviewing the patient's creatinine and BUN levels

: D Feedback: Metformin has the potential to be nephrotoxic; consequently, the nurse should monitor the patient's renal function. This drug does not typically affect patients' neutrophils, liver function, or cognition.

9. A school nurse is teaching a group of high school students about risk factors for diabetes. Which of the following actions has the greatest potential to reduce an individual's risk for developing diabetes? A) Have blood glucose levels checked annually. B) Stop using tobacco in any form. C) Undergo eye examinations regularly. D) Lose weight, if obese.

: D Feedback: Obesity is a major modifiable risk factor for diabetes. Smoking is not a direct risk factor for the disease. Eye examinations are necessary for persons who have been diagnosed with diabetes, but they do not screen for the disease or prevent it. Similarly, blood glucose checks do not prevent the diabetes.

15. An older adult patient with type 2 diabetes is brought to the emergency department by his daughter. The patient is found to have a blood glucose level of 623 mg/dL. The patient's daughter reports that the patient recently had a gastrointestinal virus and has been confused for the last 3 hours. The diagnosis of hyperglycemic hyperosmolar syndrome (HHS) is made. What nursing action would be a priority? A) Administration of antihypertensive medications B) Administering sodium bicarbonate intravenously C) Reversing acidosis by administering insulin D) Fluid and electrolyte replacement

: D Feedback: The overall approach to HHS includes fluid replacement, correction of electrolyte imbalances, and insulin administration. Antihypertensive medications are not indicated, as hypotension generally accompanies HHS due to dehydration. Sodium bicarbonate is not administered to patients with HHS, as their plasma bicarbonate level is usually normal. Insulin administration plays a less important role in the treatment of HHS because it is not needed for reversal of acidosis, as in diabetic ketoacidosis (DKA).

11. A newly admitted patient with type 1 diabetes asks the nurse what caused her diabetes. When the nurse is explaining to the patient the etiology of type 1 diabetes, what process should the nurse describe? A) "The tissues in your body are resistant to the action of insulin, making the glucose levels in your blood increase." B) "Damage to your pancreas causes an increase in the amount of glucose that it releases, and there is not enough insulin to control it." C) "The amount of glucose that your body makes overwhelms your pancreas and decreases your production of insulin." D) "Destruction of special cells in the pancreas causes a decrease in insulin production. Glucose levels rise because insulin normally breaks it down."

: D Feedback: Type 1 diabetes is characterized by the destruction of pancreatic beta cells, resulting in decreased insulin production, unchecked glucose production by the liver, and fasting hyperglycemia. Also, glucose derived from food cannot be stored in the liver and remains circulating in the blood, which leads to postprandial hyperglycemia. Type 2 diabetes involves insulin resistance and impaired insulin secretion. The body does not "make" glucose.

A patient who just suffered a suspected ischemic stroke is brought to the ED by ambulance. On what should the nurse's primary assessment focus? A) Cardiac and respiratory status B) Seizure activity C) Pain D) Fluid and electrolyte balance

A (Feedback: Acute care begins with managing ABCs. Patients may have difficulty keeping an open and clear airway secondary to decreased LOC. Neurologic assessment with close monitoring for signs of increased neurologic deficit and seizure activity occurs next. Fluid and electrolyte balance must be controlled carefully with the goal of adequate hydration to promote perfusion and decrease further brain activity.)

A patient who has experienced an ischemic stroke has been admitted to the medical unit. The patient's family in adamant that she remains on bed rest to hasten her recovery and to conserve energy. What principle of care should inform the nurse's response to the family? A) The patient should mobilize as soon as she is physically able. B) To prevent contractures and muscle atrophy, bed rest should not exceed 4 weeks. C) The patient should remain on bed rest until she expresses a desire to mobilize. D) Lack of mobility will greatly increase the patient's risk of stroke recurrence.

A (Feedback: As soon as possible, the patient is assisted out of bed and an active rehabilitation program is started. Delaying mobility causes complications, but not necessarily stroke recurrence. Mobility should not be withheld until the patient initiates.)

When preparing to discharge a patient home, the nurse has met with the family and warned them that the patient may exhibit unexpected emotional responses. The nurse should teach the family that these responses are typically a result of what cause? A) Frustration around changes in function and communication B) Unmet physiologic needs C) Changes in brain activity during sleep and wakefulness D) Temporary changes in metabolism

A (Feedback: Emotional problems associated with stroke are often related to the new challenges around ADLs and communication. These challenges are more likely than metabolic changes, unmet physiologic needs, or changes in brain activity, each of which should be ruled out.)

The nurse is assessing a patient with a suspected stroke. What assessment finding is most suggestive of a stroke? A) Facial droop B) Dysrhythmias C) Periorbital edema D) Projectile vomiting

A (Feedback: Facial drooping or asymmetry is a classic abnormal finding on a physical assessment that may be associated with a stroke. Facial edema is not suggestive of a stroke and patients less commonly experience dysrhythmias or vomiting.)

After a major ischemic stroke, a possible complication is cerebral edema. Nursing care during the immediate recovery period from an ischemic stroke should include which of the following? A) Positioning to avoid hypoxia B) Maximizing PaCO2 C) Administering hypertonic IV solution D) Initiating early mobilization

A (Feedback: Interventions during this period include measures to reduce ICP, such as administering an osmotic diuretic (e.g., mannitol), maintaining the partial pressure of carbon dioxide (PaCO2) within the range of 30 to 35 mm Hg, and positioning to avoid hypoxia. Hypertonic IV solutions are not used unless sodium depletion is evident. Mobilization would take place after the immediate threat of increased ICP has past.)

A patient who suffered an ischemic stroke now has disturbed sensory perception. What principle should guide the nurse's care of this patient? A) The patient should be approached on the side where visual perception is intact. B) Attention to the affected side should be minimized in order to decrease anxiety. C) The patient should avoid turning in the direction of the defective visual field to minimize shoulder subluxation. D) The patient should be approached on the opposite side of where the visual perception is intact to promote recovery.

A (Feedback: Patients with decreased field of vision should first be approached on the side where visual perception is intact. All visual stimuli should be placed on this side. The patient can and should be taught to turn the head in the direction of the defective visual field to compensate for this loss. The nurse should constantly remind the patient of the other side of the body and should later stand at a position that encourages the patient to move or turn to visualize who and what is in the room.)

The patient has been diagnosed with aphasia after suffering a stroke. What can the nurse do to best make the patient's atmosphere more conducive to communication? A) Provide a board of commonly used needs and phrases. B) Have the patient speak to loved ones on the phone daily. C) Help the patient complete his or her sentences. D) Speak in a loud and deliberate voice to the patient.

A (Feedback: The inability to talk on the telephone or answer a question or exclusion from conversation causes anger, frustration, fear of the future, and hopelessness. A common pitfall is for the nurse or other health care team member to complete the thoughts or sentences of the patient. This should be avoided because it may cause the patient to feel more frustrated at not being allowed to speak and may deter efforts to practice putting thoughts together and completing a sentence. The patient may also benefit from a communication board, which has pictures of commonly requested needs and phrases. The board may be translated into several languages.)

Following diagnostic testing, a patient has been admitted to the ICU and placed on cerebral aneurysm precautions. What nursing action should be included in patient's plan of care? A) Supervise the patient's activities of daily living closely. B) Initiate early ambulation to prevent complications of immobility. C) Provide a high-calorie, low-protein diet. D) Perform all of the patient's hygiene and feeding.

A (Feedback: The patient is placed on immediate and absolute bed rest in a quiet, nonstressful environment, because activity, pain, and anxiety elevate BP, which increases the risk for bleeding. As such, independent ADLs and ambulation are contraindicated. There is no need for a high-calorie or low-protein diet.)

A patient is brought by ambulance to the ED after suffering what the family thinks is a stroke. The nurse caring for this patient is aware that an absolute contraindication for thrombolytic therapy is what? A) Evidence of hemorrhagic stroke B) Blood pressure of ³ 180/110 mm Hg C) Evidence of stroke evolution D) Previous thrombolytic therapy within the past 12 months

A (Feedback: Thrombolytic therapy would exacerbate a hemorrhagic stroke with potentially fatal consequences. Stroke evolution, high BP, or previous thrombolytic therapy does not contraindicate its safe and effective use.)

21. A patient with suspected adrenal insufficiency has been ordered an adrenocorticotropic hormone (ACTH) stimulation test. Administration of ACTH caused a marked increase in cortisol levels. How should the nurse interpret this finding? A) The patients pituitary function is compromised. B) The patients adrenal insufficiency is not treatable. C) The patient has insufficient hypothalamic function. D) The patient would benefit from surgery

A Feedback: An adrenal response to the administration of a stimulating hormone suggests inadequate production of the stimulating hormone. In this case, ACTH is produced by the pituitary and, consequently, pituitary hypofunction is suggested. Hypothalamic function is not relevant to the physiology of this problem. Treatment exists, although surgery is not likely indicated.

3. A patient with thyroid cancer has undergone surgery and a significant amount of parathyroid tissue has been removed. The nurse caring for the patient should prioritize what question when addressing potential complications? A) Do you feel any muscle twitches or spasms? B) Do you feel flushed or sweaty? C) Are you experiencing any dizziness or lightheadedness? D) Are you having any pain that seems to be radiating from your bones?

A Feedback: As the blood calcium level falls, hyperirritability of the nerves occurs, with spasms of the hands and feet and muscle twitching. This is characteristic of hypoparathyroidism. Flushing, diaphoresis, dizziness, and pain are atypical signs of the resulting hypocalcemia.

35. A patient with Cushing syndrome as a result of a pituitary tumor has been admitted for a transsphenoidal hypophysectomy. What would be most important for the nurse to monitor before, during, and after surgery? A) Blood glucose B) Assessment of urine for blood C) Weight D) Oral temperature

A Feedback: Before, during, and after this surgery, blood glucose monitoring and assessment of stools for blood are carried out. The patients blood sugar is more likely to be volatile than body weight or temperature. Hematuria is not a common complication.

28. Following an addisonian crisis, a patients adrenal function has been gradually regained. The nurse should ensure that the patient knows about the need for supplementary glucocorticoid therapy in which of the following circumstances? A) Episodes of high psychosocial stress B) Periods of dehydration C) Episodes of physical exertion D) Administration of a vaccine

A Feedback: During stressful procedures or significant illnesses, additional supplementary therapy with glucocorticoids is required to prevent addisonian crisis. Physical activity, dehydration and vaccine administration would not normally be sufficiently demanding such to require glucocorticoids.

23. A nurse works in a walk-in clinic. The nurse recognizes that certain patients are at higher risk for different disorders than other patients. What patient is at a greater risk for the development of hypothyroidism? A) A 75-year-old female patient with osteoporosis B) A 50-year-old male patient who is obese C) A 45-year-old female patient who used oral contraceptives D) A 25-year-old male patient who uses recreational drugs

A Feedback: Even though osteoporosis is not a risk factor for hypothyroidism, the condition occurs most frequently in older women.

1. The nurse is caring for a patient diagnosed with hypothyroidism secondary to Hashimotos thyroiditis. When assessing this patient, what sign or symptom would the nurse expect? A)Fatigue B) Bulging eyes C) Palpitations D) Flushed skin

A Feedback: Symptoms of hypothyroidism include extreme fatigue, hair loss, brittle nails, dry skin, voice huskiness or hoarseness, menstrual disturbance, and numbness and tingling of the fingers. Bulging eyes, palpitations, and flushed skin would be signs and symptoms of hyperthyroidism.

32. The nurse providing care for a patient with Cushing syndrome has identified the nursing diagnosis of risk for injury related to weakness. How should the nurse best reduce this risk? A) Establish falls prevention measures. B) Encourage bed rest whenever possible. C) Encourage the use of assistive devices. D) Provide constant supervision.

A Feedback: The nurse should take action to prevent the patients risk for falls. Bed rest carries too many harmful effects, however, and assistive devices may or may not be necessary. Constant supervision is not normally required or practicable.

10. You are developing a care plan for a patient with Cushing syndrome. What nursing diagnosis would have the highest priority in this care plan? A) Risk for injury related to weakness B) Ineffective breathing pattern related to muscle weakness C) Risk for loneliness related to disturbed body image D) Autonomic dysreflexia related to neurologic changes

A Feedback: The nursing priority is to decrease the risk of injury by establishing a protective environment. The patient who is weak may require assistance from the nurse in ambulating to prevent falls or bumping corners or furniture. The patients breathing will not be affected and autonomic dysreflexia is not a plausible risk. Loneliness may or may not be an issue for the patient, but safety is a priority.

13. A patient presents at the walk-in clinic complaining of diarrhea and vomiting. The patient has a documented history of adrenal insufficiency. Considering the patients history and current symptoms, the nurse should anticipate that the patient will be instructed to do which of the following? A) Increase his intake of sodium until the GI symptoms improve. B) Increase his intake of potassium until the GI symptoms improve. C) Increase his intake of glucose until the GI symptoms improve. D) Increase his intake of calcium until the GI symptoms improve.

A Feedback: The patient will need to supplement dietary intake with added salt during episodes of GI losses of fluid through vomiting and diarrhea to prevent the onset of addisonian crisis. While the patient may experience the loss of other electrolytes, the major concern is the replacement of lost sodium.

31. A patient is undergoing testing for suspected adrenocortical insufficiency. The care team should ensure that the patient has been assessed for the most common cause of adrenocortical insufficiency. What is the most common cause of this health problem? A) Therapeutic use of corticosteroids B) Pheochromocytoma C) Inadequate secretion of ACTH D) Adrenal tumor

A Feedback: Therapeutic use of corticosteroids is the most common cause of adrenocortical insufficiency. The other options also cause adrenocortical insufficiency, but they are not the most common causes.

Ans: Assessment for pain Feedback: Bone biopsy can be painful and the nurse should prioritize relevant assessments. Dehiscence is not a possibility, since the incision is not linear. Signs and symptoms of infection would not be evident in the immediate recovery period and hematoma formation is not a common complication.

A bone biopsy has just been completed on a patient with suspected bone metastases. What assessment should the nurse prioritize in the immediate recovery period?

Ans: Osteoblasts Feedback: Osteoblasts function in bone formation by secreting bone matrix. Osteocytes are mature bone cells and osteoclasts are multinuclear cells involved in dissolving and resorbing bone. Lamellae are circles of mineralized bone matrix.

A child is growing at a rate appropriate for his age. What cells are responsible for the secretion of bone matrix that eventually results in bone growth?

Ans: Bone densitometry Feedback: Bone densitometry is used to detect bone density and can be used to assess the risk of fracture in osteoporosis. Arthrography is used to detect acute or chronic tears of joint capsule or supporting ligaments. Bone scans can be used to detect metastatic and primary bone tumors, osteomyelitis, certain fractures, and aseptic necrosis. Arthroscopy is used to visualize a joint.

A clinic nurse is caring for a patient with a history of osteoporosis. Which of the following diagnostic tests best allows the care team to assess the patient's risk of fracture?

Ans: Spasticity Feedback: A muscle with greater-than-normal tone is described as spastic. Soft and flabby muscle tone is defined as atony. A muscle that is limp and without tone is described as being flaccid. The state of readiness known as muscle tone (tonus) is produced by the maintenance of some of the muscle fibers in a contracted state.

A nurse is assessing a child who has a diagnosis of muscular dystrophy. Assessment reveals that the child's muscles have greater-than-normal tone. The nurse should document the presence of which of the following?

Ans: Absence of feeling, capillary refill of 4 to 5 seconds, and cool skin Feedback: Indicators of peripheral neurovascular dysfunction include pale, cyanotic, or mottled skin with a cool temperature; capillary refill greater than 3 seconds; weakness or paralysis with motion; and paresthesia, unrelenting pain, pain on passive stretch, or absence of feeling. Jaundice, diaphoresis, and warmth are inconsistent with peripheral neurovascular dysfunction.

A nurse is assessing a patient who is experiencing peripheral neurovascular dysfunction. What assessment findings are most consistent with this diagnosis?

Ans: Ensuring that there are no metal objects on or in the patient Feedback: Absolutely no metal objects can be present during MRI—their presence constitutes a serious safety risk. The procedure takes up to 90 minutes. Nut allergies and infection are not contraindications to MRI.

A nurse is caring for a patient who has an MRI scheduled. What is the priority safety action prior to this diagnostic procedure?

Ans: "You'll be encouraged to drink water after the administration of the radioisotope injection." Feedback: It is important to encourage the patient to drink plenty of fluids to help distribute and eliminate the isotopic after it is injected. There are important contraindications to the procedure, include pregnancy or an allergy to the radioisotope. The test requires the injection of an intravenous radioisotope and the scan is preformed 2 to 3 hours after the isotope is injected. A calcium solution is not utilized.

A nurse is caring for a patient who has been scheduled for a bone scan. What should the nurse teach the patient about this diagnostic test?

Ans: Fever Feedback: Following arthroscopy, the patient and family are informed of complications to watch for, including fever. Synovial fluid leakage is unlikely and crepitus would not develop as a postprocedure complication. Fasciculations are muscle twitches and do not involve joint integrity or function.

A nurse is caring for a patient who has just had an arthroscopy as an outpatient and is getting ready to go home. The nurse should teach the patient to monitor closely for what postprocedure complication?

Ans: A dull, deep ache that is "boring" in nature Feedback: Bone pain is characteristically described as a dull, deep ache that is "boring" in nature, whereas muscular pain is described as soreness or aching and is referred to as "muscle cramps." Fracture pain is sharp and piercing and is relieved by immobilization. Sharp pain may also result from bone infection with muscle spasm or pressure on a sensory nerve.

A nurse is caring for a patient whose cancer metastasis has resulted in bone pain. Which of the following are typical characteristics of bone pain?

Ans: Alkaline phosphatase Feedback: Alkaline phosphatase is elevated during early fracture healing and in diseases with increased osteoblastic activity (e.g., metastatic bone tumors). Elevated bilirubin, potassium, and creatinine would not be expected in a patient with metastatic bone tumors.

A nurse is caring for a patient with a diagnosis of cancer that has metastasized. What laboratory value would the nurse expect to be elevated in this patient?

Ans: Muscle atrophy Feedback: If a muscle is in disuse for an extended period of time, it is at risk of developing atrophy, which is the decrease in size. Clonus is a pattern of rhythmic muscle contractions and fasciculation is the involuntary twitch of muscle fibers; neither results from immobility. Lack of exercise is a risk factor for rheumatoid arthritis.

A nurse is caring for an older adult who has been diagnosed with geriatric failure to thrive. This patient's prolonged immobility creates a risk for what complication?

Ans: Thyroid hormone, Growth hormone, Estrogen Feedback: The balance between bone resorption and formation is influenced by the following factors: physical activity; dietary intake of certain nutrients, especially calcium; and several hormones, including calcitriol (i.e., activated vitamin D), parathyroid hormone (PTH), calcitonin, thyroid hormone, cortisol, growth hormone, and the sex hormones estrogen and testosterone. Luteinizing hormone and vitamin B12 do not play a role in bone formation or resorption.

A nurse is explaining a patient's decreasing bone density in terms of the balance between bone resorption and formation. What dietary nutrients and hormones play a role in the resorption and formation of adult bones? Select all that apply.

Ans: Crepitus Feedback: Crepitus is a grating, crackling sound or sensation that occurs as the irregular joint surfaces move across one another, as in arthritic conditions. Fasciculations are involuntary twitching of muscle fiber groups. Clonus is the rhythmic contractions of a muscle. Effusion is the collection of excessive fluid within the capsule of a joint.

A nurse is performing a musculoskeletal assessment of a patient with arthritis. During passive range-of-motion exercises, the nurse hears an audible grating sound. The nurse should document the presence of which of the following?

Ans: Activities of daily living Feedback: The nursing assessment is primarily a functional evaluation, focusing on the patient's ability to perform activities of daily living. The nurse also assesses strength, gait, and ROM, but these are assessed to identify their effect on functional status rather than to identify a medical diagnosis.

A nurse is performing a nursing assessment of a patient suspected of having a musculoskeletal disorder. What is the primary focus of the nursing assessment with a patient who has a musculoskeletal disorder?

Ans: An elevated parathyroid hormone level Feedback: In the response to low calcium levels in the blood, increased levels of parathyroid hormone prompt the mobilization of calcium and the demineralization of bone. Increased calcitonin levels would exacerbate hypocalcemia. Vitamin D levels do not increase in response to low calcium levels. Potassium levels would likely be unaffected.

A nurse is providing care for a patient whose pattern of laboratory testing reveals longstanding hypocalcemia. What other laboratory result is most consistent with this finding?

Ans: How does the feeling in the affected extremity compare with the feeling in the unaffected extremity? Feedback: Questions that the nurse should ask regarding altered sensations include "How does this feeling compare to sensation in the unaffected extremity?" Asking questions about strength and color are not relevant and a family history is unlikely.

A nurse is taking a health history on a new patient who has been experiencing unexplained paresthesia. What question should guide the nurse's assessment of the patient's altered sensations?

Ans: Evaluating the effects of the musculoskeletal disorder on the patient's function Feedback: The nursing assessment of the patient with musculoskeletal dysfunction includes an evaluation of the effects of the musculoskeletal disorder on the patient. This is a vital focus of the health history and supersedes the assessment of genetic risk factors and adherence to treatment, though these are both valid inclusions to the interview. Assessment of ROM occurs during the physical assessment, not the interview.

A nurse is taking a health history on a patient with musculoskeletal dysfunction. What is the primary focus of this phase of the nurse's assessment?

Ans: Pricking the skin between the great and second toe Feedback: The nurse will evaluate the sensation of the peroneal nerve by pricking the skin centered between the great and second toe. None of the other listed actions elicits the function of one of the peripheral nerves.

A nurse on the orthopedic unit is assessing a patient's peroneal nerve. The nurse will perform this assessment by doing which of the following actions?

Ans: Scoliosis Feedback: Scoliosis is evidenced by an abnormal lateral curve in the spine, shoulders that are not level, an asymmetric waistline, and a prominent scapula, accentuated by bending forward. Lordosis is the curvature in the lower back; kyphosis is an exaggerated curvature of the upper back. This finding is not suggestive of muscular dystrophy.

A nurse's assessment of a teenage girl reveals that her shoulders are not level and that she has one prominent scapula that is accentuated by bending forward. The nurse should expect to read about what health problem in the patient's electronic health record?

Ans: Injection of a contrast agent into the knee joint prior to ROM exercises Feedback: During arthrography, a radiopaque contrast agent or air is injected into the joint cavity to visualize the joint structures such as the ligaments, cartilage, tendons, and joint capsule. The joint is put through its range of motion to distribute the contrast agent while a series of x-rays are obtained. Synovial fluid is not aspirated or replaced and corticosteroids are not administered.

A patient has been experiencing an unexplained decline in knee function and has consequently been scheduled for arthrography. The nurse should teach the patient about what process?

Ans: Arthrocentesis Feedback: Arthrocentesis (joint aspiration) is carried out to obtain synovial fluid for purposes of examination or to relieve pain due to effusion. Arthrography, biopsy, and electromyography would not remove fluid and relieve pressure.

A patient has been experiencing progressive increases in knee pain and diagnostic imaging reveals a worsening effusion in the synovial capsule. The nurse should anticipate which of the following?

Ans: Contact the primary care provider immediately. Feedback: This major neurovascular problem is caused by pressure within a muscle compartment that increases to such an extent that microcirculation diminishes, leading to nerve and muscle anoxia and necrosis. Function can be permanently lost if the anoxic situation continues for longer than 6 hours. Therefore, immediate medical care is a priority over further nursing assessment. Assessment of calcium levels is unnecessary.

A patient has had a cast placed for the treatment of a humeral fracture. The nurse's most recent assessment shows signs and symptoms of compartment syndrome. What is the nurse's most appropriate action?

Ans: Wrap the joint in a compression dressing. Feedback: Interventions to perform following an arthroscopy include wrapping the joint in a compression dressing, extending and elevating the joint, and applying ice or cold packs. Passive ROM exercises, static flexion, and heat are not indicated.

A patient has just had an arthroscopy performed to assess a knee injury. What nursing intervention should the nurse implement following this procedure?

Ans: Cortical bone Feedback: The long bone shaft, which is referred to as the diaphysis, is constructed primarily of cortical bone.

A patient injured in a motor vehicle accident has sustained a fracture to the diaphysis of the right femur. Of what is the diaphysis of the femur mainly constructed?

Ans: Shuffling gait Feedback: A variety of neurologic conditions are associated with abnormal gaits, such as a spastic hemiparesis gait (stroke), steppage gait (lower motor neuron disease), and shuffling gait (Parkinson's disease). A rapid gait is not associated with Parkinson's disease.

A patient is receiving ongoing nursing care for the treatment of Parkinson's disease. When assessing this patient's gait, what finding is most closely associated with this health problem?

Ans: That the patient emptied the bladder Feedback: Before the scan, the nurse asks the patient to empty the bladder, because a full bladder interferes with accurate scanning of the pelvic bones. Bowel cleansing and fasting are not indicated for a bone scan and an allergy to penicillins is not a contraindication.

A patient is scheduled for a bone scan to rule out osteosarcoma of the pelvic bones. What would be most important for the nurse to assess before the patient's scan?

Ans: Altered serum calcium levels Feedback: Serum calcium levels are altered in patients with osteomalacia, parathyroid dysfunction, Paget's disease, metastatic bone tumors, or prolonged immobilization. Paget's disease is not directly associated with altered magnesium, potassium, or sodium levels.

A patient is undergoing diagnostic testing for suspected Paget's disease. What assessment finding is most consistent with this diagnosis?

Ans: The reparative phase Feedback: Callus formation takes place during the reparative phase of bone healing. The reactive phase occurs immediately after injury and the remodeling phase builds on the reparative phase. There is no discrete revascularization phase.

A patient's fracture is healing and callus is being deposited in the bone matrix. This process characterizes what phase of the bone healing process?

Ans: Arthritis Feedback: The leading cause of musculoskeletal-related disability in the United States is arthritis.

A public health nurse is organizing a campaign that will address the leading cause of musculoskeletal-related disability in the United States. The nurse should focus on what health problem?

7. A nurse on a burn unit is caring for a patient in the acute phase of burn care. While performing an assessment during this phase of burn care, the nurse recognizes that airway obstruction related to upper airway edema may occur up to how long after the burn injury? A) 2 days B) 3 days C) 5 days D) 1 week

A) 2 days Ans: A Feedback: Airway obstruction caused by upper airway edema can take as long as 48 hours to develop. Changes detected by x-ray and arterial blood gases may occur as the effects of resuscitative fluid and the chemical reaction of smoke ingredients with lung tissues become apparent.

22. A nurse has reported for a shift at a busy burns and plastics unit in a large university hospital. Which patient is most likely to have life-threatening complications? A) A 4-year-old scald victim burned over 24% of the body B) A 27-year-old male burned over 36% of his body in a car accident C) A 39-year-old female patient burned over 18% of her body D) A 60-year-old male burned over 16% of his body in a brush fire

A) A 4-year-old scald victim burned over 24% of the body Ans: A Feedback: Young children and the elderly continue to have increased morbidity and mortality when compared to other age groups with similar injuries and present a challenge for burn care. This is an important factor when determining the severity of injury and possible outcome for the patient.

36. A patient who is in the acute phase of recovery from a burn injury has yet to experience adequate pain control. What pain management strategy is most likely to meet this patient's needs? A) A patient-controlled analgesia (PCA) system B) Oral opioids supplemented by NSAIDs C) Distraction and relaxation techniques supplemented by NSAIDs D) A combination of benzodiazepines and topical anesthetics

A) A patient-controlled analgesia (PCA) system Ans: A Feedback: The goal of treatment is to provide a long-acting analgesic that will provide even coverage for this long-term discomfort. It is helpful to use escalating doses when initiating the medication to reach the level of pain control that is acceptable to the patient. The use of patient-controlled analgesia (PCA) gives control to the patient and achieves this goal. Patients cannot normally achieve adequate pain control without the use of opioids, and parenteral administration is usually required.

34. A patient with human immunodeficiency virus (HIV) has sought care because of the recent development of new skin lesions. The nurse should interpret these lesions as most likely suggestive of what? A) A reduction in the patient's CD4 count B) A reduction in the patient's viral load C) An adverse effect of antiretroviral therapy D) Virus-induced changes in allergy status

A) A reduction in the patient's CD4 count Ans: A Feedback: Cutaneous signs may be the first manifestation of human immunodeficiency virus (HIV), appearing in more than 90% of HIV-infected people as immune function deteriorates. These skin signs correlate with low CD4 counts and may become very atypical in immunocompromised people. Viral load increases, not decreases, as the disease progresses. Antiretrovirals are not noted to cause cutaneous changes and viruses do not change an individual's allergy status.

25. An emergency department nurse has just received a patient with burn injuries brought in by ambulance. The paramedics have started a large-bore IV and covered the burn in cool towels. The burn is estimated as covering 24% of the patient's body. How should the nurse best address the pathophysiologic changes resulting from major burns during the initial burn-shock period? A) Administer IV fluids B) Administer broad-spectrum antibiotics C) Administer IV potassium chloride D) Administer packed red blood cells

A) Administer IV fluids Ans: A Feedback: Pathophysiologic changes resulting from major burns during the initial burn-shock period include massive fluid losses. Addressing these losses is a major priority in the initial phase of treatment. Antibiotics and PRBCs are not normally administered. Potassium chloride would exacerbate the patient's hyperkalemia.

31. A nurse educator is teaching a group of medical nurses about Kaposi's sarcoma. What would the educator identify as characteristics of endemic Kaposi's sarcoma? Select all that apply. A) Affects people predominantly in the eastern half of Africa B) Affects men more than women C) Does not affect children D) Cannot infiltrate E) Can progress to lymphadenopathic forms

A) Affects people predominantly in the eastern half of Africa B) Affects men more than women E) Can progress to lymphadenopathic forms Ans: A, B, E Feedback: Endemic (African) Kaposi's sarcoma affects people predominantly in the eastern half of Africa, near the equator. Men are affected more often than women, and children can be affected as well. The disease may resemble classic KS or it may infiltrate and progress to lymphadenopathic forms.

15. The outer layer of the epidermis provides the most effective barrier to penetration of the skin by environmental factors. Which of the following is an example of penetration by an environmental factor? A) An insect bite B) Dehydration C) Sunburn D) Excessive perspiration

A) An insect bite Ans: A Feedback: The stratum corneum, the outer layer of the epidermis, provides the most effective barrier to both epidermal water loss and penetration of environmental factors, such as chemicals, microbes, insect bites, and other trauma. Dehydration, sunburn, and excessive perspiration are not examples of penetration of an environmental factor.

26. An 80-year-old patient is brought to the clinic by her son. The son asks the nurse why his mother has gotten so many spots on her skin. What would be an appropriate response by the nurse? A) As people age, they normally develop uneven pigmentation in their skin. B) These 'spots' are called 'liver spots' or 'age spots.' C) Older skin is more apt to break down and tear, causing sores. D) These are usually the result of nutritional deficits earlier in life.

A) As people age, they normally develop uneven pigmentation in their skin. Ans: A Feedback: The major changes in the skin of older people include dryness, wrinkling, uneven pigmentation, and various proliferative lesions. Stating the names of these spots and identifying older adults' vulnerability to skin damage do not answer the son's question. These lesions are not normally a result of nutritional imbalances.

17. An unresponsive Caucasian patient has been brought to the emergency room by EMS. While assessing this patient, the nurse notes that the patient's face is a cherry-red color. What should the nurse suspect? A) Carbon monoxide poisoning B) Anemia C) Jaundice D) Uremia

A) Carbon monoxide poisoning Ans: A Feedback: Carbon monoxide poisoning causes a bright cherry red color in the face and upper torso in light-skinned persons. In dark-skinned persons, there will be a cherry red color to nail beds, lips, and oral mucosa. When anemia occurs in light-skinned persons, the skin has generalized pallor. Anemia in dark-skinned persons manifests as a yellow-brown coloration. Jaundice appears as a yellow coloration of the sclerae. Uremia gives a yellow-orange tinge to the skin.

21. A patient has just been diagnosed with psoriasis and frequently has lesions around his right eye. What should the nurse teach the patient about topical corticosteroid use on these lesions? A) Cataract development is possible. B) The ointment is likely to cause weeping. C) Corticosteroid use is contraindicated on these lesions. D) The patient may develop glaucoma.

A) Cataract development is possible. Ans: A Feedback: Patients using topical corticosteroid preparations repeatedly on the face and around the eyes should be aware that cataract development is possible. Weeping and glaucoma are less likely. There is no consequent risk of glaucoma.

4. The nurse in an ambulatory care center is admitting an older adult patient who has bright red moles on the skin. Benign changes in elderly skin that appear as bright red moles are termed what? A) Cherry angiomas B) Solar lentigo C) Seborrheickeratoses D) Xanthelasma

A) Cherry angiomas Ans: A Feedback: Cherry angiomas appear as bright red moles, while solar lentigo are commonly called liver spots. Seborrheickeratoses are described as crusty brown stuck on patches, while xanthelasma appears as yellowish, waxy deposits on the upper eyelids.

30. A patient has recently been diagnosed with advanced malignant melanoma and is scheduled for a wide excision of the tumor on her chest. In writing the plan of care for this patient, what major nursing diagnosis should the nurse include? A) Deficient Knowledge about Early Signs of Melanoma B) Chronic Pain Related to Surgical Excision and Grafting C) Depression Related to Reconstructive Surgery D) Anxiety Related to Lack of Social Support

A) Deficient Knowledge about Early Signs of Melanoma Ans: A Feedback: The fact that the patient's disease was not reported until an advanced stage suggests that the patient lacked knowledge about skin lesions. Excision does not result in chronic pain. Reconstructive surgery is not a certainty, and will not necessarily lead to depression. Anxiety is likely, but this may or may not be related to a lack of social support.

29. A nurse is conducting a health interview and is assessing for integumentary conditions that are known to have a genetic component. What assessment question is most appropriate? A) Does anyone in your family have eczema or psoriasis? B) Have any of your family members been diagnosed with malignant melanoma? C) Do you have a family history of vitiligo or port-wine stains? D) Does any member of your family have a history of keloid scarring?

A) Does anyone in your family have eczema or psoriasis? Ans: A Feedback: Eczema and psoriasis are known to have a genetic component. This is not true of any of the other listed integumentary disorders.

8. A patient has sustained a severe burn injury and is thought to have an impaired intestinal mucosal barrier. Since this patient is considered at an increased risk for infection, what intervention will best assist in avoiding increased intestinal permeability and prevent early endotoxin translocation? A) Early enteral feeding B) Administration of prophylactic antibiotics C) Bowel cleansing procedures D) Administration of stool softeners

A) Early enteral feeding Ans: A Feedback: If the intestinal mucosa receives some type of protection against permeability change, infection could be avoided. Early enteral feeding is one step to help avoid this increased intestinal permeability and prevent early endotoxin translocation. Antibiotics are seldom prescribed prophylactically because of the risk of promoting resistant strains of bacteria. A bowel cleansing procedure would not be ordered for this patient. The administration of stool softeners would not assist in avoiding increased intestinal permeability and prevent early endotoxin translocation.

29. A public health nurse has reviewed local data about the incidence and prevalence of burn injuries in the community. These data are likely to support what health promotion effort? A) Education about home safety B) Education about safe storage of chemicals C) Education about workplace health threats D) Education about safe driving

A) Education about home safety Ans: A Feedback: A large majority of burns occur in the home setting; educational interventions should address this epidemiologic trend.

9. A patient has been admitted to a burn intensive care unit with extensive full-thickness burns over 25% of the body. After ensuring cardiopulmonary stability, what would be the nurse's immediate, priority concern when planning this patient's care? A) Fluid status B) Risk of infection C) Nutritional status D) Psychosocial coping

A) Fluid status Ans: A Feedback: During the early phase of burn care, the nurse is most concerned with fluid resuscitation, to correct large-volume fluid loss through the damaged skin. Infection control and early nutritional support are important, but fluid resuscitation is an immediate priority. Coping is a higher priority later in the recovery period.

30. A nurse in the emergency department (ED) is triaging a 5-year-old who has been brought to the ED by her parents for an outbreak of urticaria. What would be the most appropriate question to ask this patient and her family? A) Has she eaten any new foods today? B) Has she bathed in the past 24 hours? C) Did she go to a friend's house today? D) Was she digging in the dirt today?

A) Has she eaten any new foods today? Ans: A Feedback: Foods can cause skin reactions, especially in children. In most cases, this is a more plausible cause of urticaria than bathing, contact with other children, or soil-borne pathogens.

A patient is brought to the emergency department with a burn injury. The nurse knows that the first systemic event after a major burn injury is what? A) Hemodynamic instability B) Gastrointestinal hypermotility C) Respiratory arrest D) Hypokalemia

A) Hemodynamic instability Ans: A Feedback: The initial systemic event after a major burn injury is hemodynamic instability, which results from loss of capillary integrity and a subsequent shift of fluid, sodium, and protein from the intravascular space into the interstitial spaces. This precedes GI changes. Respiratory arrest may or may not occur, largely depending on the presence or absence of smoke inhalation. Hypokalemia does not take place in the initial phase of recovery.

3. A patient in the emergent/resuscitative phase of a burn injury has had blood work and arterial blood gases drawn. Upon analysis of the patient's laboratory studies, the nurse will expect the results to indicate what? A) Hyperkalemia, hyponatremia, elevated hematocrit, and metabolic acidosis B) Hypokalemia, hypernatremia, decreased hematocrit, and metabolic acidosis C) Hyperkalemia, hypernatremia, decreased hematocrit, and metabolic alkalosis D) Hypokalemia, hyponatremia, elevated hematocrit, and metabolic alkalosis

A) Hyperkalemia, hyponatremia, elevated hematocrit, and metabolic acidosis

27. A patient is admitted to the burn unit after being transported from a facility 1000 miles away. The patient has burns to the groin area and circumferential burns to both upper thighs. When assessing the patient's legs distal to the wound site, the nurse should be cognizant of the risk of what complication? A) Ischemia B) Referred pain C) Cellulitis D) Venous thromboembolism (VTE)

A) Ischemia Ans: A Feedback: As edema increases, pressure on small blood vessels and nerves in the distal extremities causes an obstruction of blood flow and consequent ischemia. This complication is similar to compartment syndrome. Referred pain, cellulitis, and VTE are not noted complications that occur distal to the injury site.

6. While waiting to see the physician, a patient shows the nurse skin areas that are flat, nonpalpable, and have had a change of color. The nurse recognizes that the patient is demonstrating what? A) Macules B) Papules C) Vesicles D) Pustules

A) Macules Ans: A Feedback: A macule is a flat, nonpalpable skin color change, while a papule is an elevated, solid, palpable mass. A vesicle is a circumscribed, elevated, palpable mass containing serous fluid, while a pustule is a pus-filled vesicle.

33. A patient is in the acute phase of a burn injury. One of the nursing diagnoses in the plan of care is Ineffective Coping Related to Trauma of Burn Injury. What interventions appropriately address this diagnosis? Select all that apply. A) Promote truthful communication. B) Avoid asking the patient to make decisions. C) Teach the patient coping strategies. D) Administer benzodiazepines as ordered. E) Provide positive reinforcement.

A) Promote truthful communication. C) Teach the patient coping strategies. E) Provide positive reinforcement. Ans: A, C, E Feedback: The nurse can assist the patient to develop effective coping strategies by setting specific expectations for behavior, promoting truthful communication to build trust, helping the patient practice appropriate strategies, and giving positive reinforcement when appropriate. The patient may benefit from being able to make decisions regarding his or her care. Benzodiazepines may be needed for short-term management of anxiety, but they are not used to enhance coping.

24. A patient's blistering disorder has resulted in the formation of multiple lesions in the patient's mouth. What intervention should be included in the patient's plan of care? A) Provide chlorhexidine solution for rinsing the patient's mouth. B) Avoid providing regular mouth care until the patient's lesions heal. C) Liaise with the primary care provider to arrange for parenteral nutrition. D) Encourage the patient to gargle with a hypertonic solution after each meal.

A) Provide chlorhexidine solution for rinsing the patient's mouth. Ans: A Feedback: Frequent rinsing of the mouth with chlorhexidine solution is prescribed to rid the mouth of debris and to soothe ulcerated areas. A hypertonic solution would be likely to cause pain and further skin disruption. Meticulous mouth care should be provided and there is no reason to provide nutrition parenterally.

23. A nurse is caring for a patient admitted to the medical unit with a diagnosis of pemphigus vulgaris. When writing the care plan for this patient, what nursing diagnoses should be included? Select all that apply. A) Risk for Infection Related to Lesions B) Impaired Skin Integrity Related to Epidermal Blisters C) Disturbed Body Image Related to Presence of Skin Lesions D) Acute Pain Related to Disruption in Skin Integrity E) Hyperthermia Related to Disruptions in Thermoregulation

A) Risk for Infection Related to Lesions B) Impaired Skin Integrity Related to Epidermal Blisters C) Disturbed Body Image Related to Presence of Skin Lesions D) Acute Pain Related to Disruption in Skin Integrity Ans: A, B, C, D Feedback: Blistering diseases disrupt skin integrity and are associated with pain and a risk for infection. Because of the visibility of blisters, body image is often affected. The patient faces a risk for hypothermia, not hyperthermia.

18. A nurse is caring for a patient in the emergent/resuscitative phase of burn injury. During this phase, the nurse should monitor for evidence of what alteration in laboratory values? A) Sodium deficit B) Decreased prothrombin time (PT) C) Potassium deficit D) Decreased hematocrit

A) Sodium deficit Ans: A Feedback: Anticipated fluid and electrolyte changes that occur during the emergent/resuscitative phase of burn injury include sodium deficit, potassium excess, base-bicarbonate deficit, and elevated hematocrit. PT does not typically decrease.

27. An older adult patient is diagnosed with a vitamin D deficiency. What would be an appropriate recommendation by the nurse? A) Spend time outdoors at least twice per week B) Increase intake of leafy green vegetables C) Start taking a multivitamin each morning D) Eat red meat at least once per week

A) Spend time outdoors at least twice per week Ans: A Feedback: Skin exposed to ultraviolet light can convert substances necessary for synthesizing vitamin D (cholecalciferol). It is estimated that most people need five to thirty minutes of sun exposure twice a week in order for this synthesis to occur. Multivitamins may not resolve a specific vitamin D deficiency. Vitamin D is unrelated to meat and vegetable intake.

35. A burn patient is transitioning from the acute phase of the injury to the rehabilitation phase. The patient tells the nurse, I can't wait to have surgery to reconstruct my face so I look normal again. What would be the nurse's best response? A) That's something that you and your doctor will likely talk about after your scars mature. B) That is something for you to talk to your doctor about because it's not a nursing responsibility. C) I know this is really important to you, but you have to realize that no one can make you look like you used to. D) Unfortunately, it's likely that you will have most of these scars for the rest of your life.

A) That's something that you and your doctor will likely talk about after your scars mature. Ans: A Feedback: Burn reconstruction is a treatment option after all scars have matured and is discussed within the first few years after injury. Even though this is not a nursing responsibility, the nurse should still respond appropriately to the patient's query. It is true that the patient will not realistically look like he or she used to, but this does not instill hope.

12. An emergency department nurse learns from the paramedics that they are transporting a patient who has suffered injury from a scald from a hot kettle. What variables will the nurse consider when determining the depth of burn? A) The causative agent B) The patient's preinjury health status C) The patient's prognosis for recovery D) The circumstances of the accident

A) The causative agent Ans: A Feedback: The following factors are considered in determining the depth of a burn: how the injury occurred, causative agent (such as flame or scalding liquid), temperature of the burning agent, duration of contact with the agent, and thickness of the skin. The patient's preinjury status, circumstances of the accident, and prognosis for recovery are important, but are not considered when determining the depth of the burn.

15. A patient presents at the free clinic with a black, wart-like lesion on his face, stating, I've done some research, and I'm pretty sure I have malignant melanoma. Subsequent diagnostic testing results in a diagnosis of seborrheic keratosis. The nurse should recognize what significance of this diagnosis? A) The patient requires no treatment unless he finds the lesion to be cosmetically unacceptable. B) The patient's lesion will be closely observed for 6 months before a plan of treatment is chosen. C) The patient has one of the few dermatologic malignancies that respond to chemotherapy. D) The patient will likely require wide excision.

A) The patient requires no treatment unless he finds the lesion to be cosmetically unacceptable. Ans: A Feedback: Seborrheic keratoses are benign, wart like lesions of various sizes and colors, ranging from light tan to black. There is no harm in allowing these growths to remain because there is no medical significance to their presence.

2. A nurse is caring for a patient who has been diagnosed with psoriasis. The nurse is creating an education plan for the patient. What information should be included in this plan? A) Use caution when taking nonprescription medications. B) Avoid public places until symptoms subside. C) Wash skin frequently to prevent infection. D) Liberally apply corticosteroids as needed.

A) Use caution when taking nonprescription medications. Ans: A Feedback: The patient should be cautioned about taking nonprescription medications because some may aggravate mild psoriasis. Psoriasis is not contagious. Many patients need reassurance that the condition is not infectious, not a reflection of poor personal hygiene, and not skin cancer. Excessively frequent washing of skin produces more soreness and scaling. Overuse of topical corticosteroids can result in skin atrophy, striae, and medication resistance.

27. A nurse is providing self-care education to a patient who has been receiving treatment for acne vulgaris. What instruction should the nurse provide to the patient? A) Wash your face with water and gentle soap each morning and evening. B) Before bedtime, clean your face with rubbing alcohol on a cotton pad. C) Gently burst new pimples before they form a visible 'head'. D) Set aside time each day to squeeze blackheads and remove the plug.

A) Wash your face with water and gentle soap each morning and evening. Ans: A Feedback: The nurse should inform the patient to wash the face and other affected areas with mild soap and water twice each day to remove surface oils and prevent obstruction of the oil glands. Cleansing with rubbing alcohol is not recommended and all forms of manipulation should be avoided.

34. A 55-year-old woman is scheduled to have a chemical face peel. The nurse is aware that the patient is likely seeking treatment for which of the following? A) Wrinkles near the lips and eyes B) Removal of acne scars C) Vascular lesions on the cheeks D) Real or perceived misshaping of the eyes

A) Wrinkles near the lips and eyes Ans: A Feedback: Chemical face peeling is especially useful for wrinkles at the upper and lower lip, forehead, and periorbital areas. Chemical face peeling does not remove acne scars, remove vascular lesions, or reshape the eyes.

33. A patient with Cushing syndrome has been hospitalized after a fall. The dietician consulted works with the patient to improve the patients nutritional intake. What foods should a patient with Cushing syndrome eat to optimize health? Select all that apply. A) Foods high in vitamin D B) Foods high in calories C) Foods high in protein D) Foods high in calcium E) Foods high in sodium

A, C, D Feedback: Foods high in vitamin D, protein, and calcium are recommended to minimize muscle wasting and osteoporosis. Referral to a dietitian may assist the patient in selecting appropriate foods that are also low in sodium and calories.

During a patient's recovery from stroke, the nurse should be aware of predictors of stroke outcome in order to help patients and families set realistic goals. What are the predictors of stroke outcome? Select all that apply. A) National Institutes of Health Stroke Scale (NIHSS) score B) Race C) LOC at time of admission D) Gender E) Age

A, C, E (Feedback: It is helpful for clinicians to be knowledgeable about the relative importance of predictors of stroke outcome (age, NIHSS score, and LOC at time of admission) to provide stroke survivors and their families with realistic goals. Race and gender are not predictors of stroke outcome.)

11. The nurse is performing a shift assessment of a patient with aldosteronism. What assessments should the nurse include? Select all that apply. A) Urine output B) Signs or symptoms of venous thromboembolism C) Peripheral pulses D) Blood pressure E) Skin integrity

A, D Feedback: The principal action of aldosterone is to conserve body sodium. Alterations in aldosterone levels consequently affect urine output and BP. The patients peripheral pulses, risk of VTE, and skin integrity are not typically affected by aldosteronism.

A patient with end-stage liver disease has developed hypervolemia. What nursing interventions would be most appropriate when addressing the patient's fluid volume excess? Select all that apply. A. Administering diuretics B. Administering calcium channel blockers C. Implementing fluid restrictions D. Implementing a 1500 kcal/day restriction E. Enhancing patient positioning

A. Administering diuretics B. Administering calcium channel blockers E. Enhancing patient positioning Rationale: Administering diuretics, implementing fluid restrictions, and enhancing patient positioning can optimize the management of fluid volume excess. Calcium channel blockers and calorie restriction do not address this problem.

A patient's physician has ordered a liver panel in response to the patient's development of jaundice. When reviewing the results of this laboratory testing, the nurse should expect to review what blood tests? Select all that apply. A. Alanine aminotransferase (ALT) B. C-reactive protein (CRP) C. Gamma-glutamyl transferase (GGT) D. Aspartate aminotransferase (AST) E. B-type natriuretic peptide (BNP)

A. Alanine aminotransferase (ALT) B. C-reactive protein (CRP) D. Aspartate aminotransferase (AST) Rationale: Liver function testing includes GGT, ALT, and AST. CRP addresses the presence of generalized inflammation and BNP is relevant to heart failure; neither is included in a liver panel.

A nurse is caring for a patient with hepatic encephalopathy. While making the initial shift assessment, the nurse notes that the patient has a flapping tremor of the hands. The nurse should document the presence of what sign of liver disease? A. Asterixis B. Constructional apraxia C. Fector hepaticus D. Palmar erythema

A. Asterixis Rationale: The nurse will document that a patient exhibiting a flapping tremor of the hands is demonstrating asterixis. While constructional apraxia is a motor disturbance, it is the inability to reproduce a simple figure. Fector hepaticus is a sweet, slightly fecal odor to the breath and not associated with a motor disturbance. Skin changes associated with liver dysfunction may include palmar erythema, which is a reddening of the palms, but is not a flapping tremor.

A 55-year-old female patient with hepatocellular carcinoma (HCC) is undergoing radiofrequency ablation. The nurse should recognize what goal of this treatment? A. Destruction of the patient's liver tumor B. Restoration of portal vein patency C. Destruction of a liver abscess D. Reversal of metastasis

A. Destruction of the patient's liver tumor Rationale: Using radiofrequency ablation, a tumor up to 5 cm in size can be destroyed in one treatment session. This technique does not address circulatory function or abscess formation. It does not allow for the reversal of metastasis.

A group of nurses have attended an inservice on the prevention of occupationally acquired diseases that affect healthcare providers. What action has the greatest potential to reduce a nurse's risk of acquiring hepatitis C in the workplace? A. Disposing of sharps appropriately and not recapping needles B. Performing meticulous hand hygiene at the appropriate moments in care C. Adhering to the recommended schedule of immunizations D. Wearing an N95 mask when providing care for patients on airborne precautions

A. Disposing of sharps appropriately and not recapping needles Rationale: HCV is blood-borne. Consequently, prevention of needle stick injuries are paramount. Hand hygiene, immunizations and appropriate use of masks are important aspects of overall infection control, but these actions do not directly mitigate the risk of HCV.

A nurse is caring for a patient with cancer of the liver whose condition has required the insertion of a percutaneous biliary drainage system. The nurse's most recent assessment reveals the presence of dark green fluid in the collection container. What is the nurse's best response to this assessment finding? A. Document the presence of normal bile output. B. Irrigate the drainage system with normal saline as ordered. C. Aspirate a sample of the drainage for culture. D. Promptly report this assessment finding to the primary care provider.

A. Document the presence of normal bile output. Rationale: Bile is usually a dark green or brownish-yellow color, so this would constitute an expected assessment finding, with no other action necessary.

During a health education session, a participant has asked about the hepatitis E virus. What prevention measure should the nurse recommend for preventing infection with this virus? A. Following proper hand-washing techniques B. Avoiding chemicals that are toxic to the liver C. Wearing a condom during sexual contact D. Limiting alcohol intake

A. Following proper hand-washing techniques Rationale: Avoiding contact with the hepatitis E virus through good hygiene, including hand-washing, is the major method of prevention. Hepatitis E is transmitted by the fecaloral route, principally through contaminated water in areas with poor sanitation. Consequently, none of the other listed preventative measure is indicated.

A triage nurse in the emergency department is assessing a patient who presented with complaints of general malaise. Assessment reveals the presence of jaundice and increased abdominal girth. What assessment question best addresses the possible etiology of this patient's presentation? A. How many alcoholic drinks do you typically consume in a week? B. To the best of your knowledge, are your immunizations up to date? C. Have you ever worked in an occupation where you might have been exposed to toxins? D. Has anyone in your family ever experienced symptoms similar to yours?

A. How many alcoholic drinks do you typically consume in a week? Rationale: Signs or symptoms of hepatic dysfunction indicate a need to assess for alcohol use. Immunization status, occupational risks, and family history are also relevant considerations, but alcohol use is a more common etiologic factor in liver disease.

A nurse educate is teaching a group of recent nursing graduates about their occupational risks for contracting hepatitis B. What preventative measures should the educator promote? Select all that apply. A. Immunizations B. Use of standard precautions C. Consumption of vitamin-rich diet D. Annual vitamin K injections E. Annual vitamin B12 injections

A. Immunizations B. Use of standard precautions Rationale: People who are at high risk, including nurses and other health care personnel exposed to blood or blood products, should receive active immunization. The consistent use of standard precautions is also highly beneficial. Vitamin supplementation is unrelated to an individual's risk of HBV.

A nurse on a solid organ transplant unit is planning the care of a patient who will soon be admitted upon immediate recovery following liver transplantation. What aspect of nursing care is the nurse's priority? A. Implementation of infection-control measures B. Close monitoring of skin integrity and color C. Frequent assessment of the patient's psychosocial status D. Administration of antiretroviral medications

A. Implementation of infection-control measures Rationale: Infection control is paramount following liver transplantation. This is a priority over skin integrity and psychosocial status, even though these are valid areas of assessment and intervention. Antiretrovirals are not indicated.

A patient with a liver mass is undergoing a percutaneous liver biopsy. What action should the nurse perform when assisting with this procedure? A. Position the patient on the right side with a pillow under the costal margin after the procedure B. Administer 1 unit of albumin 90 minutes before the procedure as ordered C. Administer at least 1 unit of packed red blood cells as ordered the day before the scheduled procedure D. Confirm that the patient's electrolyte levels have been assessed prior to the procedure

A. Position the patient on the right side with a pillow under the costal margin after the procedure. Rationale: Immediately after a percutaneous liver biopsy, assist the patient to turn onto the right side and place a pillow under the costal margin. Prior administration of albumin or PRBC's is unnecessary. Coagulation tests should be performed, but electrolyte analysis is not necessary.

A patient is being discharged after a liver transplant and the nurse is performing discharge education. When planning the patient's continuing care, the nurse should prioritize which of the following risk diagnoses? A. Risk for infection related to immunosuppressant use B. Risk for injury related to decreased hemostasis C. Risk for unstable blood glucose related to impaired gluconeogensis D. Risk for contamination related to accumulation of ammonia

A. Risk for infection related to immunosuppressant use Rationale: Infection is the leading cause of death after liver transplantation. Pulmonary and fungal infections are common; susceptibility to infection is increased by the immunosuppressive therapy that is needed to prevent rejection. This risk exceeds the threats of injury and unstable blood glucose. The diagnosis of risk for contamination related to environmental toxin exposure.

A patient has developed hepatic encephalopathy secondary to cirrhosis and is receiving care on the medical unit. The patient's current medication regimen includes lactulose (Cephulac) four times daily. What desired outcome should the nurse relate to this pharmacologic intervention? A. Two to 3 soft bowel movements daily B. Significant increase in appetite and food intake C. Absence of nausea and vomiting D. Absence of blood or mucus in stool

A. Two to 3 soft bowel movements daily Rationale: Lactulose (Cephulac) is administered to reduce serum ammonia levels. Two or three soft stools per day are desirable; this indicates that lactulose is performing as intended. Lactulose does not address the patient's appetite, symptoms of nausea and vomiting, or the development of blood and mucus in the stool.

Ans: Kyphosis Feedback: Kyphosis is the increase in thoracic curvature of the spine. Scoliosis is a deviation in the lateral curvature of the spine. Epiphyses are the ends of the long bones. Lordosis is the exaggerated curvature of the lumbar spine.

An older adult patient has come to the clinic for a regular check-up. The nurse's initial inspection reveals an increased thoracic curvature of the patient's spine. The nurse should document the presence of which of the following?

Ans: Bone densitometry Feedback: Bone densitometry is considered the most accurate test for osteoporosis and for predicting a fracture. As such, it is more likely to be used than CT, MRI, or x-rays.

An older adult patient has symptoms of osteoporosis and is being assessed during her annual physical examination. The assessment shows that the patient will require further testing related to a possible exacerbation of her osteoporosis. The nurse should anticipate what diagnostic test?

33. A patient is scheduled for a myelogram and the nurse explains to the patient that this is an invasive procedure, which assesses for any lesions in the spinal cord. The nurse should explain that the preparation is similar to which of the following neurologic tests? A) Lumbar puncture B) MRI C) Cerebral angiography D) EEG

Ans: A Feedback: A myelogram is an x-ray of the spinal subarachnoid space taken after the injection of a contrast agent into the spinal subarachnoid space through a lumbar puncture. Patient preparation for a myelogram would be similar to that for lumbar puncture. The other listed diagnostic tests do not involve lumbar puncture.

14. A patient with a C5 spinal cord injury is tetraplegic. After being moved out of the ICU, the patient complains of a severe throbbing headache. What should the nurse do first? A) Check the patient's indwelling urinary catheter for kinks to ensure patency. B) Lower the HOB to improve perfusion. C) Administer analgesia. D) Reassure the patient that headaches are expected after spinal cord injuries.

Ans: A Feedback: A severe throbbing headache is a common symptom of autonomic dysreflexia, which occurs after injuries to the spinal cord above T6. The syndrome is usually brought on by sympathetic stimulation, such as bowel and bladder distention. Lowering the HOB can increase ICP. Before administering analgesia, the nurse should check the patient's catheter, record vital signs, and perform an abdominal assessment. A severe throbbing headache is a dangerous symptom in this patient and is not expected.

17. A patient with lower back pain is scheduled for myelography using metrizamide (a water-soluble contrast dye). After the test, the nurse should prioritize what action? A) Positioning the patient with the head of the bed elevated 45 degrees B) Administering IV morphine sulfate to prevent headache C) Limiting fluids for the next 12 hours D) Helping the patient perform deep breathing and coughing exercises

Ans: A Feedback: After myelography, the patient lies in bed with the head of the bed elevated 30 to 45 degrees. The patient is advised to remain in bed in the recommended position for 3 hours or as prescribed. Drinking liberal amounts of fluid for rehydration and replacement of CSF may decrease the incidence of post-lumbar puncture headache. Deep breathing and coughing exercises are not normally necessary since there is no consequent risk of atelectasis.

29. The school nurse has been called to the football field where player is immobile on the field after landing awkwardly on his head during a play. While awaiting an ambulance, what action should the nurse perform? A) Ensure that the player is not moved. B) Obtain the player's vital signs, if possible. C) Perform a rapid assessment of the player's range of motion. D) Assess the player's reflexes.

Ans: A Feedback: At the scene of the injury, the patient must be immobilized on a spinal (back) board, with the head and neck maintained in a neutral position, to prevent an incomplete injury from becoming complete. This is a priority over determining the patient's vital signs. It would be inappropriate to test ROM or reflexes.

30. The nurse is caring for a patient whose spinal cord injury has caused recent muscle spasticity. What medication should the nurse expect to be ordered to control this? A) Baclofen (Lioresal) B) Dexamethasone (Decadron) C) Mannitol (Osmitrol) D) Phenobarbital (Luminal)

Ans: A Feedback: Baclofen is classified as an antispasmodic agent in the treatment of muscles spasms related to spinal cord injury. Decadron is an anti-inflammatory medication used to decrease inflammation in both SCI and head injury. Mannitol is used to decrease cerebral edema in patients with head injury. Phenobarbital is an anticonvulsant that is used in the treatment of seizure activity.

37. A patient who is being treated in the hospital for a spinal cord injury is advocating for the removal of his urinary catheter, stating that he wants to try to resume normal elimination. What principle should guide the care team's decision regarding this intervention? A) Urinary retention can have serious consequences in patients with SCIs. B) Urinary function is permanently lost following an SCI. C) Urinary catheters should not remain in place for more than 7 days. D) Overuse of urinary catheters can exacerbate nerve damage.

Ans: A Feedback: Bladder distention, a major cause of autonomic dysreflexia, can also cause trauma. For this reason, removal of a urinary catheter must be considered with caution. Extended use of urinary catheterization is often necessary following SCI. The effect of a spinal cord lesion on urinary function depends on the level of the injury. Catheter use does not cause nerve damage, although it is a major risk factor for UTIs.

22. A patient is admitted to the neurologic ICU with a suspected diffuse axonal injury. What would be the primary neuroimaging diagnostic tool used on this patient to evaluate the brain structure? A) MRI B) PET scan C) X-ray D) Ultrasound

Ans: A Feedback: CT and MRI scans, the primary neuroimaging diagnostic tools, are useful in evaluating the brain structure. Ultrasound would not show the brain nor would an x-ray. A PET scan shows brain function, not brain structure.

17. A patient with a T2 injury is in spinal shock. The nurse will expect to observe what assessment finding? A) Absence of reflexes along with flaccid extremities B) Positive Babinski's reflex along with spastic extremities C) Hyperreflexia along with spastic extremities D) Spasticity of all four extremities

Ans: A Feedback: During the period immediately following a spinal cord injury, spinal shock occurs. In spinal shock, all reflexes are absent and the extremities are flaccid. When spinal shock subsides, the patient demonstrates a positive Babinski's reflex, hyperreflexia, and spasticity of all four extremities.

31. The nurse is planning the care of a patient with a T1 spinal cord injury. The nurse has identified the diagnosis of "risk for impaired skin integrity." How can the nurse best address this risk? A) Change the patient's position frequently. B) Provide a high-protein diet. C) Provide light massage at least daily. D) Teach the patient deep breathing and coughing exercises.

Ans: A Feedback: Frequent position changes are among the best preventative measures against pressure ulcers. A high-protein diet can benefit wound healing, but does not necessarily prevent skin breakdown. Light massage and deep breathing do not protect or restore skin integrity.

16. A patient is being given a medication that stimulates her parasympathetic system. Following administration of this medication, the nurse should anticipate what effect? A) Constricted pupils B) Dilated bronchioles C) Decreased peristaltic movement D) Relaxed muscular walls of the urinary bladder

Ans: A Feedback: Parasympathetic stimulation results in constricted pupils, constricted bronchioles, increased peristaltic movement, and contracted muscular walls of the urinary bladder.

29. A gerontologic nurse educator is providing practice guidelines to unlicensed care providers. Because reaction to painful stimuli is sometimes blunted in older adults, what must be used with caution? A) Hot or cold packs B) Analgesics C) Anti-inflammatory medications D) Whirlpool baths

Ans: A Feedback: Reaction to painful stimuli may be decreased with age. Because pain is an important warning signal, caution must be used when hot or cold packs are used. The older patient may be burned or suffer frostbite before being aware of any discomfort. Any medication is used with caution in the elderly, but not because of the decreased sense of heat or cold. Whirlpool baths are generally not a routine treatment ordered for the elderly.

4. The nurse is caring for a patient with increased intracranial pressure (ICP) caused by a traumatic brain injury. Which of the following clinical manifestations would suggest that the patient may be experiencing increased brain compression causing brain stem damage? A) Hyperthermia B) Tachycardia C) Hypertension D) Bradypnea

Ans: A Feedback: Signs of increasing ICP include slowing of the heart rate (bradycardia), increasing systolic BP, and widening pulse pressure. As brain compression increases, respirations become rapid, BP may decrease, and the pulse slows further. A rapid rise in body temperature is regarded as unfavorable. Hyperthermia increases the metabolic demands of the brain and may indicate brain stem damage.

14. A patient exhibiting an uncoordinated gait has presented at the clinic. Which of the following is the most plausible cause of this patient's health problem? A) Cerebellar dysfunction B) A lesion in the pons C) Dysfunction of the medulla D) A hemorrhage in the midbrain

Ans: A Feedback: The cerebellum controls fine movement, balance, position sense, and integration of sensory input. Portions of the pons control the heart, respiration, and blood pressure. Cranial nerves IX through XII connect to the brain in the medulla. Cranial nerves III and IV originate in the midbrain.

22. A patient is admitted to the medical unit with an exacerbation of multiple sclerosis. When assessing this patient, the nurse has the patient stick out her tongue and move it back and forth. What is the nurse assessing? A) Function of the hypoglossal nerve B) Function of the vagus nerve C) Function of the spinal nerve D) Function of the trochlear nerve

Ans: A Feedback: The hypoglossal nerve is the 12th cranial nerve. It is responsible for movement of the tongue. None of the other listed nerves affects motor function in the tongue.

11. The patient in the ED has just had a diagnostic lumbar puncture. To reduce the incidence of a post-lumbar puncture headache, what is the nurse's most appropriate action? A) Position the patient prone. B) Position the patient supine with the head of bed flat. C) Position the patient left side-lying. D) Administer acetaminophen as ordered.

Ans: A Feedback: The lumbar puncture headache may be avoided if a small-gauge needle is used and if the patient remains prone after the procedure. Acetaminophen is not administered as a preventative measure for post-lumbar puncture headaches.

34. The nurse is providing health education to a patient who has a C6 spinal cord injury. The patient asks why autonomic dysreflexia is considered an emergency. What would be the nurse's best answer? A) "The sudden increase in BP can raise the ICP or rupture a cerebral blood vessel." B) "The suddenness of the onset of the syndrome tells us the body is struggling to maintain its normal state." C) "Autonomic dysreflexia causes permanent damage to delicate nerve fibers that are healing." D) "The sudden, severe headache increases muscle tone and can cause further nerve damage."

Ans: A Feedback: The sudden increase in BP may cause a rupture of one or more cerebral blood vessels or lead to increased ICP. Autonomic dysreflexia does not directly cause nerve damage.

24. An 82-year-old man is admitted for observation after a fall. Due to his age, the nurse knows that the patient is at increased risk for what complication of his injury? A) Hematoma B) Skull fracture C) Embolus D) Stroke

Ans: A Feedback: Two major factors place older adults at increased risk for hematomas. First, the dura becomes more adherent to the skull with increasing age. Second, many older adults take aspirin and anticoagulants as part of routine management of chronic conditions. Because of these factors, the patient's risk of a hematoma is likely greater than that of stroke, embolism, or skull fracture.

38. A patient with spinal cord injury is ready to be discharged home. A family member asks the nurse to review potential complications one more time. What are the potential complications that should be monitored for in this patient? Select all that apply. A) Orthostatic hypotension B) Autonomic dysreflexia C) DVT D) Salt-wasting syndrome E) Increased ICP

Ans: A, B, C Feedback: For a spinal cord-injured patient, based on the assessment data, potential complications that may develop include DVT, orthostatic hypotension, and autonomic dysreflexia. Salt-wasting syndrome or increased ICP are not typical complications following the immediate recovery period.

28. The school nurse is giving a presentation on preventing spinal cord injuries (SCI). What should the nurse identify as prominent risk factors for SCI? Select all that apply. A) Young age B) Frequent travel C) African American race D) Male gender E) Alcohol or drug use

Ans: A, D, E Feedback: The predominant risk factors for SCI include young age, male gender, and alcohol and drug use. Ethnicity and travel are not risk factors.

26. The nurse is caring for a patient who exhibits abnormal results of the Weber test and Rinne test. The nurse should suspect dysfunction involving what cranial nerve? A) Trigeminal B) Acoustic C) Hypoglossal D) Trochlear

Ans: B Feedback: Abnormal hearing can correlate with damage to cranial nerve VIII (acoustic). The acoustic nerve functions in hearing and equilibrium. The trigeminal nerve functions in facial sensation, corneal reflex, and chewing. The hypoglossal nerve moves the tongue. The trochlear nerve controls muscles that move the eye.

16. An elderly woman found with a head injury on the floor of her home is subsequently admitted to the neurologic ICU. What is the best rationale for the following physician orders: elevate the HOB; keep the head in neutral alignment with no neck flexion or head rotation; avoid sharp hip flexion? A) To decrease cerebral arterial pressure B) To avoid impeding venous outflow C) To prevent flexion contractures D) To prevent aspiration of stomach contents

Ans: B Feedback: Any activity or position that impedes venous outflow from the head may contribute to increased volume inside the skull and possibly increase ICP. Cerebral arterial pressure will be affected by the balance between oxygen and carbon dioxide. Flexion contractures are not a priority at this time. Stomach contents could still be aspirated in this position.

3. A nurse is caring for a critically ill patient with autonomic dysreflexia. What clinical manifestations would the nurse expect in this patient? A) Respiratory distress and projectile vomiting B) Bradycardia and hypertension C) Tachycardia and agitation D) Third-spacing and hyperthermia

Ans: B Feedback: Autonomic dysreflexia is characterized by a pounding headache, profuse sweating, nasal congestion, piloerection ("goose bumps"), bradycardia, and hypertension. It occurs in cord lesions above T6 after spinal shock has resolved; it does not result in vomiting, tachycardia, or third-spacing.

6. The nurse is doing an initial assessment on a patient newly admitted to the unit with a diagnosis of cerebrovascular accident (CVA). The patient has difficulty copying a figure that the nurse has drawn and is diagnosed with visual-receptive aphasia. What brain region is primarily involved in this deficit? A) Temporal lobe B) Parietal-occipital area C) Inferior posterior frontal areas D) Posterior frontal area

Ans: B Feedback: Difficulty copying a figure that the nurse has drawn would be considered visual-receptive aphasia, which involves the parietal-occipital area. Expressive aphasia, the inability to express oneself, is often associated with damage to the frontal area. Receptive aphasia, the inability to understand what someone else is saying, is often associated with damage to the temporal lobe area.

2. A patient is brought to the trauma center by ambulance after sustaining a high cervical spinal cord injury 1½ hours ago. Endotracheal intubation has been deemed necessary and the nurse is preparing to assist. What nursing diagnosis should the nurse associate with this procedure? A) Risk for impaired skin integrity B) Risk for injury C) Risk for autonomic dysreflexia D) Risk for suffocation

Ans: B Feedback: If endotracheal intubation is necessary, extreme care is taken to avoid flexing or extending the patient's neck, which can result in extension of a cervical injury. Intubation does not directly cause autonomic dysreflexia and the threat to skin integrity is a not a primary concern. Intubation does not carry the potential to cause suffocation.

20. Following a spinal cord injury a patient is placed in halo traction. While performing pin site care, the nurse notes that one of the traction pins has become detached. The nurse would be correct in implementing what priority nursing action? A) Complete the pin site care to decrease risk of infection. B) Notify the neurosurgeon of the occurrence. C) Stabilize the head in a lateral position. D) Reattach the pin to prevent further head trauma.

Ans: B Feedback: If one of the pins became detached, the head is stabilized in neutral position by one person while another notifies the neurosurgeon. Reattaching the pin as a nursing intervention would not be done due to risk of increased injury. Pin site care would not be a priority in this instance. Prevention of neurologic injury is the priority.

37. The nurse is performing a neurologic assessment of a patient whose injuries have rendered her unable to follow verbal commands. How should the nurse proceed with assessing the patient's level of consciousness (LOC)? A) Assess the patient's vital signs and correlate these with the patient's baselines. B) Assess the patient's eye opening and response to stimuli. C) Document that the patient currently lacks a level of consciousness. D) Facilitate diagnostic testing in an effort to obtain objective data.

Ans: B Feedback: If the patient is not alert or able to follow commands, the examiner observes for eye opening; verbal response and motor response to stimuli, if any; and the type of stimuli needed to obtain a response. Vital signs and diagnostic testing are appropriate, but neither will allow the nurse to gauge the patient's LOC. Inability to follow commands does not necessarily denote an absolute lack of consciousness.

8. A patient with spinal cord injury has a nursing diagnosis of altered mobility and the nurse recognizes the increased the risk of deep vein thrombosis (DVT). Which of the following would be included as an appropriate nursing intervention to prevent a DVT from occurring? A) Placing the patient on a fluid restriction as ordered B) Applying thigh-high elastic stockings C) Administering an antifibrinolyic agent D) Assisting the patient with passive range of motion (PROM) exercises

Ans: B Feedback: It is important to promote venous return to the heart and prevent venous stasis in a patient with altered mobility. Applying elastic stockings will aid in the prevention of a DVT. The patient should not be placed on fluid restriction because a dehydrated state will increase the risk of clotting throughout the body. Antifibrinolytic agents cause the blood to clot, which is absolutely contraindicated in this situation. PROM exercises are not an effective protection against the development of DVT.

6. The staff educator is precepting a nurse new to the critical care unit when a patient with a T2 spinal cord injury is admitted. The patient is soon exhibiting manifestations of neurogenic shock. In addition to monitoring the patient closely, what would be the nurse's most appropriate action? A) Prepare to transfuse packed red blood cells. B) Prepare for interventions to increase the patient's BP. C) Place the patient in the Trendelenberg position. D) Prepare an ice bath to lower core body temperature.

Ans: B Feedback: Manifestations of neurogenic shock include decreased BP and heart rate. Cardiac markers would be expected to rise in cardiogenic shock. Transfusion, repositioning, and ice baths are not indicated interventions.

38. In the course of a focused neurologic assessment, the nurse is palpating the patient's major muscle groups at rest and during passive movement. Data gleaned from this assessment will allow the nurse to describe which of the following aspects of neurologic function? A) Muscle dexterity B) Muscle tone C) Motor symmetry D) Deep tendon reflexes

Ans: B Feedback: Muscle tone (the tension present in a muscle at rest) is evaluated by palpating various muscle groups at rest and during passive movement. Data from this assessment do not allow the nurse to ascertain the patient's dexterity, reflexes, or motor symmetry.

25. A neurologic flow chart is often used to document the care of a patient with a traumatic brain injury. At what point in the patient's care should the nurse begin to use a neurologic flow chart? A) When the patient's condition begins to deteriorate B) As soon as the initial assessment is made C) At the beginning of each shift D) When there is a clinically significant change in the patient's condition

Ans: B Feedback: Neurologic parameters are assessed initially and as frequently as the patient's condition requires. As soon as the initial assessment is made, the use of a neurologic flowchart is started and maintained. A new chart is not begun at the start of every shift.

10. A patient who has sustained a nondepressed skull fracture is admitted to the acute medical unit. Nursing care should include which of the following? A) Preparation for emergency craniotomy B) Watchful waiting and close monitoring C) Administration of inotropic drugs D) Fluid resuscitation

Ans: B Feedback: Nondepressed skull fractures generally do not require surgical treatment; however, close observation of the patient is essential. A craniotomy would not likely be needed if the fracture is nondepressed. Even if treatment is warranted, it is unlikely to include inotropes or fluid resuscitation.

21. The nurse is planning the care of a patient with Parkinson's disease. The nurse should be aware that treatment will focus on what pathophysiological phenomenon? A) Premature degradation of acetylcholine B) Decreased availability of dopamine C) Insufficient synthesis of epinephrine D) Delayed reuptake of serotonin

Ans: B Feedback: Parkinson's disease develops from decreased availability of dopamine, not acetylcholine, epinephrine, or serotonin.

2. A patient scheduled for magnetic resonance imaging (MRI) has arrived at the radiology department. The nurse who prepares the patient for the MRI should prioritize which of the following actions? A) Withholding stimulants 24 to 48 hours prior to exam B) Removing all metal-containing objects C) Instructing the patient to void prior to the MRI D) Initiating an IV line for administration of contrast

Ans: B Feedback: Patient preparation for an MRI consists of removing all metal-containing objects prior to the examination. Withholding stimulants would not affect an MRI; this relates to an electroencephalography (EEG). Instructing the patient to void is patient preparation for a lumbar puncture. Initiating an IV line for administration of contrast would be done if the patient was having a CT scan with contrast.

3. A gerontologic nurse planning the neurologic assessment of an older adult is considering normal, age-related changes. Of what phenomenon should the nurse be aware? A) Hyperactive deep tendon reflexes B) Reduction in cerebral blood flow C) Increased cerebral metabolism D) Hypersensitivity to painful stimuli

Ans: B Feedback: Reduction in cerebral blood flow (CBF) is a change that occurs in the normal aging process. Deep tendon reflexes can be decreased or, in some cases, absent. Cerebral metabolism decreases as the patient advances in age. Reaction to painful stimuli may be decreased with age. Because pain is an important warning signal, caution must be used when hot or cold packs are used.

34. The physician has ordered a somatosensory evoked responses (SERs) test for a patient for whom the nurse is caring. The nurse is justified in suspecting that this patient may have a history of what type of neurologic disorder? A) Hypothalamic disorder B) Demyelinating disease C) Brainstem deficit D) Diabetic neuropathy

Ans: B Feedback: SERs are used to detect deficits in the spinal cord or peripheral nerve conduction and to monitor spinal cord function during surgical procedures. The test is also useful in the diagnosis of demyelinating diseases, such as multiple sclerosis and polyneuropathies, where nerve conduction is slowed. The test is not done to diagnose hypothalamic disorders, brainstem deficits, or diabetic neuropathies.

30. A trauma patient in the ICU has been declared brain dead. What diagnostic test is used in making the determination of brain death? A) Magnetic resonance imaging (MRI) B) Electroencephalography (EEG) C) Electromyelography (EMG) D) Computed tomography (CT)

Ans: B Feedback: The EEG can be used in determining brain death. MRI, CT, and EMG are not normally used in determining brain death.

18. A nurse is reviewing the trend of a patient's scores on the Glasgow Coma Scale (GCS). This allows the nurse to gauge what aspect of the patient's status? A) Reflex activity B) Level of consciousness C) Cognitive ability D) Sensory involvement

Ans: B Feedback: The Glasgow Coma Scale (GCS) examines three responses related to LOC: eye opening, best verbal response, and best motor response.

1. A patient is brought to the ER following a motor vehicle accident in which he sustained head trauma. Preliminary assessment reveals a vision deficit in the patient's left eye. The nurse should associate this abnormal finding with trauma to which of the following cerebral lobes? A) Temporal B) Occipital C) Parietal D) Frontal

Ans: B Feedback: The posterior lobe of the cerebral hemisphere is responsible for visual interpretation. The temporal lobe contains the auditory receptive areas. The parietal lobe contains the primary sensory cortex, and is essential to an individual's awareness of the body in space, as well as orientation in space and spatial relations. The frontal lobe functions in concentration, abstract thought, information storage or memory, and motor function.

4. The nurse has admitted a new patient to the unit. One of the patient's admitting orders is for an adrenergic medication. The nurse knows that this medication will have what effect on the circulatory system? A) Thin, watery saliva B) Increased heart rate C) Decreased BP D) Constricted bronchioles

Ans: B Feedback: The term "adrenergic" refers to the sympathetic nervous system. Sympathetic effects include an increased rate and force of the heartbeat. Cholinergic effects, which correspond to the parasympathetic division of the autonomic nervous system, include thin, watery saliva, decreased rate and force of heartbeat, and decreased BP.

24. Assessment is crucial to the care of patients with neurologic dysfunction. What does accurate and appropriate assessment require? Select all that apply. A) The ability to select mediations for the neurologic dysfunction B) Understanding of the tests used to diagnose neurologic disorders C) Knowledge of nursing interventions related to assessment and diagnostic testing D) Knowledge of the anatomy of the nervous system E) The ability to interpret the results of diagnostic tests

Ans: B, C, D Feedback: Assessment requires knowledge of the anatomy and physiology of the nervous system and an understanding of the array of tests and procedures used to diagnose neurologic disorders. Knowledge about the nursing implications and interventions related to assessment and diagnostic testing is also essential. Selecting medications and interpreting diagnostic tests are beyond the normal scope of the nurse.

19. The nurse educator is reviewing the assessment of cranial nerves. What should the educator identify as the specific instances when cranial nerves should be assessed? Select all that apply. A) When a neurogenic bladder develops B) When level of consciousness is decreased C) With brain stem pathology D) In the presence of peripheral nervous system disease E) When a spinal reflex is interrupted

Ans: B, C, D Feedback: Cranial nerves are assessed when level of consciousness is decreased, with brain stem pathology, or in the presence of peripheral nervous system disease. Abnormalities in muscle tone and involuntary movements are less likely to prompt the assessment of cranial nerves, since these nerves do not directly mediate most aspects of muscle tone and movement.

19. The nurse is caring for a patient who is rapidly progressing toward brain death. The nurse should be aware of what cardinal signs of brain death? Select all that apply. A) Absence of pain response B) Apnea C) Coma D) Absence of brain stem reflexes E) Absence of deep tendon reflexes

Ans: B, C, D Feedback: The three cardinal signs of brain death upon clinical examination are coma, the absence of brain stem reflexes, and apnea. Absences of pain response and deep tendon reflexes are not necessarily indicative of brain death.

40. A patient is admitted to the neurologic ICU with a C4 spinal cord injury. When writing the plan of care for this patient, which of the following nursing diagnoses would the nurse prioritize in the immediate care of this patient? A) Risk for impaired skin integrity related to immobility and sensory loss B) Impaired physical mobility related to loss of motor function C) Ineffective breathing patterns related to weakness of the intercostal muscles D) Urinary retention related to inability to void spontaneously

Ans: C Feedback: A nursing diagnosis related to breathing pattern would be the priority for this patient. A C4 spinal cord injury will require ventilatory support, due to the diaphragm and intercostals being affected. The other nursing diagnoses would be used in the care plan, but not designated as a higher priority than ineffective breathing patterns.

5. A patient is brought to the ED by her family after falling off the roof. A family member tells the nurse that when the patient fell she was "knocked out," but came to and "seemed okay." Now she is complaining of a severe headache and not feeling well. The care team suspects an epidural hematoma, prompting the nurse to prepare for which priority intervention? A) Insertion of an intracranial monitoring device B) Treatment with antihypertensives C) Emergency craniotomy D) Administration of anticoagulant therapy

Ans: C Feedback: An epidural hematoma is considered an extreme emergency. Marked neurologic deficit or respiratory arrest can occur within minutes. Treatment consists of making an opening through the skull to decrease ICP emergently, remove the clot, and control the bleeding. Antihypertensive medications would not be a priority. Anticoagulant therapy should not be ordered for a patient who has a cranial bleed. This could further increase bleeding activity. Insertion of an intracranial monitoring device may be done during the surgery, but is not priority for this patient.

35. A patient had a lumbar puncture performed at the outpatient clinic and the nurse has phoned the patient and family that evening. What does this phone call enable the nurse to determine? A) What are the patient's and family's expectations of the test B) Whether the patient's family had any questions about why the test was necessary C) Whether the patient has had any complications of the test D) Whether the patient understood accurately why the test was done

Ans: C Feedback: Contacting the patient and family after diagnostic testing enables the nurse to determine whether they have any questions about the procedure or whether the patient had any untoward results. The other listed information should have been elicited from the patient and family prior to the test.

25. When caring for a patient with an altered level of consciousness, the nurse is preparing to test cranial nerve VII. What assessment technique would the nurse use to elicit a response from cranial nerve VII? A) Palpate trapezius muscle while patient shrugs should against resistance. B) Administer the whisper or watch-tick test. C) Observe for facial movement symmetry, such as a smile. D) Note any hoarseness in the patient's voice.

Ans: C Feedback: Cranial nerve VII is the facial nerve. An appropriate assessment technique for this cranial nerve would include observing for symmetry while the patient performs facial movements: smiles, whistles, elevates eyebrows, and frowns. Palpating and noting strength of the trapezius muscle while the patient shrugs shoulders against resistance would be completed to assess cranial nerve XI (spinal accessory). Assessing cranial nerve VIII (acoustic) would involve using the whisper or watch-tick test to evaluate hearing. Noting any hoarseness in the patient's voice would involve assessment of cranial nerve X (vagus).

9. Paramedics have brought an intubated patient to the RD following a head injury due to acceleration-deceleration motor vehicle accident. Increased ICP is suspected. Appropriate nursing interventions would include which of the following? A) Keep the head of the bed (HOB) flat at all times. B) Teach the patient to perform the Valsalva maneuver. C) Administer benzodiazepines on a PRN basis. D) Perform endotracheal suctioning every hour.

Ans: C Feedback: If the patient with a brain injury is very agitated, benzodiazepines are the most commonly used sedatives and do not affect cerebral blood flow or ICP. The HOB should be elevated 30 degrees. Suctioning should be done a limited basis, due to increasing the pressure in the cranium. The Valsalva maneuver is to be avoided. This also causes increased ICP.

15. A patient is admitted to the neurologic ICU with a spinal cord injury. When assessing the patient the nurse notes there is a sudden depression of reflex activity in the spinal cord below the level of injury. What should the nurse suspect? A) Epidural hemorrhage B) Hypertensive emergency C) Spinal shock D) Hypovolemia

Ans: C Feedback: In spinal shock, the reflexes are absent, BP and heart rate fall, and respiratory failure can occur. Hypovolemia, hemorrhage, and hypertension do not cause this sudden change in neurologic function.

39. The nurse recognizes that a patient with a SCI is at risk for muscle spasticity. How can the nurse best prevent this complication of an SCI? A) Position the patient in a high Fowler's position when in bed. B) Support the knees with a pillow when the patient is in bed. C) Perform passive ROM exercises as ordered. D) Administer NSAIDs as ordered.

Ans: C Feedback: Passive ROM exercises can prevent muscle spasticity following SCI. NSAIDs are not used for this purpose. Pillows and sitting upright do not directly address the patient's risk of muscle spasticity.

12. A patient is admitted to the neurologic ICU with a spinal cord injury. In writing the patient's care plan, the nurse specifies that contractures can best be prevented by what action? A) Repositioning the patient every 2 hours B) Initiating range-of-motion exercises (ROM) as soon as the patient initiates C) Initiating (ROM) exercises as soon as possible after the injury D) Performing ROM exercises once a day

Ans: C Feedback: Passive ROM exercises should be implemented as soon as possible after injury. It would be inappropriate to wait for the patient to first initiate exercises. Toes, metatarsals, ankles, knees, and hips should be put through a full ROM at least four, and ideally five, times daily. Repositioning alone will not prevent contractures.

31. A patient is scheduled for CT scanning of the head because of a recent onset of neurologic deficits. What should the nurse tell the patient in preparation for this test? A) "No metal objects can enter the procedure room." B) "You need to fast for 8 hours prior to the test." C) "You will need to lie still throughout the procedure." D) "There will be a lot of noise during the test."

Ans: C Feedback: Preparation for CT scanning includes teaching the patient about the need to lie quietly throughout the procedure. If the patient were having an MRI, metal and noise would be appropriate teaching topics. There is no need to fast prior to a CT scan of the brain.

40. During the performance of the Romberg test, the nurse observes that the patient sways slightly. What is the nurse's most appropriate action? A) Facilitate a referral to a neurologist. B) Reposition the patient supine to ensure safety. C) Document successful completion of the assessment. D) Follow up by having the patient perform the Rinne test.

Ans: C Feedback: Slight swaying during the Romberg test is normal, but a loss of balance is abnormal and is considered a positive Romberg test. Slight swaying is not a significant threat to the patient's safety. The Rinne test assesses hearing, not balance.

18. A patient is having a "fight or flight response" after receiving bad news about his prognosis. What affect will this have on the patient's sympathetic nervous system? A) Constriction of blood vessels in the heart muscle B) Constriction of bronchioles C) Increase in the secretion of sweat D) Constriction of pupils

Ans: C Feedback: Sympathetic nervous system stimulation results in dilated blood vessels in the heart and skeletal muscle, dilated bronchioles, increased secretion of sweat, and dilated pupils.

23. A trauma patient was admitted to the ICU with a brain injury. The patient had a change in level of consciousness, increased vital signs, and became diaphoretic and agitated. The nurse should recognize which of the following syndromes as the most plausible cause of these symptoms? A) Adrenal crisis B) Hypothalamic collapse C) Sympathetic storm D) Cranial nerve deficit

Ans: C Feedback: Sympathetic storm is a syndrome associated with changes in level of consciousness, altered vital signs, diaphoresis, and agitation that may result from hypothalamic stimulation of the sympathetic nervous system following traumatic brain injury. Alterations in cranial nerve or adrenal function would not have this result.

27. The nurse caring for an 80 year-old patient knows that she has a pre-existing history of dulled tactile sensation. The nurse should first consider what possible cause for this patient's diminished tactile sensation? A) Damage to cranial nerve VIII B) Adverse medication effects C) Age-related neurologic changes D) An undiagnosed cerebrovascular accident in early adulthood

Ans: C Feedback: Tactile sensation is dulled in the elderly person due to a decrease in the number of sensory receptors. While thorough assessment is necessary, it is possible that this change is unrelated to pathophysiological processes.

36. Splints have been ordered for a patient who is at risk of developing footdrop following a spinal cord injury. The nurse caring for this patient knows that the splints are removed and reapplied when? A) At the patient's request B) Each morning and evening C) Every 2 hours D) One hour prior to mobility exercises

Ans: C Feedback: The feet are prone to footdrop; therefore, various types of splints are used to prevent footdrop. When used, the splints are removed and reapplied every 2 hours.

20. A patient in the OR goes into malignant hyperthermia due to an abnormal reaction to the anesthetic. The nurse knows that the area of the brain that regulates body temperature is which of the following? A) Cerebellum B) Thalamus C) Hypothalamus D) Midbrain

Ans: C Feedback: The hypothalamus plays an important role in the endocrine system because it regulates the pituitary secretion of hormones that influence metabolism, reproduction, stress response, and urine production. It works with the pituitary to maintain fluid balance through hormonal release and maintains temperature regulation by promoting vasoconstriction or vasodilatation. The cerebellum, thalamus, and midbrain and not directly involved in temperature regulation.

26. The nurse planning the care of a patient with head injuries is addressing the patient's nursing diagnosis of "sleep deprivation." What action should the nurse implement? A) Administer a benzodiazepine at bedtime each night. B) Do not disturb the patient between 2200 and 0600. C) Cluster overnight nursing activities to minimize disturbances. D) Ensure that the patient does not sleep during the day.

Ans: C Feedback: To allow the patient longer times of uninterrupted sleep and rest, the nurse can group nursing care activities so that the patient is disturbed less frequently. However, it is impractical and unsafe to provide no care for an 8-hour period. The use of benzodiazepines should be avoided.

35. The nurse caring for a patient with a spinal cord injury notes that the patient is exhibiting early signs and symptoms of disuse syndrome. Which of the following is the most appropriate nursing action? A) Limit the amount of assistance provided with ADLs. B) Collaborate with the physical therapist and immobilize the patient's extremities temporarily. C) Increase the frequency of ROM exercises. D) Educate the patient about the importance of frequent position changes.

Ans: C Feedback: To prevent disuse syndrome, ROM exercises must be provided at least four times a day, and care is taken to stretch the Achilles tendon with exercises. The patient is repositioned frequently and is maintained in proper body alignment whether in bed or in a wheelchair. The patient must be repositioned by caregivers, not just taught about repositioning. It is inappropriate to limit assistance for the sole purpose of preventing disuse syndrome.

32. A patient with a spinal cord injury has experienced several hypotensive episodes. How can the nurse best address the patient's risk for orthostatic hypotension? A) Administer an IV bolus of normal saline prior to repositioning. B) Maintain bed rest until normal BP regulation returns. C) Monitor the patient's BP before and during position changes. D) Allow the patient to initiate repositioning.

Ans: C Feedback: To prevent hypotensive episodes, close monitoring of vital signs before and during position changes is essential. Prolonged bed rest carries numerous risks and it is not possible to provide a bolus before each position change. Following the patient's lead may or may not help regulate BP.

8. The nurse is caring for a patient with an upper motor neuron lesion. What clinical manifestations should the nurse anticipate when planning the patient's neurologic assessment? A) Decreased muscle tone B) Flaccid paralysis C) Loss of voluntary control of movement D) Slow reflexes

Ans: C Feedback: Upper motor neuron lesions do not cause muscle atrophy, flaccid paralysis, or slow reflexes. However, upper motor neuron lesions normally cause loss of voluntary control.

5. A nurse is assessing reflexes in a patient with hyperactive reflexes. When the patient's foot is abruptly dorsiflexed, it continues to "beat" two to three times before settling into a resting position. How would the nurse document this finding? A) Rigidity B) Flaccidity C) Clonus D) Ataxia

Ans: C Feedback: When reflexes are very hyperactive, a phenomenon called clonus may be elicited. If the foot is abruptly dorsiflexed, it may continue to "beat" two to three times before it settles into a position of rest. Rigidity is an increase in muscle tone at rest characterized by increased resistance to passive stretch. Flaccidity is lack of muscle tone. Ataxia is the inability to coordinate muscle movements, resulting in difficulty walking, talking, and performing self-care activities.

11. A patient who suffered a spinal cord injury is experiencing an exaggerated autonomic response. What aspect of the patient's current health status is most likely to have precipitated this event? A) The patient received a blood transfusion. B) The patient's analgesia regimen was recent changed. C) The patient was not repositioned during the night shift. D) The patient's urinary catheter became occluded.

Ans: D Feedback: A distended bladder is the most common cause of autonomic dysreflexia. Infrequent positioning is a less likely cause, although pressure ulcers or tactile stimulation can cause it. Changes in mediations or blood transfusions are unlikely causes.

21. The ED is notified that a 6-year-old is in transit with a suspected brain injury after being struck by a car. The child is unresponsive at this time, but vital signs are within acceptable limits. What will be the primary goal of initial therapy? A) Promoting adequate circulation B) Treating the child's increased ICP C) Assessing secondary brain injury D) Preserving brain homeostasis

Ans: D Feedback: All therapy is directed toward preserving brain homeostasis and preventing secondary brain injury, which is injury to the brain that occurs after the original traumatic event. The scenario does not indicate the child has increased ICP or a secondary brain injury at this point. Promoting circulation is likely secondary to the broader goal of preserving brain homeostasis.

28. A 72-year-old man has been brought to his primary care provider by his daughter, who claims that he has been experiencing uncharacteristic lapses in memory. What principle should underlie the nurse's assessment and management of this patient? A) Loss of short-term memory is normal in older adults, but loss of long-term memory is pathologic. B) Lapses in memory in older adults are considered benign unless they have negative consequences. C) Gradual increases in confusion accompany the aging process. D) Thorough assessment is necessary because changes in cognition are always considered to be pathologic.

Ans: D Feedback: Although mental processing time decreases with age, memory, language, and judgment capacities remain intact. Change in mental status should never be assumed to be a normal part of aging.

39. The neurologic nurse is testing the function of a patient's cerebellum and basal ganglia. What action will most accurately test these structures? A) Have the patient identify the location of a cotton swab on his or her skin with the eyes closed. B) Elicit the patient's response to a hypothetical problem. C) Ask the patient to close his or her eyes and discern between hot and cold stimuli. D) Guide the patient through the performance of rapid, alternating movements.

Ans: D Feedback: Cerebellar and basal ganglia influence on the motor system is reflected in balance control and coordination. Coordination in the hands and upper extremities is tested by having the patient perform rapid, alternating movements and point-to-point testing. The cerebellum and basal ganglia do not mediate cutaneous sensation or judgment.

13. A nurse is caring for a patient diagnosed with Ménière's disease. While completing a neurologic examination on the patient, the nurse assesses cranial nerve VIII. The nurse would be correct in identifying the function of this nerve as what? A) Movement of the tongue B) Visual acuity C) Sense of smell D) Hearing and equilibrium

Ans: D Feedback: Cranial nerve VIII (acoustic) is responsible for hearing and equilibrium. Cranial nerve XII (hypoglossal) is responsible for movement of the tongue. Cranial nerve II (optic) is responsible for visual acuity and visual fields. Cranial nerve I (olfactory) functions in sense of smell.

15. The nursing students are learning how to assess function of cranial nerve VIII. To assess the function of cranial nerve VIII the students would be correct in completing which of the following assessment techniques? A) Have the patient identify familiar odors with the eyes closed. B) Assess papillary reflex. C) Utilize the Snellen chart. D) Test for air and bone conduction (Rinne test).

Ans: D Feedback: Cranial nerve VIII is the acoustic nerve. It functions in hearing and equilibrium. When assessing this nerve, the nurse would test for air and bone conduction (Rinne) with a tuning fork. Assessment of papillary reflex would be completed for cranial nerves III (oculomotor), IV (trochlear), and VI (abducens). The Snellen chart would be used to assess cranial nerve II (optic).

27. The nurse has implemented interventions aimed at facilitating family coping in the care of a patient with a traumatic brain injury. How can the nurse best facilitate family coping? A) Help the family understand that the patient could have died. B) Emphasize the importance of accepting the patient's new limitations. C) Have the members of the family plan the patient's inpatient care. D) Assist the family in setting appropriate short-term goals.

Ans: D Feedback: Helpful interventions to facilitate coping include providing family members with accurate and honest information and encouraging them to continue to set well-defined, short-term goals. Stating that a patient's condition could be worse downplays their concerns. Emphasizing the importance of acceptance may not necessarily help the family accept the patient's condition. Family members cannot normally plan a patient's hospital care, although they may contribute to the care in some ways.

23. A 13-year-old was brought to the ED, unconscious, after being hit in the head by a baseball. When the child regains consciousness, 5 hours after being admitted, he cannot remember the traumatic event. MRI shows no structural sign of injury. What injury would the nurse suspect the patient has? A) Diffuse axonal injury B) Grade 1 concussion with frontal lobe involvement C) Contusion D) Grade 3 concussion with temporal lobe involvement

Ans: D Feedback: In a grade 3 concussion there is a loss of consciousness lasting from seconds to minutes. Temporal lobe involvement results in amnesia. Frontal lobe involvement can cause uncharacteristic behavior and a grade 1 concussion does not involve loss of consciousness. Diagnostic studies may show no apparent structural sign of injury, but the duration of unconsciousness is an indicator of the severity of the concussion. Diffuse axonal injury (DAI) results from widespread shearing and rotational forces that produce damage throughout the brain—to axons in the cerebral hemispheres, corpus callosum, and brain stem. In cerebral contusion, a moderate to severe head injury, the brain is bruised and damaged in a specific area because of severe acceleration-deceleration force or blunt trauma.

32. A patient for whom the nurse is caring has positron emission tomography (PET) scheduled. In preparation, what should the nurse explain to the patient? A) The test will temporarily limit blood flow through the brain. B) An allergy to iodine precludes getting the radio-opaque dye. C) The patient will need to endure loud noises during the test. D) The test may result in dizziness or lightheadedness.

Ans: D Feedback: Key nursing interventions for PET scan include explaining the test and teaching the patient about inhalation techniques and the sensations (e.g., dizziness, light-headedness, and headache) that may occur. A PET scan does not impede blood flow through the brain. An allergy to iodine precludes the dye for an MRI, and loud noise is heard in an MRI.

9. The nurse is admitting a patient to the unit who is diagnosed with a lower motor neuron lesion. What entry in the patient's electronic record is most consistent with this diagnosis? A) "Patient exhibits increased muscle tone." B) "Patient demonstrates normal muscle structure with no evidence of atrophy." C) "Patient demonstrates hyperactive deep tendon reflexes." D) "Patient demonstrates an absence of deep tendon reflexes."

Ans: D Feedback: Lower motor neuron lesions cause flaccid muscle paralysis, muscle atrophy, decreased muscle tone, and loss of voluntary control.

33. A nurse on the neurologic unit is providing care for a patient who has spinal cord injury at the level of C4. When planning the patient's care, what aspect of the patient's neurologic and functional status should the nurse consider? A) The patient will be unable to use a wheelchair. B) The patient will be unable to swallow food. C) The patient will be continent of urine, but incontinent of bowel. D) The patient will require full assistance for all aspects of elimination.

Ans: D Feedback: Patients with a lesion at C4 are fully dependent for elimination. The patient is dependent for feeding, but is able to swallow. The patient will be capable of using an electric wheelchair.

12. The nurse is conducting a focused neurologic assessment. When assessing the patient's cranial nerve function, the nurse would include which of the following assessments? A) Assessment of hand grip B) Assessment of orientation to person, time, and place C) Assessment of arm drift D) Assessment of gag reflex

Ans: D Feedback: The gag reflex is governed by the glossopharyngeal nerve, one of the cranial nerves. Hand grip and arm drifting are part of motor function assessment. Orientation is an assessment parameter related to a mental status examination.

7. An ED nurse has just received a call from EMS that they are transporting a 17-year-old man who has just sustained a spinal cord injury (SCI). The nurse recognizes that the most common cause of this type of injury is what? A) Sports-related injuries B) Acts of violence C) Injuries due to a fall D) Motor vehicle accidents

Ans: D Feedback: The most common causes of SCIs are motor vehicle crashes (46%), falls (22%), violence (16%), and sports (12%).

36. A patient is currently being stimulated by the parasympathetic nervous system. What effect will this nervous stimulation have on the patient's bladder? A) The parasympathetic nervous system causes urinary retention. B) The parasympathetic nervous system causes bladder spasms. C) The parasympathetic nervous system causes urge incontinence. D) The parasympathetic nervous system makes the bladder contract.

Ans: D Feedback: The parasympathetic division of the nervous system causes contraction (stimulation) of the urinary bladder muscles and a decrease (inhibition) in heart rate, whereas the sympathetic division produces relaxation (inhibition) of the urinary bladder and an increase (stimulation) in the rate and force of the heartbeat.

10. An elderly patient is being discharged home. The patient lives alone and has atrophy of his olfactory organs. The nurse tells the patient's family that it is essential that the patient have what installed in the home? A) Grab bars B) Nonslip mats C) Baseboard heaters D) A smoke detector

Ans: D Feedback: The sense of smell deteriorates with age. The olfactory organs are responsible for smell. This may present a safety hazard for the patient because he or she may not smell smoke or gas leaks. Smoke detectors are universally necessary, but especially for this patient.

7. What term is used to describe the fibrous connective tissue that hugs the brain closely and extends into every fold of the brain's surface? A) Dura mater B) Arachnoid C) Fascia D) Pia mater

Ans: D Feedback: The term "meninges" describes the fibrous connective tissue that covers the brain and spinal cord. The meninges have three layers, the dura mater, arachnoid, and pia mater. The pia mater is the innermost membrane that hugs the brain closely and extends into every fold of the brain's surface. The dura mater, the outermost layer, covers the brain and spinal cord. The arachnoid, the middle membrane, is responsible for the production of cerebrospinal fluid.

13. A patient with a head injury has been increasingly agitated and the nurse has consequently identified a risk for injury. What is the nurse's best intervention for preventing injury? A) Restrain the patient as ordered. B) Administer opioids PRN as ordered. C) Arrange for friends and family members to sit with the patient. D) Pad the side rails of the patient's bed.

Ans: D Feedback: To protect the patient from self-injury, the nurse uses padded side rails. The nurse should avoid restraints, because straining against them can increase ICP or cause other injury. Narcotics used to control restless patients should be avoided because these medications can depress respiration, constrict the pupils, and alter the patient's responsiveness. Visitors should be limited if the patient is agitated.

The nurse is planning the care of a patient who has been recently diagnosed with a cerebellar tumor. Due to the location of this patients tumor, the nurse should implement measures to prevent what complication? A. Falls B. Audio hallucinations C. Respiratory depression D. Labile BP

Ans: A Feedback: A cerebellar tumor causes dizziness, an ataxic or staggering gait with a tendency to fall toward the side of the lesion, and marked muscle incoordination. Because of this, the patient faces a high risk of falls. Hallucinations and unstable vital signs are not closely associated with cerebellar tumors.

A nurse is assessing a patient with an acoustic neuroma who has been recently admitted to an oncology unit. What symptoms is the nurse likely to find during the initial assessment? A. Loss of hearing, tinnitus, and vertigo B. Loss of vision, change in mental status, and hyperthermia C. Loss of hearing, increased sodium retention, and hypertension D. Loss of vision, headache, and tachycardia

Ans: A Feedback: An acoustic neuroma is a tumor of the eighth cranial nerve, the cranial nerve most responsible for hearing and balance. The patient with an acoustic neuroma usually experiences loss of hearing, tinnitus, and episodes of vertigo and staggering gait. Acoustic neuromas do not cause loss of vision, increased sodium retention, or tachycardia.

A patient with Huntington disease has just been admitted to a long-term care facility. The charge nurse is creating a care plan for this patient. Nutritional management for a patient with Huntington disease should be informed by what principle? A. The patient is likely to have an increased appetite. B. The patient is likely to required enzyme supplements. C. The patient will likely require a clear liquid diet. D. The patient will benefit from a low-protein diet.

Ans: A Feedback: Due to the continuous involuntary movements, patients will have a ravenous appetite. Despite this ravenous appetite, patients usually become emaciated and exhausted. As the disease progresses, patients experience difficulty in swallowing and thin liquids should be avoided. Protein will not be limited with this disease. Enzyme supplements are not normally required.

A gerontologic nurse is advocating for diagnostic testing of an 81-year-old patient who is experiencing personality changes. The nurse is aware of what factor that is known to affect the diagnosis and treatment of brain tumors in older adults? A. The effects of brain tumors are often attributed to the cognitive effects of aging. B. Brain tumors in older adults do not normally produce focal effects. C. Older adults typically have numerous benign brain tumors by the eighth decade of life. D. Brain tumors cannot normally be treated in patient over age 75.

Ans: A Feedback: In older adult patients, early signs and symptoms of intracranial tumors can be easily overlooked or incorrectly attributed to cognitive and neurologic changes associated with normal aging. Brain tumors are not normally benign and they produce focal effects in all patients. Treatment options are not dependent primarily on age.

A male patient presents at the free clinic with complaints of impotency. Upon physical examination, the nurse practitioner notes the presence of hypogonadism. What diagnosis should the nurse suspect? A. Prolactinoma B. Angioma C. Glioma D. Adrenocorticotropic hormone (ACTH)producing adenoma

Ans: A Feedback: Male patients with prolactinomas may present with impotence and hypogonadism. An ACTH-producing adenoma would cause acromegaly. The scenario contains insufficient information to know if the tumor is an angioma, glioma, or neuroma.

An older adult has encouraged her husband to visit their primary care provider, stating that she is concerned that he may have Parkinsons disease. Which of the wifes descriptions of her husbands health and function is most suggestive of Parkinsons disease? A. Lately he seems to move far more slowly than he ever has in the past. B. He often complains that his joints are terribly stiff when he wakes up in the morning. C. He's forgotten the names of some people that we've known for years. D. He's losing weight even though he has a ravenous appetite.

Ans: A Feedback: Parkinsons disease is characterized by bradykinesia. It does not manifest as memory loss, increased appetite, or joint stiffness.

A patient with a new diagnosis of amyotrophic lateral sclerosis (ALS) is overwhelmed by his diagnosis and the known complications of the disease. How can the patient best make known his wishes for care as his disease progresses? A. Prepare an advance directive. B. Designate a most responsible physician (MRP) early in the course of the disease. C. Collaborate with representatives from the Amyotrophic Lateral Sclerosis Association. D. Ensure that witnesses are present when he provides instruction.

Ans: A Feedback: Patients with ALS are encouraged to complete an advance directive or living will to preserve their autonomy in decision making. None of the other listed actions constitutes a legally binding statement of end-of-life care.

A patient has been admitted to the neurologic ICU with a diagnosis of a brain tumor. The patient is scheduled to have a tumor resection/removal in the morning. Which of the following assessment parameters should the nurse include in the initial assessment? A. Gag reflex B. Deep tendon reflexes C. Abdominal girth D. Hearing acuity

Ans: A Feedback: Preoperatively, the gag reflex and ability to swallow are evaluated. In patients with diminished gag response, care includes teaching the patient to direct food and fluids toward the unaffected side, having the patient sit upright to eat, offering a semisoft diet, and having suction readily available. Deep tendon reflexes, abdominal girth, and hearing acuity are less commonly affected by brain tumors and do not affect the risk for aspiration.

A nurse is planning discharge education for a patient who underwent a cervical diskectomy. What strategies would the nurse assess that would aid in planning discharge teaching? A. Care of the cervical collar B. Technique for performing neck ROM exercises C. Home assessment of ABGs D. Techniques for restoring nerve function

Ans: A Feedback: Prior to discharge, the nurse should assess the patients use and care of the cervical collar. Neck ROM exercises would be contraindicated and ABGs cannot be assessed in the home. Nerve function is not compromised by a diskectomy.

The nurse caring for a patient diagnosed with Parkinsons disease has prepared a plan of care that would include what goal? A. Promoting effective communication B. Controlling diarrhea C. Preventing cognitive decline D. Managing choreiform movements

Ans: A Feedback: The goals for the patient may include improving functional mobility, maintaining independence in ADLs, achieving adequate bowel elimination, attaining and maintaining acceptable nutritional status, achieving effective communication, and developing positive coping mechanisms. Constipation is more likely than diarrhea and cognition largely remains intact. Choreiform movements are related to Huntington disease.

A patient with suspected Parkinsons disease is initially being assessed by the nurse. When is the best time to assess for the presence of a tremor? A. When the patient is resting B. When the patient is ambulating C. When the patient is preparing his or her meal tray to eat D. When the patient is participating in occupational therapy

Ans: A Feedback: The tremor is present while the patient is at rest; it increases when the patient is walking, concentrating, or feeling anxious. Resting tremor characteristically disappears with purposeful movement, but is evident when the extremities are motionless. Consequently, the nurse should assess for the presence of a tremor when the patient is not performing deliberate actions.

37. A patient with MS has been admitted to the hospital following an acute exacerbation. When planning the patient's care, the nurse addresses the need to enhance the patient's bladder control. What aspect of nursing care is most likely to meet this goal? A) Establish a timed voiding schedule. B) Avoid foods that change the pH of urine. C) Perform intermittent catheterization q6h. D) Administer anticholinergic drugs as ordered.

Ans: A A timed voiding schedule addresses many of the challenges with urinary continence that face the patient with MS. Interventions should be implemented to prevent the need for catheterization and anticholinergics are not normally used.

18. The nurse is caring for a patient who is hospitalized with an exacerbation of MS. To ensure the patient's safety, what nursing action should be performed? A) Ensure that suction apparatus is set up at the bedside. B) Pad the patient's bed rails. C) Maintain bed rest whenever possible. D) Provide several small meals each day.

Ans: A Because of the patient's risk of aspiration, it is important to have a suction apparatus at hand. Bed rest should be generally be minimized, not maximized, and there is no need to pad the patient's bed rails or to provide multiple small meals.

5. A patient diagnosed with Bell's palsy is being cared for on an outpatient basis. During health education, the nurse should promote which of the following actions? A) Applying a protective eye shield at night B) Chewing on the affected side to prevent unilateral neglect C) Avoiding the use of analgesics whenever possible D) Avoiding brushing the teeth

Ans: A Corneal irritation and ulceration may occur if the eye is unprotected. While paralysis lasts, the involved eye must be protected. The patient should be encouraged to eat on the unaffected side, due to swallowing difficulties. Analgesics are used to control the facial pain. The patient should continue to provide self-care including oral hygiene.

23. The nurse is caring for a patient whose recent health history includes an altered LOC. What should be the nurses first action when assessing this patient? A) Assessing the patients verbal response B) Assessing the patients ability to follow complex commands C) Assessing the patients judgment D) Assessing the patients response to pain

Ans: A Feedback: Assessment of the patient with an altered LOC often starts with assessing the verbal response through determining the patients orientation to time, person, and place. In most cases, this assessment will precede each of the other listed assessments, even though each may be indicated.

17. A patient has developed diabetes insipidus after having increased ICP following head trauma. What nursing assessment best addresses this complication? A) Vigilant monitoring of fluid balance B) Continuous BP monitoring C) Serial arterial blood gases (ABGs) D) Monitoring of the patients airway for patency

Ans: A Feedback: Diabetes insipidus requires fluid and electrolyte replacement, along with the administration of vasopressin, to replace and slow the urine output. Because of these alterations in fluid balance, careful monitoring is necessary. None of the other listed assessments directly addresses the major manifestations of diabetes insipidus.

14. The nurse has created a plan of care for a patient who is at risk for increased ICP. The patients care plan should specify monitoring for what early sign of increased ICP? A) Disorientation and restlessness B) Decreased pulse and respirations C) Projectile vomiting D) Loss of corneal reflex

Ans: A Feedback: Early indicators of ICP include disorientation and restlessness. Later signs include decreased pulse and respirations, projectile vomiting, and loss of brain stem reflexes, such as the corneal reflex.

26. The nurse is caring for a patient who sustained a moderate head injury following a bicycle accident. The nurses most recent assessment reveals that the patients respiratory effort has increased. What is the nurses most appropriate response? A) Inform the care team and assess for further signs of possible increased ICP. B) Administer bronchodilators as ordered and monitor the patients LOC. C) Increase the patients bed height and reassess in 30 minutes. D) Administer a bolus of normal saline as ordered.

Ans: A Feedback: Increased respiratory effort can be suggestive of increasing ICP, and the care team should be promptly informed. A bolus of IV fluid will not address the problem. Repositioning the patient and administering bronchodilators are insufficient responses, even though these actions may later be ordered.

33. A patient is postoperative day 1 following intracranial surgery. The nurses assessment reveals that the patients LOC is slightly decreased compared with the day of surgery. What is the nurses best response to this assessment finding? A) Recognize that this may represent the peak of post-surgical cerebral edema. B) Alert the surgeon to the possibility of an intracranial hemorrhage. C) Understand that the surgery may have been unsuccessful. D) Recognize the need to refer the patient to the palliative care team.

Ans: A Feedback: Some degree of cerebral edema occurs after brain surgery, it tends to peak 24 to 36 hours after surgery, producing decreased responsiveness on the second postoperative day. As such, there is not necessarily any need to deem the surgery unsuccessful or to refer the patient to palliative care. A decrease in LOC is not evidence of an intracranial hemorrhage.

29. When caring for a patient with increased ICP the nurse knows the importance of monitoring for possible secondary complications, including syndrome of inappropriate antidiuretic hormone (SIADH). What nursing interventions would the nurse most likely initiate if the patient developed SIADH? A) Fluid restriction B) Transfusion of platelets C) Transfusion of fresh frozen plasma (FFP) D) Electrolyte restriction

Ans: A Feedback: The nurse also assesses for complications of increased ICP, including diabetes insipidus, and SIADH. SIADH requires fluid restriction and monitoring of serum electrolyte levels. Transfusions are unnecessary.

21. The nurse is caring for a patient with permanent neurologic impairments resulting from a traumatic head injury. When working with this patient and family, what mutual goal should be prioritized? A) Achieve as high a level of function as possible. B) Enhance the quantity of the patients life. C) Teach the family proper care of the patient. D) Provide community assistance.

Ans: A Feedback: The overarching goals of care are to achieve as high a level of function as possible and to enhance the quality of life for the patient with neurologic impairment and his or her family. This goal encompasses family and community participation.

31. A patient is recovering from intracranial surgery performed approximately 24 hours ago and is complaining of a headache that the patient rates at 8 on a 10-point pain scale. What nursing action is most appropriate? A) Administer morphine sulfate as ordered. B) Reposition the patient in a prone position. C) Apply a hot pack to the patients scalp. D) Implement distraction techniques.

Ans: A Feedback: The patient usually has a headache after a craniotomy as a result of stretching and irritation of nerves in the scalp during surgery. Morphine sulfate may also be used in the management of postoperative pain in patients who have undergone a craniotomy. Prone positioning is contraindicated due to the consequent increase in ICP. Distraction would likely be inadequate to reduce pain and a hot pack may cause vasodilation and increased pain.

37. A hospital patient has experienced a seizure. In the immediate recovery period, what action best protects the patients safety? A) Place the patient in a side-lying position. B) Pad the patients bed rails. C) Administer antianxiety medications as ordered. D) Reassure the patient and family members.

Ans: A Feedback: To prevent complications, the patient is placed in the side-lying position to facilitate drainage of oral secretions. Suctioning is performed, if needed, to maintain a patent airway and prevent aspiration. None of the other listed actions promotes safety during the immediate recovery period.

6. The nurse is working with a patient who is newly diagnosed with MS. What basic information should the nurse provide to the patient? A) MS is a progressive demyelinating disease of the nervous system. B) MS usually occurs more frequently in men. C) MS typically has an acute onset. D) MS is sometimes caused by a bacterial infection.

Ans: A MS is a chronic, degenerative, progressive disease of the central nervous system, characterized by the occurrence of small patches of demyelination in the brain and spinal cord. The cause of MS is not known, and the disease affects twice as many women as men.

29. The nurse is teaching a patient with Guillain-Barré syndrome about the disease. The patient asks how he can ever recover if demyelination of his nerves is occurring. What would be the nurse's best response? A) Guillain-Barré spares the Schwann cell, which allows for remyelination in the recovery phase of the disease. B) In Guillain-Barré, Schwann cells replicate themselves before the disease destroys them, so remyelination is possible. C) I know you understand that nerve cells do not remyelinate, so the physician is the best one to answer your question. D) For some reason, in Guillain-Barré, Schwann cells become activated and take over the remyelination process.

Ans: A Myelin is a complex substance that covers nerves, providing insulation and speeding the conduction of impulses from the cell body to the dendrites. The cell that produces myelin in the peripheral nervous system is the Schwann cell. In Guillain-Barré syndrome, the Schwann cell is spared, allowing for remyelination in the recovery phase of the disease. The nurse should avoid downplaying the patient's concerns by wholly deferring to the physician.

33. A patient with diabetes presents to the clinic and is diagnosed with a mononeuropathy. This patient's nursing care should involve which of the following? A) Protection of the affected limb from injury B) Passive and active ROM exercises for the affected limb C) Education about improvements to glycemic control D) Interventions to prevent contractures

Ans: A Nursing care involves protection of the affected limb or area from injury, as well as appropriate patient teaching about mononeuropathy and its treatment. Nursing care for this patient does not likely involve exercises or assistive devices, since these are unrelated to the etiology of the disease. Improvements to diabetes management may or may not be necessary.

20. The nurse is developing a plan of care for a patient with Guillain-Barré syndrome. Which of the following interventions should the nurse prioritize for this patient? A) Using the incentive spirometer as prescribed B) Maintaining the patient on bed rest C) Providing aids to compensate for loss of vision D) Assessing frequently for loss of cognitive function

Ans: A Respiratory function can be maximized with incentive spirometry and chest physiotherapy. Nursing interventions toward enhancing physical mobility should be utilized. Nursing interventions are aimed at preventing a deep vein thrombosis. Guillain-Barré syndrome does not affect cognitive function or vision.

26. You are the clinic nurse caring for a patient with a recent diagnosis of myasthenia gravis. The patient has begun treatment with pyridostigmine bromide (Mestinon). What change in status would most clearly suggest a therapeutic benefit of this medication? A) Increased muscle strength B) Decreased pain C) Improved GI function D) Improved cognition

Ans: A The goal of treatment using pyridostigmine bromide is improvement of muscle strength and control of fatigue. The drug is not intended to treat pain, or cognitive or GI functions.

2. The nurse is planning discharge education for a patient with trigeminal neuralgia. The nurse knows to include information about factors that precipitate an attack. What would the nurse be correct in teaching the patient to avoid? A) Washing his face B) Exposing his skin to sunlight C) Using artificial tears D) Drinking large amounts of fluids

Ans: A Washing the face should be avoided if possible because this activity can trigger an attack of pain in a patient with trigeminal neuralgia. Using artificial tears would be an appropriate behavior. Exposing the skin to sunlight would not be harmful to this patient. Temperature extremes in beverages should be avoided.

5. When writing a plan of care for a patient with psoriasis, the nurse would know that an appropriate nursing diagnosis for this patient would be what? A) Impaired Skin Integrity Related to Scaly Lesions B) Acute Pain Related to Blistering and Erosions of the Oral Cavity C) Impaired Tissue Integrity Related to Epidermal Shedding D) Anxiety Related to Risk for Melanoma

Ans: A Feedback: An appropriate diagnosis for a patient with psoriasis would include Impaired Skin Integrity as it relates to scaly lesions. Psoriasis causes pain but does not normally affect the oral cavity. Similarly, tissue integrity is impaired, but not through the process of epidermal shedding. Psoriasis is not related to an increased risk for melanoma.

A patient's electronic health record notes that the patient has hallux valgus. What signs and symptoms would the nurse expect this patient to manifest? A) Deviation of a great toe laterally B) Abnormal flexion of the great toe C) An exaggerated arch of the foot D) Fusion of the toe joints

Ans: A Feedback: A deformity in which the great toe deviates laterally and there is a marked prominence of the medial aspect of the first metatarsal-phalangeal joint and exostosis is referred to as hallux valgus (bunion). Hallux valgus does not result in abnormal flexion, abnormalities of the arch, or joint fusion.

A patient who has undergone a lower limb amputation is preparing to be discharged home. What outcome is necessary prior to discharge? A) Patient can demonstrate safe use of assistive devices. B) Patient has a healed, nontender, nonadherent scar. C) Patient can perform activities of daily living independently. D) Patientis free of pain.

Ans: A Feedback: A patient should be able to use assistive devices appropriately and safely prior to discharge. Scar formation will not be complete at the time of hospital discharge. It is anticipated that the patient will require some assistance with ADLs postdischarge. Pain should be well managed, but may or may not be wholly absent.

A patient was fitted with an arm cast after fracturing her humerus. Twelve hours after the application of the cast, the patient tells the nurse that her arm hurts. Analgesics do not relieve the pain. What would be the most appropriate nursing action? A) Prepare the patient for opening or bivalving of the cast. B) Obtain an order for a different analgesic. C) Encourage the patient to wiggle and move the fingers. D) Petal the edges of the patient's cast.

Ans: A Feedback: Acute compartment syndrome involves a sudden and severe decrease in blood flow to the tissues distal to an area of injury that results in ischemic necrosis if prompt, decisive intervention does not occur. Removing or bivalving the cast is necessary to relieve pressure. Ordering different analgesics does not address the underlying problem. Encouraging the patient to move the fingers or perform range-of-motion exercises will not treat or prevent compartment syndrome. Petaling the edges of a cast with tape prevents abrasions and skin breakdown, not compartment syndrome.

A rehabilitation nurse is working with a patient who has had a below-the-knee amputation. The nurse knows the importance of the patient's active participation in self-care. In order to determine the patient's ability to be an active participant in self-care, the nurse should prioritize assessment of what variable? A) The patient's attitude B) The patient's learning style C) The patient's nutritional status D) The patient's presurgical level of function

Ans: A Feedback: Amputation of an extremity affects the patient's ability to provide adequate self-care. The patient is encouraged to be an active participant in self-care. The patient and the nurse need to maintain positive attitudes and to minimize fatigue and frustration during the learning process. Balanced nutrition and the patient's learning style are important variables in the rehabilitation process but the patient's attitude is among the most salient variables. The patient's presurgical level of function may or may not affect participation in rehabilitation.

A nurse is providing discharge teaching for a patient who underwent foot surgery. The nurse is collaborating with the occupational therapist and discussing the use of assistive devices. On what variables does the choice of assistive devices primarily depend? A) Patient's general condition, balance, and weight-bearing prescription B) Patient's general condition, strength, and gender C) Patient's motivation, age, and weight-bearing prescription D) Patient's occupation, motivation, and age

Ans: A Feedback: Assistive devices (e.g., crutches, walker) may be needed. The choice of the devices depends on the patient's general condition and balance, and on the weight-bearing prescription. The patient's strength, motivation, and weight restrictions are not what the choice of assistive devices is based on.

A nurse is planning the care of an older adult patient with osteomalacia. What action should the nurse recommend in order to promote vitamin D synthesis? A) Ensuring adequate exposure to sunlight B) Eating a low-purine diet C) Performing cardiovascular exercise while avoiding weight-bearing exercises D) Taking thyroid supplements as ordered

Ans: A Feedback: Because sunlight is necessary for synthesizing vitamin D, patients should be encouraged to spend some time in the sun. A low-purine diet is not a relevant action and thyroid supplements do not directly affect bone function. Action must be taken to prevent fractures, but weight-bearing exercise within safe parameters is not necessarily contraindicated.

An older adult woman's current medication regimen includes alendronate (Fosamax). What outcome would indicate successful therapy? A) Increased bone mass B) Resolution of infection C) Relief of bone pain D) Absence of tumor spread

Ans: A Feedback: Bisphosphonates such as Fosamax increase bone mass and decrease bone loss by inhibiting osteoclast function. These drugs do not treat infection, pain, or tumors.

An elderly female with osteoporosis has been hospitalized. Prior to discharge, when teaching the patient, the nurse should include information about which major complication of osteoporosis? A) Bone fracture B) Loss of estrogen C) Negative calcium balance D) Dowager's hump

Ans: A Feedback: Bone fracture is a major complication of osteoporosis that results when loss of calcium and phosphate increases the fragility of bones. Estrogen deficiencies result from menopause, not osteoporosis. Calcium and vitamin D supplements may be used to support normal bone metabolism, but a negative calcium balance is not a complication of osteoporosis. Dowager's hump results from bone fractures. It develops when repeated vertebral fractures increase spinal curvature.

A patient tells the nurse that he has pain and numbness to his thumb, first finger, and second finger of the right hand. The nurse discovers that the patient is employed as an auto mechanic, and that the pain is increased while working. This may indicate that the patient could possibly have what health problem? A) Carpel tunnel syndrome B) Tendonitis C) Impingement syndrome D) Dupuytren's contracture

Ans: A Feedback: Carpel tunnel syndrome may be manifested by numbness, pain, paresthesia, and weakness along the median nerve. Tendonitis is inflammation of muscle tendons. Impingement syndrome is a general term that describes all lesions that involve the rotator cuff of the shoulder. Dupuytren's contracture is a slowly progressive contracture of the palmar fascia.

The health care team is caring for a patient with osteomalacia. It has been determined that the osteomalacia is caused by malabsorption. What is the usual treatment for osteomalacia caused by malabsorption? A) Supplemental calcium and increased doses of vitamin D B) Exogenous parathyroid hormone and multivitamins C) Colony-stimulating factors and calcitonin D) Supplemental potassium and pancreatic enzymes

Ans: A Feedback: If osteomalacia is caused by malabsorption, increased doses of vitamin D, along with supplemental calcium, are usually prescribed.

The surgical nurse is admitting a patient from postanesthetic recovery following the patient's below-the-knee amputation. The nurse recognizes the patient's high risk for postoperative hemorrhage and should keep which of the following at the bedside? A) A tourniquet B) A syringe preloaded with vitamin K C) A unit of packed red blood cells, placed on ice D) A dose of protamine sulfate

Ans: A Feedback: Immediate postoperative bleeding may develop slowly or may take the form of massive hemorrhage resulting from a loosened suture. A large tourniquet should be in plain sight at the patient's bedside so that, if severe bleeding occurs, it can be applied to the residual limb to control the hemorrhage. PRBCs cannot be kept at the bedside. Vitamin K and protamine sulfate are antidotes to warfarin and heparin, but are not administered to treat active postsurgical bleeding.

A 25-year-old man is involved in a motorcycle accident and injures his arm. The physician diagnoses the man with an intra-articular fracture and splints the injury. The nurse implements the teaching plan developed for this patient. What sequela of intra-articular fractures should the nurse describe regarding this patient? A) Post-traumatic arthritis B) Fat embolism syndrome (FES) C) Osteomyelitis D) Compartment syndrome

Ans: A Feedback: Intra-articular fractures often lead to post-traumatic arthritis. Research does not indicate a correlation between intra-articular fractures and FES, osteomyelitis, or compartment syndrome.

A nurse is assessing a patient who reports a throbbing, burning sensation in the right foot. The patient states that the pain is worst during the day but notes that the pain is relieved with rest. The nurse should recognize the signs and symptoms of what health problem? A) Morton's neuroma B) Pescavus C) Hallux valgus D) Onychocryptosis

Ans: A Feedback: Morton's neuroma is a swelling of the third (lateral) branch of the median plantar nerve, which causes a throbbing, burning pain, usually relieved with rest. Pescavus refers to a foot with an abnormally high arch and a fixed equinus deformity of the forefoot. Hallux valgus (bunion) is a deformity in which the great toe deviates laterally and there is a marked prominence of the medial aspect of the first metatarsal-phalangeal joint and exostosis. Onychocryptosis (ingrown toenail) occurs when the free edge of a nail plate penetrates the surrounding skin, laterally or anteriorly.

A patient presents to a clinic complaining of a leg ulcer that isn't healing; subsequent diagnostic testing suggests osteomyelitis. The nurse is aware that the most common pathogen to cause osteomyelitis is what? A) Staphylococcus aureus B) Proteus C) Pseudomonas D) Escherichia coli

Ans: A Feedback: S. aureus causes over 50% of bone infections. Proteus, Pseudomonas, and E. coli are also causes, but to a lesser extent.

The patient scheduled for a Syme amputation is concerned about the ability to eventually stand on the amputated extremity. How should the nurse best respond to the patient's concern? A) "You will eventually be able to withstand full weight-bearing after the amputation." B) "You will have minimal weight-bearing on this extremity but you'll be taught how to use an assistive device." C) "You likely will not be able to use this extremity but you will receive teaching on use of a wheelchair." D) "You will be fitted for a prosthesis which may or may not allow you to walk."

Ans: A Feedback: Syme amputation (modified ankle disarticulation amputation) is performed most frequently for extensive foot trauma and produces a painless, durable extremity end that can withstand full weight-bearing. Therefore, each of the other teaching statements is incorrect.

A nurse is teaching a patient with osteomalacia about the role of diet. What would be the best choice for breakfast for a patient with osteomalacia? A) Cereal with milk, a scrambled egg, and grapefruit B) Poached eggs with sausage and toast C) Waffles with fresh strawberries and powdered sugar D) A bagel topped with butter and jam with a side dish of grapes

Ans: A Feedback: The best meal option is the one that contains the highest dietary sources of calcium and vitamin D. The best selection among those listed is cereal with milk, and eggs, as these foods contain calcium and vitamin D in a higher quantity over the other menu options.

A nurse is caring for a patient who had a right below-the-knee amputation (BKA). The nurse recognizes the importance of implementing measures that focus on preventing flexion contracture of the hip and maintaining proper positioning. Which of the following measures will best achieve these goals? A) Encouraging the patient to turn from side to side and to assume a prone position B) Initiating ROM exercises of the hip and knee 10 to 12 weeks after the amputation C) Minimizing movement of the flexor muscles of the hip D) Encouraging the patient to sit in a chair for at least 8 hours a day

Ans: A Feedback: The nurse encourages the patient to turn from side to side and to assume a prone position, if possible, to stretch the flexor muscles and to prevent flexion contracture of the hip. Postoperative ROM exercises are started early, because contracture deformities develop rapidly. ROM exercises include hip and knee exercises for patients with BKAs. The nurse also discourages sitting for prolonged periods of time.

A patient has returned to the postsurgical unit from the PACU after an above-the-knee amputation of the right leg. Results of the nurse's initial postsurgical assessment were unremarkable but the patient has called out. The nurse enters the room and observes copious quantities of blood at the surgical site. What should be the nurse's initial action? A) Apply a tourniquet. B) Elevate the residual limb. C) Apply sterile gauze. D) Call the surgeon.

Ans: A Feedback: The nurse should apply a tourniquet in the event of postsurgical hemorrhage. Elevating the limb and applying sterile gauze are likely insufficient to stop the hemorrhage. The nurse should attempt to control the immediate bleeding before contacting the surgeon.

A nurse is writing a care plan for a patient admitted to the emergency department (ED) with an open fracture. The nurse will assign priority to what nursing diagnosis for a patient with an open fracture of the radius? A) Risk for Infection B) Risk for Ineffective Role Performance C) Risk for Perioperative Positioning Injury D) Risk for Powerlessness

Ans: A Feedback: The patient has a significant risk for osteomyelitis and tetanus due to the fact that the fracture is open. Powerlessness and ineffective role performance are psychosocial diagnoses that may or may not apply, and which would be superseded by immediate physiologic threats such as infection. Surgical positioning injury is not plausible, since surgery is not likely indicated.

A 32-year-old patient comes to the clinic complaining of shoulder tenderness, pain, and limited movement. Upon assessment the nurse finds edema. An MRI shows hemorrhage of the rotator cuff tendons and the patient is diagnosed with impingement syndrome. What action should the nurse recommend in order to promote healing? A) Support the affected arm on pillows at night. B) Take prescribed corticosteroids as ordered. C) Put the shoulder through its full range of motion 3 times daily. D) Keep the affected arm in a sling for 2 to 4 weeks.

Ans: A Feedback: The patient should support the affected arm on pillows while sleeping to keep from turning onto the shoulder. Corticosteroids are not commonly prescribed and a sling is not normally necessary. ROM exercises are indicated, but putting the arm through its full ROM may cause damage during the healing process.

A nurse is collaborating with the physical therapist to plan the care of a patient with osteomyelitis. What principle should guide the management of activity and mobility in this patient? A) Stress on the weakened bone must be avoided. B) Increased heart rate enhances perfusion and bone healing. C) Bed rest results in improved outcomes in patients with osteomyelitis. D) Maintenance of baseline ADLs is the primary goal during osteomyelitis treatment

Ans: A Feedback: The patient with osteomyelitis has bone that is weakened by the infective process and must be protected by avoidance of stress on the bone.This risk guides the choice of activity in a patient with osteomyelitis. Bed rest is not normally indicated, however. Maintenance of prediagnosis ADLs may be an unrealistic short-term goal for many patients.

A nurse is caring for a patient who is 12 hours postoperative following foot surgery. The nurse assesses the presence of edema in the foot. What nursing measure will the nurse implement to control the edema? A) Elevate the foot on several pillows. B) Apply warm compresses intermittently to the surgical area. C) Administer a loop diuretic as ordered. D) Increase circulation through frequent ambulation.

Ans: A Feedback: To control the edema in the foot of a patient who experienced foot surgery, the nurse will elevate the foot on several pillows when the patient is sitting or lying. Diuretic therapy is not an appropriate intervention for edema related to inflammation. Intermittent ice packs should be applied to the surgical area during the first 24 to 48 hours after surgery to control edema and provide some pain relief. Ambulation will gradually be resumed based on the guidelines provided by the surgeon.

An 80-year-old man in a long-term care facility has a chronic leg ulcer and states that the area has become increasingly painful in recent days. The nurse notes that the site is now swollen and warm to the touch. The patient should undergo diagnostic testing for what health problem? A) Osteomyelitis B) Osteoporosis C) Osteomalacia D) Septic arthritis

Ans: A Feedback: When osteomyelitis develops from the spread of an adjacent infection, no signs of septicemia are present, but the area becomes swollen, warm, painful, and tender to touch. Osteoporosis is the most prevalent bone disease in the world. Osteomalacia is a metabolic bone disease characterized by inadequate mineralization of bone. Septicarthritis occurs when joints become infected through spread of infection from other parts of the body (hematogenous spread) or directly through trauma or surgical instrumentation.

An emergency department nurse is assessing a 17-year-old soccer player who presented with a knee injury. The patient's description of the injury indicates that his knee was struck medially while his foot was on the ground. The nurse knows that the patient likely has experienced what injury? A) Lateral collateral ligament injury B) Medial collateral ligament injury C) Anterior cruciate ligament injury D) Posterior cruciate ligament injury

Ans: A Feedback: When the knee is struck medially, damage may occur to the lateral collateral ligament. If the knee is struck laterally, damage may occur to the medial collateral ligament. The ACL and PCL are not typically injured in this way.

A nurse is caring for an adult patient diagnosed with a back strain. What health education should the nurse provide to this patient? A) Avoid lifting more than one-third of body weight without assistance. B) Focus on using back muscles efficiently when lifting heavy objects. C) Lift objects while holding the object a safe distance from the body. D) Tighten the abdominal muscles and lock the knees when lifting of an object.

Ans: A Feedback: The nurse will instruct the patient on the safe and correct way to lift objects—using the strong quadriceps muscles of the thighs, with minimal use of the weak back muscles. To prevent recurrence of acute low back pain, the nurse may instruct the patient to avoid lifting more than one-third of his weight without help. The patient should be informed to place the feet a hip-width apart to provide a wide base of support, the person should bend the knees, tighten the abdominal muscles, and lift the object close to the body with a smooth motion, avoiding twisting and jerking.

A patient was brought to the emergency department after a fall. The patient is taken to the operating room to receive a right hip prosthesis. In the immediate postoperative period, what health education should the nurse emphasize? A) "Make sure you don't bring your knees close together." B) "Try to lie as still as possible for the first few days." C) "Try to avoid bending your knees until next week." D) "Keep your legs higher than your chest whenever you can."

Ans: A Feedback: After receiving a hip prosthesis, the affected leg should be kept abducted. Mobility should be encouraged within safe limits. There is no need to avoid knee flexion and the patient's legs do not need to be higher than the level of the chest.

The nurse is helping to set up Buck's traction on an orthopedic patient. How often should the nurse assess circulation to the affected leg? A) Within 30 minutes, then every 1 to 2 hours B) Within 30 minutes, then every 4 hours C) Within 30 minutes, then every 8 hours D) Within 30 minutes, then every shift

Ans: A Feedback: After skin traction is applied, the nurse assesses circulation of the foot or hand within 15 to 30 minutes and then every 1 to 2 hours.

A nurse is planning the care of a patient who will require a prolonged course of skeletal traction. When planning this patient's care, the nurse should prioritize interventions related to which of the following risk nursing diagnoses? A) Risk for Impaired Skin Integrity B) Risk for Falls C) Risk for Imbalanced Fluid Volume D) Risk for Aspiration

Ans: A Feedback: Impaired skin integrity is a high-probability risk in patients receiving traction. Falls are not a threat, due to the patient's immobility. There are not normally high risks of fluid imbalance or aspiration associated with traction.

A nurse is planning the care of a patient who has undergone orthopedic surgery. What main goal should guide the nurse's choice of interventions? A) Improving the patient's level of function B) Helping the patient come to terms with limitations C) Administering medications safely D) Improving the patient's adherence to treatment

Ans: A Feedback: Improving function is the overarching goal after orthopedic surgery. Some patients may need to come to terms with limitations, but this is not true of every patient. Safe medication administration is imperative, but this is not a goal that guides other aspects of care. Similarly, adherence to treatment is important, but this is motivated by the need to improve functional status.

The nurse educator on an orthopedic trauma unit is reviewing the safe and effective use of traction with some recent nursing graduates. What principle should the educator promote? A) Knots in the rope should not be resting against pulleys. B) Weights should rest against the bed rails. C) The end of the limb in traction should be braced by the footboard of the bed. D) Skeletal traction may be removed for brief periods to facilitate the patient's independence.

Ans: A Feedback: Knots in the rope should not rest against pulleys, because this interferes with traction. Weights are used to apply the vector of force necessary to achieve effective traction and should hang freely at all times. To avoid interrupting traction, the limb in traction should not rest against anything. Skeletal traction is never interrupted.

A nurse is admitting a patient to the unit who presented with a lower extremity fracture. What signs and symptoms would suggest to the nurse that the patient may have aperoneal nerve injury? A) Numbness and burning of the foot B) Pallor to the dorsal surface of the foot C) Visible cyanosis in the toes D) Inadequate capillary refill to the toes

Ans: A Feedback: Peroneal nerve injury may result in numbness, tingling, and burning in the feet. Cyanosis, pallor, and decreased capillary refill are signs of inadequate circulation.

The nursing care plan for a patient in traction specifies regular assessments for venous thromboembolism (VTE). When assessing a patient's lower limbs, what sign or symptom is suggestive of deep vein thrombosis (DVT)? A) Increased warmth of the calf B) Decreased circumference of the calf C) Loss of sensation to the calf D) Pale-appearing calf

Ans: A Feedback: Signs of DVT include increased warmth, redness, swelling, and calf tenderness. These findings are promptly reported to the physician for definitive evaluation and therapy. Signs and symptoms of a DVT do not include a decreased circumference of the calf, a loss of sensation in the calf, or a pale-appearing calf.

A nurse is caring for a patient who is postoperative day 1 right hip replacement. How should the nurse position the patient? A) Keep the patient's hips in abduction at all times. B) Keep hips flexed at no less than 90 degrees. C) Elevate the head of the bed to high Fowler's. D) Seat the patient in a low chair as soon as possible.

Ans: A Feedback: The hips should be kept in abduction by an abductor pillow. Hips should not be flexed more than 90 degrees, and the head of bed should not be elevated more than 60 degrees. The patient's hips should be higher than the knees; as such, high seat chairs should be used.

A patient broke his arm in a sports accident and required the application of a cast. Shortly following application, the patient complained of an inability to straighten his fingers and was subsequently diagnosed with Volkmann contracture. What pathophysiologic process caused this complication? A) Obstructed arterial blood flow to the forearm and hand B) Simultaneous pressure on the ulnar and radial nerves C) Irritation of Merkel cells in the patient's skin surfaces D) Uncontrolled muscle spasms in the patient's forearm

Ans: A Feedback: Volkmann contracture occurs when arterial blood flow is restricted to the forearm and hand and results in contractures of the fingers and wrist. It does not result from nerve pressure, skin irritation, or spasms.

A patient has been admitted to the medical unit for the treatment of Paget's disease. When reviewing the medication administration record, the nurse should anticipate what medications? Select all that apply. A) Calcitonin B) Bisphosphonates C) Alkaline phosphatase D) Calcium gluconate E) Estrogen

Ans: A, B Feedback: Bisphosphonates are the cornerstone of Paget therapy in that they stabilize the rapid bone turnover. Calcitonin is also used because it retards bone resorption by decreasing the number and availability of osteoclasts. Alkaline phosphatase is a naturally occurring enzyme, not a drug. Calcium gluconate and estrogen are not used in the treatment of Paget's disease.

A family member of a patient diagnosed with Huntington disease calls you at the clinic. She is requesting help from the Huntingtons Disease Society of America. What kind of help can this patient and family receive from this organization? Select all that apply. A. Information about this disease B. Referrals C. Public education D. Individual assessments E. Appraisals of research studies

Ans: A, B, C Feedback: The Huntingtons Disease Society of America helps patients and families by providing information, referrals, family and public education, and support for research. It does not provide individual assessments or appraisals of individual research studies.

30. The nurse is admitting a patient to the unit who is scheduled for removal of an intracranial mass. What diagnostic procedures might be included in this patients admission orders? Select all that apply. A) Transcranial Doppler flow study B) Cerebral angiography C) MRI D) Cranial radiography E) Electromyelography (EMG)

Ans: A, B, C Feedback: Preoperative diagnostic procedures may include a CT scan to demonstrate the lesion and show the degree of surrounding brain edema, the ventricular size, and the displacement. An MRI scan provides information similar to that of a CT scan with improved tissue contrast, resolution, and anatomic definition. Cerebral angiography may be used to study a tumors blood supply or to obtain information about vascular lesions. Transcranial Doppler flow studies are used to evaluate the blood flow within intracranial blood vessels. Regular x-rays of the skull would not be diagnostic for an intracranial mass. An EMG would not be ordered prior to intracranial surgery to remove a mass.

A nurse is providing care for a patient who has a recent diagnosis of Paget's disease. When planning this patient's nursing care, interventions should address what nursing diagnoses? Select all that apply. A) Impaired Physical Mobility B) Acute Pain C) Disturbed Auditory Sensory Perception D) Risk for Injury E) Risk for Unstable Blood Glucose

Ans: A, B, C, D Feedback: Patient's with Paget's disease are at risk of decreased mobility, pain, hearing loss, and injuries resulting from decreased bone density. Paget's disease does not affect blood glucose levels.

40. An adult patient has sought care for the treatment of headaches that have become increasingly severe and frequent over the past several months. Which of the following questions addresses potential etiological factors? Select all that apply? A) Are you exposed to any toxins or chemicals at work? B) How would you describe your ability to cope with stress? C) What medications are you currently taking? D) When was the last time you were hospitalized? E) Does anyone else in your family struggle with headaches?

Ans: A, B, C, E Feedback: Headaches are multifactorial, and may involve medications, exposure to toxins, family history, and stress. Hospitalization is an unlikely contributor to headaches.

A nurse is planning the care of an older adult patient who will soon be discharged home after treatment for a fractured hip. In an effort to prevent future fractures, the nurse should encourage which of the following? Select all that apply. A) Regular bone density testing B) A high-calcium diet C) Use of falls prevention precautions D) Use of corticosteroids as ordered E) Weight-bearing exercise

Ans: A, B, C, E Feedback: Health promotion measures after an older adult's hip fracture include weight-bearing exercise, promotion of a healthy diet, falls prevention, and bone density testing. Corticosteroids have the potential to reduce bone density and increase the risk for fractures.

An older adult patient experienced a fall and required treatment for a fractured hip on the orthopedic unit. Which of the following are contributory factors to the incidence of falls and fractured hips among the older adult population? Select all that apply. A) Loss of visual acuity B) Adverse medication effects C) Slowed reflexes D) Hearing loss E) Muscle weakness

Ans: A, B, C, E Feedback: Older adults are generally vulnerable to falls and have a high incidence of hip fracture. Weak quadriceps muscles, medication effects, vision loss, and slowed reflexes are among the factors that contribute to the incidence of falls. Decreased hearing is not noted to contribute to the incidence of falls.

35. A 35-year-old woman is diagnosed with a peripheral neuropathy. When making her plan of care, the nurse knows to include what in patient teaching? Select all that apply. A) Inspect the lower extremities for skin breakdown. B) Footwear needs to be accurately sized. C) Immediate family members should be screened for the disease. D) Assistive devices may be needed to reduce the risk of falls. E) Dietary modifications are likely necessary.

Ans: A, B, D The plan of care includes inspection of the lower extremities for skin breakdown. Footwear should be accurately sized. Assistive devices, such as a walker or cane, may decrease the risk of falls. Bath water temperature is checked to avoid thermal injury. Peripheral neuropathies do not have a genetic component and diet is unrelated.

A patient with an inoperable brain tumor has been told that he has a short life expectancy. On what aspects of assessment and care should the home health nurse focus? Select all that apply. A. Pain control B. Management of treatment complications C. Interpretation of diagnostic tests D. Assistance with self-care E. Administration of treatments

Ans: A, B, D, E Feedback: Home care needs and interventions focus on four major areas: palliation of symptoms and pain control, assistance in self-care, control of treatment complications, and administration of specific forms of treatment, such as parenteral nutrition. Interpretation of diagnostic tests is normally beyond the purview of the nurse.

25. The nurse is caring for a 77-year-old woman with MS. She states that she is very concerned about the progress of her disease and what the future holds. The nurse should know that elderly patients with MS are known to be particularly concerned about what variables? Select all that apply. A) Possible nursing home placement B) Pain associated with physical therapy C) Increasing disability D) Becoming a burden on the family E) Loss of appetite

Ans: A, C, D Elderly patients with MS are particularly concerned about increasing disability, family burden, marital concern, and the possible future need for nursing home care. Older adults with MS are not noted to have particular concerns regarding the pain of therapy or loss of appetite.

A patient has had a brace prescribed to facilitate recovery from a knee injury. What are the potential therapeutic benefits of a brace? Select all that apply. A) Preventing additional injury B) Immobilizing prior to surgery C) Providing support D) Controlling movement E) Promoting bone remodeling

Ans: A, C, D Feedback: Braces (i.e., orthoses) are used to provide support, control movement, and prevent additional injury. They are not used to immobilize body parts or to facilitate bone remodeling.

24. The nurse caring for a patient in a persistent vegetative state is regularly assessing for potential complications. Complications of neurologic dysfunction for which the nurse should assess include which of the following? Select all that apply. A) Contractures B) Hemorrhage C) Pressure ulcers D) Venous thromboembolism E) Pneumonia

Ans: A, C, D, E Feedback: Based on the assessment data, potential complications may include respiratory distress or failure, pneumonia, aspiration, pressure ulcer, deep vein thrombosis (DVT), and contractures. The pathophysiology of decreased LOC does not normally create a heightened risk for hemorrhage.

A nurse is caring for a patient who is being assessed following complaints of severe and persistent low back pain. The patient is scheduled for diagnostic testing in the morning. Which of the following are appropriate diagnostic tests for assessing low back pain? that apply. A) Computed tomography (CT) B) Angiography C) Magnetic resonance imaging (MRI) D) Ultrasound E) X-ray

Ans: A, C, D, E Feedback: A variety of diagnostic tests can be used to address lower back pain, including CT, MRI, ultrasound, and X-rays. Angiography is not related to the etiology of back pain.

The nurse is caring for a patient diagnosed with Parkinsons disease. The patient is having increasing problems with rising from the sitting to the standing position. What should the nurse suggest to the patient to use that will aid in getting from the sitting to the standing position as well as aid in improving bowel elimination? A. Use of a bedpan B. Use of a raised toilet seat C. Sitting quietly on the toilet every 2 hours D. Following the outlined bowel program

Ans: B Feedback: A raised toilet seat is useful, because the patient has difficulty in moving from a standing to a sitting position. A handicapped toilet is not high enough and will not aid in improving bowel elimination. Sitting quietly on the toilet every 2 hours will not aid in getting from the sitting to standing position; neither will following the outlined bowel program.

A patient has just returned to the unit from the PACU after surgery for a tumor within the spine. The patient complains of pain. When positioning the patient for comfort and to reduce injury to the surgical site, the nurse will position to patient in what position? A. In the high Fowlers position B. In a flat side-lying position C. In the Trendelenberg position D. In the reverse Trendelenberg position

Ans: B Feedback: After spinal surgery, the bed is usually kept flat initially. The side-lying position is usually the most comfortable because this position imposes the least pressure on the surgical site. The Fowlers position, Trendelenberg position, and reverse Trendelenberg position are inappropriate for this patient because they would result in increased pain and complications.

The nurse is caring for a patient who is scheduled for a cervical discectomy the following day. During health education, the patient should be made aware of what potential complications? A. Vertebral fracture B. Hematoma at the surgical site C. Scoliosis D. Renal trauma

Ans: B Feedback: Based on all the assessment data, the potential complications of diskectomy may include hematoma at the surgical site, resulting in cord compression and neurologic deficit and recurrent or persistent pain after surgery. Renal trauma and fractures are unlikely; scoliosis is a congenital malformation of the spine.

A patient diagnosed with a pituitary adenoma has arrived on the neurologic unit. When planning the patients care, the nurse should be aware that the effects of the tumor will primarily depend on what variable? A. Whether the tumor utilizes aerobic or anaerobic respiration B. The specific hormones secreted by the tumor C. The patients pre-existing health status D. Whether the tumor is primary or the result of metastasis

Ans: B Feedback: Functioning pituitary tumors can produce one or more hormones normally produced by the anterior pituitary and the effects of the tumor depend largely on the identity of these hormones. This variable is more significant than the patients health status or whether the tumor is primary versus secondary. Anaerobic and aerobic respiration is not relevant.

The nurse in an extended care facility is planning the daily activities of a patient with postpolio syndrome. The nurse recognizes the patient will best benefit from physical therapy when it is scheduled at what time? A. Immediately after meals B. In the morning C. Before bedtime D. In the early evening

Ans: B Feedback: Important activities for patients with postpolio syndrome should be planned for the morning, as fatigue often increases in the afternoon and evening.

A 25-year-old female patient with brain metastases is considering her life expectancy after her most recent meeting with her oncologist. Based on the fact that the patient is not receiving treatment for her brain metastases, what is the nurses most appropriate action? A. Promoting the patients functional status and ADLs B. Ensuring that the patient receives adequate palliative care C. Ensuring that the family does not tell the patient that her condition is terminal D. Promoting adherence to the prescribed medication regimen

Ans: B Feedback: Patients with intracerebral metastases who are not treated have a steady downhill course with a limited survival time, whereas those who are treated may survive for slightly longer periods, but for most cure is not possible. Palliative care is thus necessary. This is a priority over promotion of function and the family should not normally withhold information from the patient. Adherence to medications such as analgesics is important, but palliative care is a high priority.

A patient has been admitted to the neurologic unit for the treatment of a newly diagnosed brain tumor. The patient has just exhibited seizure activity for the first time. What is the nurses priority response to this event? A. Identify the triggers that precipitated the seizure. B. Implement precautions to ensure the patients safety. C. Teach the patients family about the relationship between brain tumors and seizure activity. D. Ensure that the patient is housed in a private room.

Ans: B Feedback: Patients with seizures are carefully monitored and protected from injury. Patient safety is a priority over health education, even though this is appropriate and necessary. Specific triggers may or may not be evident; identifying these is not the highest priority. A private room is preferable, but not absolutely necessary.

The nurse is caring for a boy who has muscular dystrophy. When planning assistance with the patients ADLs, what goal should the nurse prioritize? A. Promoting the patients recovery from the disease B. Maximizing the patients level of function C. Ensuring the patients adherence to treatment D. Fostering the familys participation in care

Ans: B Feedback: Priority for the care of the child with muscular dystrophy is the need to maximize the patients level of function. Family participation is also important, but should be guided by this goal. Adherence is not a central goal, even though it is highly beneficial, and the disease is not curable.

A patient newly diagnosed with a cervical disk herniation is receiving health education from the clinic nurse. What conservative management measures should the nurse teach the patient to implement? A. Perform active ROM exercises three times daily. B. Sleep on a firm mattress. C. Apply cool compresses to the back of the neck daily. D. Wear the cervical collar for at least 2 hours at a time.

Ans: B Feedback: Proper positioning on a firm mattress and bed rest for 1 to 2 days may bring dramatic relief from pain. The patient may need to wear a cervical collar 24 hours a day during the acute phase of pain from a cervical disk herniation. Hot, moist compresses applied to the back of the neck will increase blood flow to the muscles and help relax the spastic muscles.

The clinic nurse caring for a patient with Parkinsons disease notes that the patient has been taking levodopa and carbidopa (Sinemet) for 7 years. For what common side effect of Sinemet would the nurse assesses this patient? A. Pruritus B. Dyskinesia C. Lactose intolerance D. Diarrhea

Ans: B Feedback: Within 5 to 10 years of taking levodopa, most patients develop a response to the medication characterized by dyskinesia (abnormal involuntary movements). Another potential complication of longterm dopaminergic medication use is neuroleptic malignant syndrome characterized by severe rigidity, stupor, and hyperthermia. Side effects of long-term Sinemet therapy are not pruritus, lactose intolerance, or diarrhea.

15. A patient with herpes simplex virus encephalitis (HSV) has been admitted to the ICU. What medication would the nurse expect the physician to order for the treatment of this disease process? A) Cyclosporine (Neoral) B) Acyclovir (Zovirax) C) Cyclobenzaprine (Flexeril) D) Ampicillin (Prinicpen)

Ans: B Acyclovir (Zovirax) or ganciclovir (Cytovene), antiviral agents, are the medications of choice in the treatment of HSV. The mode of action is the inhibition of viral DNA replication. To prevent relapse, treatment would continue for up to 3 weeks. Cyclosporine is an immunosuppressant and antirheumatic. Cyclobenzaprine is a centrally acting skeletal muscle relaxant. Ampicillin, an antibiotic, is ineffective against viruses.

10. The nurse is developing a plan of care for a patient newly diagnosed with Bell's palsy. The nurse's plan of care should address what characteristic manifestation of this disease? A) Tinnitus B) Facial paralysis C) Pain at the base of the tongue D) Diplopia

Ans: B Bell's palsy is characterized by facial dysfunction, weakness, and paralysis. It does not result in diplopia, pain at the base of the tongue, or tinnitus.

38. A patient with MS has developed dysphagia as a result of cranial nerve dysfunction. What nursing action should the nurse consequently perform? A) Arrange for the patient to receive a low residue diet. B) Position the patient upright during feeding. C) Suction the patient following each meal. D) Withhold liquids until the patient has finished eating.

Ans: B Correct, upright positioning is necessary to prevent aspiration in the patient with dysphagia. There is no need for a low-residue diet and suctioning should not be performed unless there is an apparent need. Liquids do not need to be withheld during meals in order to prevent aspiration.

3. The nurse is caring for a patient with multiple sclerosis (MS). The patient tells the nurse the hardest thing to deal with is the fatigue. When teaching the patient how to reduce fatigue, what action should the nurse suggest? A) Taking a hot bath at least once daily B) Resting in an air-conditioned room whenever possible C) Increasing the dose of muscle relaxants D) Avoiding naps during the day

Ans: B Fatigue is a common symptom of patients with MS. Lowering the body temperature by resting in an air-conditioned room may relieve fatigue; however, extreme cold should be avoided. A hot bath or shower can increase body temperature, producing fatigue. Muscle relaxants, prescribed to reduce spasticity, can cause drowsiness and fatigue. Planning for frequent rest periods and naps can relieve fatigue. Other measures to reduce fatigue in the patient with MS include treating depression, using occupational therapy to learn energy conservation techniques, and reducing spasticity.

12. The nurse is participating in the care of a patient with increased ICP. What diagnostic test is contraindicated in this patients treatment? A) Computed tomography (CT) scan B) Lumbar puncture C) Magnetic resonance imaging (MRI) D) Venous Doppler studies

Ans: B Feedback: A lumbar puncture in a patient with increased ICP may cause the brain to herniate from the withdrawal of fluid and change in pressure during the lumbar puncture. Herniation of the brain is a dire and frequently fatal event. CT, MRI, and venous Doppler are considered noninvasive procedures and they would not affect the ICP itself.

38. A nurse is caring for a patient who experiences debilitating cluster headaches. The patient should be taught to take appropriate medications at what point in the course of the onset of a new headache? A) As soon as the patients pain becomes unbearable B) As soon as the patient senses the onset of symptoms C) Twenty to 30 minutes after the onset of symptoms D) When the patient senses his or her symptoms peaking

Ans: B Feedback: A migraine or a cluster headache in the early phase requires abortive medication therapy instituted as soon as possible. Delaying medication administration would lead to unnecessary pain.

16. A clinic nurse is caring for a patient diagnosed with migraine headaches. During the patient teaching session, the patient questions the nurse regarding alcohol consumption. What would the nurse be correct in telling the patient about the effects of alcohol? A) Alcohol causes hormone fluctuations. B) Alcohol causes vasodilation of the blood vessels. C) Alcohol has an excitatory effect on the CNS. D) Alcohol diminishes endorphins in the brain.

Ans: B Feedback: Alcohol causes vasodilation of the blood vessels and may exacerbate migraine headaches. Alcohol has a depressant effect on the CNS. Alcohol does not cause hormone fluctuations, nor does it decrease endorphins (morphine-like substances produced by the body) in the brain.

10. While completing a health history on a patient who has recently experienced a seizure, the nurse would assess for what characteristic associated with the postictal state? A) Epileptic cry B) Confusion C) Urinary incontinence D) Body rigidity

Ans: B Feedback: In the postictal state (after the seizure), the patient is often confused and hard to arouse and may sleep for hours. The epileptic cry occurs from the simultaneous contractions of the diaphragm and chest muscles that occur during the seizure. Urinary incontinence and intense rigidity of the entire body are followed by alternating muscle relaxation and contraction (generalized tonicclonic contraction) during the seizure.

9. A patient exhibiting an altered level of consciousness (LOC) due to blunt-force trauma to the head is admitted to the ED. The physician determines the patients injury is causing increased intracranial pressure (ICP). The nurse should gauge the patients LOC on the results of what diagnostic tool? A) Monro-Kellie hypothesis B) Glasgow Coma Scale C) Cranial nerve function D) Mental status examination

Ans: B Feedback: LOC, a sensitive indicator of neurologic function, is assessed based on the criteria in the Glasgow Coma Scale: eye opening, verbal response, and motor response. The Monro-Kellie hypothesis states that because of the limited space for expansion within the skull, an increase in any one of the components (blood, brain tissue, cerebrospinal fluid) causes a change in the volume of the others. Cranial nerve function and the mental status examination would be part of the neurologic examination for this patient, but would not be the priority in evaluating LOC.

2. The nurse is providing care for a patient who is unconscious. What nursing intervention takes highest priority? A) Maintaining accurate records of intake and output B) Maintaining a patent airway C) Inserting a nasogastric (NG) tube as ordered D) Providing appropriate pain control

Ans: B Feedback: Maintaining a patent airway always takes top priority, even though each of the other listed actions is necessary and appropriate.

13. The nurse is caring for a patient who is in status epilepticus. What medication does the nurse know may be given to halt the seizure immediately? A) Intravenous phenobarbital (Luminal) B) Intravenous diazepam (Valium) C) Oral lorazepam (Ativan) D) Oral phenytoin (Dilantin)

Ans: B Feedback: Medical management of status epilepticus includes IV diazepam (Valium) and IV lorazepam (Ativan) given slowly in an attempt to halt seizures immediately. Other medications (phenytoin, phenobarbital) are given later to maintain a seizure-free state. Oral medications are not given during status epilepticus.

34. A school nurse is called to the playground where a 6-year-old girl has been found unresponsive and staring into space, according to the playground supervisor. How would the nurse document the girls activity in her chart at school? A) Generalized seizure B) Absence seizure C) Focal seizure D) Unclassified seizure

Ans: B Feedback: Staring episodes characterize an absence seizure, whereas focal seizures, generalized seizures, and unclassified seizures involve uncontrolled motor activity.

15. The neurologic ICU nurse is admitting a patient following a craniotomy using the supratentorial approach. How should the nurse best position the patient? A) Position the patient supine. B) Maintain head of bed (HOB) elevated at 30 to 45 degrees. C) Position patient in prone position. D) Maintain bed in Trendelenberg position.

Ans: B Feedback: The patient undergoing a craniotomy with a supratentorial (above the tentorium) approach should be placed with the HOB elevated 30 to 45 degrees, with the neck in neutral alignment. Each of the other listed positions would cause a dangerous elevation in ICP.

4. The nurse is caring for a patient who is postoperative following a craniotomy. When writing the plan of care, the nurse identifies a diagnosis of deficient fluid volume related to fluid restriction and osmotic diuretic use. What would be an appropriate intervention for this diagnosis? A) Change the patients position as indicated. B) Monitor serum electrolytes. C) Maintain NPO status. D) Monitor arterial blood gas (ABG) values.

Ans: B Feedback: The postoperative fluid regimen depends on the type of neurosurgical procedure and is determined on an individual basis. The volume and composition of fluids are adjusted based on daily serum electrolyte values, along with fluid intake and output. Fluids may have to be restricted in patients with cerebral edema. Changing the patients position, maintaining an NPO status, and monitoring ABG values do not relate to the nursing diagnosis of deficient fluid volume.

14. To alleviate pain associated with trigeminal neuralgia, a patient is taking Tegretol (carbamazepine). What health education should the nurse provide to the patient before initiating this treatment? A) Concurrent use of calcium supplements is contraindicated. B) Blood levels of the drug must be monitored. C) The drug is likely to cause hyperactivity and agitation. D) Tegretol can cause tinnitus during the first few days of treatment.

Ans: B Side effects of Tegretol include nausea, dizziness, drowsiness, and aplastic anemia. The patient must also be monitored for bone marrow depression during long-term therapy. Skin discoloration, insomnia, and tinnitus are not side effects of Tegretol.

8. A patient with metastatic cancer has developed trigeminal neuralgia and is taking carbamazepine (Tegretol) for pain relief. What principle applies to the administration of this medication? A) Tegretol is not known to have serious adverse effects. B) The patient should be monitored for bone marrow depression. C) Side effects of the medication include renal dysfunction. D) The medication should be first taken in the maximum dosage form to be effective.

Ans: B The anticonvulsant agents carbamazepine (Tegretol) and phenytoin (Dilantin) relieve pain in most patients diagnosed with trigeminal neuralgia by reducing the transmission of impulses at certain nerve terminals. Side effects include nausea, dizziness, drowsiness, and aplastic anemia. Carbamazepine should be gradually increased until pain relief is obtained.

30. A patient diagnosed with myasthenia gravis has been hospitalized to receive plasmapheresis for a myasthenic exacerbation. The nurse knows that the course of treatment for plasmapheresis in a patient with myasthenia gravis is what? A) Every day for 1 week B) Determined by the patient's response C) Alternate days for 10 days D) Determined by the patient's weight

Ans: B The typical course of plasmapheresis consists of daily or alternate-day treatment, and the number of treatments is determined by the patient's response.

9. A male patient presents to the clinic complaining of a headache. The nurse notes that the patient is guarding his neck and tells the nurse that he has stiffness in the neck area. The nurse suspects the patient may have meningitis. What is another well-recognized sign of this infection? A) Negative Brudzinski's sign B) Positive Kernig's sign C) Hyperpatellar reflex D) Sluggish pupil reaction

Ans: B Meningeal irritation results in a number of well-recognized signs commonly seen in meningitis, such as a positive Kernig's sign, a positive Brudzinski's sign, and photophobia. Hyperpatellar reflex and a sluggish pupil reaction are not commonly recognized signs of meningitis.

A nurse admits a patient who has a fracture of the nose that has resulted in a skin tear and involvement of the mucous membranes of the nasal passages. The orthopedic nurse is aware that this description likely indicates which type of fracture? A) Compression B) Compound C) Impacted D) Transverse

Ans: B Feedback: A compound fracture involves damage to the skin or mucous membranes and is also called an open fracture. A compression fracture involves compression of bone and is seen in vertebral fractures. An impacted fracture occurs when a bone fragment is driven into another bone fragment. A transverse fracture occurs straight across the bone shaft.

A patient has been admitted to the hospital with a spontaneous vertebral fracture related to osteoporosis. Which of the following nursing diagnoses must be addressed in the plan of care? A) Risk for Aspiration Related to Vertebral Fracture B) Constipation Related to Vertebral Fracture C) Impaired Swallowing Related to Vertebral Fracture D) Decreased Cardiac Output Related to Vertebral Fracture

Ans: B Feedback: Constipation is a problem related to immobility and medications used to treat vertebral fractures. The patient's risks of aspiration, dysphagia, and decreased cardiac output are not necessarily heightened.

A patient has returned to the unit after undergoing limb-sparing surgery to remove a metastatic bone tumor. The nurse providing postoperative care in the days following surgery assesses for what complication from surgery? A) Deficient fluid volume B) Delayed wound healing C) Hypocalcemia D) Pathologic fractures

Ans: B Feedback: Delayed wound healing is a complication of surgery due to tissue trauma from the surgery. Nutritional deficiency is usually due to the effects of chemotherapy and radiation therapy, which may cause weight loss. Pathologic fractures are not a complication of surgery.

A patient with diabetes is attending a class on the prevention of associated diseases. What action should the patient perform to reduce the risk of osteomyelitis? A) Increase calcium and vitamin intake. B) Perform meticulous foot care. C) Exercise 3 to 4 times weekly for at least 30 minutes. D) Take corticosteroids as ordered.

Ans: B Feedback: Diabetic foot ulcers have a high potential for progressing to osteomyelitis. Meticulous foot care can help mitigate this risk. Corticosteroids can exacerbate the risk of osteomyelitis. Increased intake of calcium and vitamins as well as regular exercise are beneficial health promotion exercises, but they do not directly reduce the risk of osteomyelitis.

An older adult patient has fallen in her home and is brought to the emergency department by ambulance with a suspected fractured hip. X-rays confirm a fracture of the left femoral neck. When planning assessments during the patient's presurgical care, the nurse should be aware of the patient's heightened risk of what complication? A) Osteomyelitis B) Avascular necrosis C) Phantom pain D) Septicemia

Ans: B Feedback: Fractures of the neck of the femur may damage the vascular system that supplies blood to the head and the neck of the femur, and the bone may become ischemic. For this reason, AVN is common in patients with femoral neck fractures. Infections are not immediate complications and phantom pain applies to patients with amputations, not hip fractures.

A nurse is caring for a patient who is being treated in the hospital for a spontaneous vertebral fracture related to osteoporosis. The nurse should address the nursing diagnosis of Acute Pain Related to Fracture by implementing what intervention? A) Maintenance of high Fowler's positioning whenever possible B) Intermittent application of heat to the patient's back C) Use of a pressure-reducing mattress D) Passive range of motion exercises

Ans: B Feedback: Intermittent local heat and back rubs promote muscle relaxation following osteoporotic vertebral fractures. High Fowler's positioning is likely to exacerbate pain. The mattress must be adequately supportive, but pressure reduction is not necessarily required. Passive range of motion exercises to the back would cause pain and impair healing.

A patient has come to the clinic for a routine annual physical. The nurse practitioner notes a palpable, painless projection of bone at the patient's shoulder. The projection appears to be at the distal end of the humerus. The nurse should suspect the presence of which of the following? A) Osteomyelitis B) Osteochondroma C) Osteomalacia D) Paget's disease

Ans: B Feedback: Osteochondroma is the most common benign bone tumor. It usually occurs as a large projection of bone at the end of long bones (at the knee or shoulder). Osteomyelitis, osteomalacia, and Paget's disease do not involve the development of excess bone tissue.

A patient with diabetes has been diagnosed with osteomyelitis. The nurse notes that the patient's right foot is pale and mottled, cool to touch, with a capillary refill of greater than 3 seconds. The nurse should suspect what type of osteomyelitis? A) Hematogenous osteomyelitis B) Osteomyelitis with vascular insufficiency C) Contiguous-focus osteomyelitis D) Osteomyelitis with muscular deterioration

Ans: B Feedback: Osteomyelitis is classified as hematogenous osteomyelitis (i.e., due to blood-borne spread of infection); contiguous-focus osteomyelitis, from contamination from bone surgery, open fracture, or traumatic injury (e.g., gunshot wound); and osteomyelitis with vascular insufficiency, seen most commonly among patients with diabetes and peripheral vascular disease, most commonly affecting the feet. Osteomyelitis with muscular deterioration does not exist.

Which of the following patients should the nurse recognize as being at the highest risk for the development of osteomyelitis? A) A middle-age adult who takes ibuprofen daily for rheumatoid arthritis B) An elderly patient with an infected pressure ulcer in the sacral area C) A 17-year-old football player who had orthopedic surgery 6 weeks prior D) An infant diagnosed with jaundice

Ans: B Feedback: Patients who are at high risk of osteomyelitis include those who are poorly nourished, elderly, and obese. The elderly patient with an infected sacral pressure ulcer is at the greatest risk for the development of osteomyelitis, as this patient has two risk factors: age and the presence of a soft-tissue infection that has the potential to extend into the bone. The patient with rheumatoid arthritis has one risk factor and the infant with jaundice has no identifiable risk factors. The patient 6 weeks postsurgery is beyond the usual window of time for the development of a postoperative surgical wound infection.

An elite high school football player has been diagnosed with a shoulder dislocation. The patient has been treated and is eager to resume his role on his team, stating that he is not experiencing pain. What should the nurse emphasize during health education? A) The need to take analgesia regardless of the short-term absence of pain B) The importance of adhering to the prescribed treatment and rehabilitation regimen C) The fact that he has a permanently increased risk of future shoulder dislocations D) The importance of monitoring for intracapsular bleeding once he resumes playing

Ans: B Feedback: Patients who have experienced sports-related injuries are often highly motivated to return to their previous level of activity. Adherence to restriction of activities and gradual resumption of activities needs to be reinforced. Appropriate analgesia use must be encouraged, but analgesia does not necessarily have to be taken in the absence of pain. If healing is complete, the patient does not likely have a greatly increased risk of reinjury. Dislocations rarely cause bleeding after the healing process.

A nurse is caring for a patient who has suffered an unstable thoracolumbar fracture. Which of the following is the priority during nursing care? A) Preventing infection B) Maintaining spinal alignment C) Maximizing function D) Preventing increased intracranial pressure

Ans: B Feedback: Patients with an unstable fracture must have their spine in alignment at all times in order to prevent neurologic damage. This is a greater threat, and higher priority, than promoting function and preventing infection, even though these are both valid considerations. Increased ICP is not a high risk.

Which of the following is the most appropriate nursing intervention to facilitate healing in a patient who has suffered a hip fracture? A) Administer analgesics as required. B) Place a pillow between the patient's legs when turning. C) Maintain prone positioning at all times. D) Encourage internal and external rotation of the affected leg.

Ans: B Feedback: Placing a pillow between the patient's legs when turning prevents adduction and supports the patient's legs. Administering analgesics addresses pain but does not directly protect bone remodeling and promote healing. Rotation of the affected leg can cause dislocation and must be avoided. Prone positioning does not need to be maintained at all times.

A patient presents at a clinic complaining of pain in his heel so bad that it inhibits his ability to walk. The patient is subsequently diagnosed with plantar fasciitis. This patient's plan of care should include what intervention? A) Wrapping the affected area in lamb's wool or gauze to relieve pressure B) Gently stretching the foot and the Achilles tendon C) Wearing open-toed shoes at all times D) Applying topical analgesic ointment to plantar surface each morning

Ans: B Feedback: Plantar fasciitis leads to pain that is localized to the anterior medial aspect of the heel and diminishes with gentle stretching of the foot and Achilles tendon. Dressings of any kind are not of therapeutic benefit and analgesic ointments do not address the pathology of the problem. Open-toed shoes are of no particular benefit.

The orthopedic nurse should assess for signs and symptoms of Volkmann's contracture if a patient has fractured which of the following bones? A) Femur B) Humerus C) Radial head D) Clavicle

Ans: B Feedback: The most serious complication of a supracondylar fracture of the humerus is Volkmann's ischemic contracture, which results from antecubital swelling or damage to the brachial artery. This complication is specific to humeral fractures.

A patient has sustained a long bone fracture and the nurse is preparing the patient's care plan. Which of the following should the nurse include in the care plan? A) Administer vitamin D and calcium supplements as ordered. B) Monitor temperature and pulses of the affected extremity. C) Perform passive range of motion exercises as tolerated. D) Administer corticosteroids as ordered.

Ans: B Feedback: The nurse should include monitoring for sufficient blood supply by assessing the color, temperature, and pulses of the affected extremity. Weight-bearing exercises are encouraged, but passive ROM exercises have the potential to cause pain and inhibit healing. Corticosteroids, vitamin D, and calcium are not normally administered.

A patient with a simple arm fracture is receiving discharge education from the nurse. What would the nurse instruct the patient to do? A) Elevate the affected extremity to shoulder level when at rest. B) Engage in exercises that strengthen the unaffected muscles. C) Apply topical anesthetics to accessible skin surfaces as needed. D) Avoid using analgesics so that further damage is not masked.

Ans: B Feedback: The nurse will encourage the patient to engage in exercises that strengthen the unaffected muscles. Comfort measures may include appropriate use of analgesics and elevation of the affected extremity to the heart level. Topical anesthetics are not typically used.

The nurse is providing care for a patient who has had a below-the-knee amputation. The nurse enters the patient's room and finds him resting in bed with his residual limb supported on pillow. What is the nurse's most appropriate action? A) Inform the surgeon of this finding. B) Explain the risks of flexion contracture to the patient. C) Transfer the patient to a sitting position. D) Encourage the patient to perform active ROM exercises with the residual limb.

Ans: B Feedback: The residual limb should not be placed on a pillow, because a flexion contracture of the hip may result. There is no acute need to contact the patient's surgeon. Encouraging exercise or transferring the patient does not address the risk of flexion contracture.

A nurse's assessment of a patient's knee reveals edema, tenderness, muscle spasms, and ecchymosis. The patient states that 2 days ago he ran 10 miles and now it "really hurts to stand up." The nurse should plan care based on the belief that the patient has experienced what? A) A first-degree strain B) A second-degree strain C) A first-degree sprain D) A second-degree sprain

Ans: B Feedback: A second-degree strain involves tearing of muscle fibers and is manifested by notable loss of load-bearing strength with accompanying edema, tenderness, muscle spasm, and ecchymosis. A first-degree strain reflects tearing of a few muscle fibers and is accompanied by minor edema, tenderness, and mild muscle spasm, without noticeable loss of function. However, this patient states a loss of function. A sprain normally involves twisting, which is inconsistent with the patient's overuse injury.

A physician writes an order to discontinue skeletal traction on an orthopedic patient. The nurse should anticipate what subsequent intervention? A) Application of a walking boot B) Application of a cast C) Education on how to use crutches D) Passive range of motion exercises

Ans: B Feedback: After skeletal traction is discontinued, internal fixation, casts, or splints are then used to immobilize and support the healing bone. The use of a walking boot, crutches, or ROM exercises could easily damage delicate, remodeled bone.

A nurse is reviewing a patient's activities of daily living prior to discharge from total hip replacement. The nurse should identify what activity as posing a potential risk for hip dislocation? A) Straining during a bowel movement B) Bending down to put on socks C) Lifting items above shoulder level D) Transferring from a sitting to standing position

Ans: B Feedback: Bending to put on socks or shoes can cause hip dislocation. None of the other listed actions poses a serious threat to the integrity of the new hip.

A patient has recently been admitted to the orthopedic unit following total hip arthroplasty. The patient has a closed suction device in place and the nurse has determined that there were 320 mL of output in the first 24 hours. How should the nurse best respond to this assessment finding? A) Inform the primary care provider promptly. B) Document this as an expected assessment finding. C) Limit the patient's fluid intake to 2 liters for the next 24 hours. D) Administer a loop diuretic as ordered.

Ans: B Feedback: Drainage of 200 to 500 mL in the first 24 hours is expected. Consequently, the nurse does not need to inform the physician. Fluid restriction and medication administration are not indicated.

A patient has suffered a muscle strain and is complaining of pain that she rates at 6 on a 10-point scale. The nurse should recommend what action? A) Taking an opioid analgesic as ordered B) Applying a cold pack to the injured site C) Performing passive ROM exercises D) Applying a heating pad to the affected muscle

Ans: B Feedback: Most pain can be relieved by elevating the involved part, applying cold packs, and administering analgesics as prescribed. Heat may exacerbate the pain by increasing blood circulation, and ROM exercises would likely be painful. Analgesia is likely necessary, but NSAIDs would be more appropriate than opioids.

The orthopedic surgeon has prescribed balanced skeletal traction for a patient. What advantage is conferred by balanced traction? A) Balanced traction can be applied at night and removed during the day. B) Balanced traction allows for greater patient movement and independence than other forms of traction. C) Balanced traction is portable and may accompany the patient's movements. D) Balanced traction facilitates bone remodeling in as little as 4 days.

Ans: B Feedback: Often, skeletal traction is balanced traction, which supports the affected extremity, allows for some patient movement, and facilitates patient independence and nursing care while maintaining effective traction. It is not portable, however, and it cannot be removed. Bone remodeling takes longer than 4 days.

A patient is being prepared for a total hip arthroplasty, and the nurse is providing relevant education. The patient is concerned about being on bed rest for several days after the surgery. The nurse should explain what expectation for activity following hip replacement? A) "Actually, patients are only on bed rest for 2 to 3 days before they begin walking with assistance." B) "The physical therapist will likely help you get up using a walker the day after your surgery." C) "Our goal will actually be to have you walking normally within 5 days of your surgery." D) "For the first two weeks after the surgery, you can use a wheelchair to meet your mobility needs."

Ans: B Feedback: Patients post-THA begin ambulation with the assistance of a walker or crutches within a day after surgery. Wheelchairs are not normally utilized. Baseline levels of mobility are not normally achieved until several weeks after surgery, however.

A nurse is assessing a patient who is receiving traction. The nurse's assessment confirms that the patient is able to perform plantar flexion. What conclusion can the nurse draw from this finding? A) The leg that was assessed is free from DVT. B) The patient's tibial nerve is functional. C) Circulation to the distal extremity is adequate. D) The patient does not have peripheral neurovascular dysfunction.

Ans: B Feedback: Plantar flexion demonstrates function of the tibial nerve. It does not demonstrate the absence of DVT and does not allow the nurse to ascertain adequate circulation. The nurse must perform more assessments on more sites in order to determine an absence of peripheral neurovascular dysfunction.

While assessing a patient who has had knee replacement surgery, the nurse notes that the patient has developed a hematoma at the surgical site. The affected leg has a decreased pedal pulse. What would be the priority nursing diagnosis for this patient? A) Risk for Infection B) Risk for Peripheral Neurovascular Dysfunction C) Unilateral Neglect D) Disturbed Kinesthetic Sensory Perception

Ans: B Feedback: The hematoma may cause an interruption of tissue perfusion, so the most appropriate nursing diagnosis is Risk of Peripheral Neurovascular Dysfunction. There is also an associated risk for infection because of the hematoma, but impaired neurovascular function is a more acute threat. Unilateral neglect and impaired sensation are lower priorities than neurovascular status.

A patient with a right tibial fracture is being discharged home after having a cast applied. What instruction should the nurse provide in relationship to the patient's cast care? A) "Cover the cast with a blanket until the cast dries." B) "Keep your right leg elevated above heart level." C) "Use a clean object to scratch itches inside the cast." D) "A foul smell from the cast is normal after the first few days."

Ans: B Feedback: The leg should be elevated to promote venous return and prevent edema. The cast shouldn't be covered while drying because this will cause heat buildup and prevent air circulation. No foreign object should be inserted inside the cast because of the risk of cutting the skin and causing an infection. A foul smell from a cast is never normal and may indicate an infection.

A nurse is caring for an older adult patient who is preparing for discharge following recovery from a total hip replacement. Which of the following outcomes must be met prior to discharge? A) Patient is able to perform ADLs independently. B) Patient is able to perform transfers safely. C) Patient is able to weight-bear equally on both legs. D) Patient is able to demonstrate full ROM of the affected hip.

Ans: B Feedback: The patient must be able to perform transfers and to use mobility aids safely. Each of the other listed goals is unrealistic for the patient who has undergone recent hip replacement.

A patient has just begun been receiving skeletal traction and the nurse is aware that muscles in the patient's affected limb are spastic. How does this change in muscle tone affect the patient's traction prescription? A) Traction must temporarily be aligned in a slightly different direction. B) Extra weight is needed initially to keep the limb in proper alignment. C) A lighter weight should be initially used. D) Weight will temporarily alternate between heavier and lighter weights.

Ans: B Feedback: The traction weights applied initially must overcome the shortening spasms of the affected muscles. As the muscles relax, the traction weight is reduced to prevent fracture dislocation and to promote healing. Weights never alternate between heavy and light.

A nurse is caring for a patient receiving skeletal traction. Due to the patient's severe limits on mobility, the nurse has identified a risk for atelectasis or pneumonia. What intervention should the nurse provide in order to prevent these complications? A) Perform chest physiotherapy once per shift and as needed. B) Teach the patient to perform deep breathing and coughing exercises. C) Administer prophylactic antibiotics as ordered. D) Administer nebulized bronchodilators and corticosteroids as ordered.

Ans: B Feedback: To prevent these complications, the nurse should educate the patient about performing deep-breathing and coughing exercises to aid in fully expanding the lungs and clearing pulmonary secretions. Antibiotics, bronchodilators, and steroids are not used on a preventative basis and chest physiotherapy is unnecessary and implausible for a patient in traction.

A patient with a total hip replacement is progressing well and expects to be discharged tomorrow. On returning to bed after ambulating, he complains of a new onset of pain at the surgical site. What is the nurse's best action? A) Administer pain medication as ordered. B) Assess the surgical site and the affected extremity. C) Reassure the patient that pain is a direct result of increased activity. D) Assess the patient for signs and symptoms of systemic infection.

Ans: B Feedback: Worsening pain after a total hip replacement may indicate dislocation of the prosthesis. Assessment of pain should include evaluation of the wound and the affected extremity. Assuming he's anxious about discharge and administering pain medication do not address the cause of the pain. Sudden severe pain is not considered normal after hip replacement. Sudden pain is rarely indicative of a systemic infection.

21. A 69-year-old patient is brought to the ED by ambulance because a family member found him lying on the floor disoriented and lethargic. The physician suspects bacterial meningitis and admits the patient to the ICU. The nurse knows that risk factors for an unfavorable outcome include what? Select all that apply. A) Blood pressure greater than 140/90 mm Hg B) Heart rate greater than 120 bpm C) Older age D) Low Glasgow Coma Scale E) Lack of previous immunizations

Ans: B, C, D Risks for an unfavorable outcome of meningitis include older age, a heart rate greater than 120 beats/minute, low Glasgow Coma Scale score, cranial nerve palsies, and a positive Gram stain 1 hour after presentation to the hospital. A BP greater than 140/90 mm Hg is indicative of hypertension, but is not necessarily related to poor outcomes related to meningitis. Immunizations are not normally relevant to the course of the disease.

A nurse in a busy emergency department provides care for many patients who present with contusions, strains, or sprains. Treatment modalities that are common to all of these musculoskeletal injuries include which of the following? Select all that apply. A) Massage B) Applying ice C) Compression dressings D) Resting the affected extremity E) Corticosteroids F) Elevating the injured limb

Ans: B, C, D, F Feedback: Treatment of contusions, strains, and sprains consists of resting and elevating the affected part, applying cold, and using a compression bandage. Massage and corticosteroids are not used to treat these injuries.

A patient with Parkinsons disease is undergoing a swallowing assessment because she has recently developed adventitious lung sounds. The patients nutritional needs should be met by what method? A. Total parenteral nutrition (TPN) B. Provision of a low-residue diet C. Semisolid food with thick liquids D. Minced foods and a fluid restriction

Ans: C Feedback: A semisolid diet with thick liquids is easier for a patient with swallowing difficulties to consume than is a solid diet. Low-residue foods and fluid restriction are unnecessary and counterproductive to the patients nutritional status. The patients status does not warrant TPN.

A patient with a brain tumor has begun to exhibit signs of cachexia. What subsequent assessment should the nurse prioritize? A. Assessment of peripheral nervous function B. Assessment of cranial nerve function C. Assessment of nutritional status D. Assessment of respiratory status

Ans: C Feedback: Cachexia is a wasting syndrome of weight loss, muscle atrophy, fatigue, weakness, and significant loss of appetite. Consequently, nutritional assessment is paramount.

A patient, brought to the clinic by his wife and son, is diagnosed with Huntington disease. When providing anticipatory guidance, the nurse should address the future possibility of what effect of Huntington disease? A. Metastasis B. Risk for stroke C. Emotional and personality changes D. Pathologic bone fractures

Ans: C Feedback: Huntington disease causes profound changes to personality and behavior. It is a nonmalignant disease and stroke is not a central risk. The disease is not associated with pathologic bone fractures.

The nurse responds to the call light of a patient who has had a cervical diskectomy earlier in the day. The patient states that she is having severe pain that had a sudden onset. What is the nurses most appropriate action? A. Palpate the surgical site. B. Remove the dressing to assess the surgical site. C. Call the surgeon to report the patients pain. D. Administer a dose of an NSAID.

Ans: C Feedback: If the patient experiences a sudden increase in pain, extrusion of the graft may have occurred, requiring reoperation. A sudden increase in pain should be promptly reported to the surgeon. Administration of an NSAID would be an insufficient response and the dressing should not be removed without an order. Palpation could cause further damage.

A patient with amyotrophic lateral sclerosis (ALS) is being visited by the home health nurse who is creating a care plan. What nursing diagnosis is most likely for a patient with this condition? A. Chronic confusion B. Impaired urinary elimination C. Impaired verbal communication D. Bowel incontinence

Ans: C Feedback: Impaired communication is an appropriate nursing diagnosis; the voice in patients with ALS assumes a nasal sound and articulation becomes so disrupted that speech is unintelligible. Intellectual function is marginally impaired in patients with late ALS. Usually, the anal and bladder sphincters are intact because the spinal nerves that control muscles of the rectum and urinary bladder are not affected.

A patient with Parkinsons disease is experiencing episodes of constipation that are becoming increasingly frequent and severe. The patient states that he has been achieving relief for the past few weeks by using OTC laxatives. How should the nurse respond? A. Its important to drink plenty of fluids while youre taking laxatives. B. Make sure that you supplement your laxatives with a nutritious diet. C. Lets explore other options, because laxatives can have side effects and create dependency. D. You should ideally be using herbal remedies rather than medications to promote bowel function.

Ans: C Feedback: Laxatives should be avoided in patients with Parkinsons disease due to the risk of adverse effects and dependence. Herbal bowel remedies are not necessarily less risky.

The nurse is writing a care plan for a patient with brain metastases. The nurse decides that an appropriate nursing diagnosis is anxiety related to lack of control over the health circumstances. In establishing this plan of care for the patient, the nurse should include what intervention? A. The patient will receive antianxiety medications every 4 hours. B. The patients family will be instructed on planning the patients care. C. The patient will be encouraged to verbalize concerns related to the disease and its treatment. D. The patient will begin intensive therapy with the goal of distraction.

Ans: C Feedback: Patients need the opportunity to exercise some control over their situation. A sense of mastery can be gained as they learn to understand the disease and its treatment and how to deal with their feelings. Distraction and administering medications will not allow the patient to gain control over anxiety. Delegating planning to the family will not help the patient gain a sense of control and autonomy.

The nurse is caring for a patient newly diagnosed with a primary brain tumor. The patient asks the nurse where his tumor came from. What would be the nurses best response? A. Your tumor originated from somewhere outside the CNS. B. Your tumor likely started out in one of your glands. C. Your tumor originated from cells within your brain itself. D. Your tumor is from nerve tissue somewhere in your body.

Ans: C Feedback: Primary brain tumors originate from cells and structures within the brain. Secondary brain tumors are metastatic tumors that originate somewhere else in the body. The scenario does not indicate that the patients tumor is a pituitary tumor or a neuroma.

A patient who has been experiencing numerous episodes of unexplained headaches and vomiting has subsequently been referred for testing to rule out a brain tumor. What characteristic of the patients vomiting is most consistent with a brain tumor? A. The patients vomiting is accompanied by epistaxis. B. The patients vomiting does not relieve his nausea. C. The patients vomiting is unrelated to food intake. D. The patients emesis is blood-tinged.

Ans: C Feedback: Vomiting is often unrelated to food intake if caused by a brain tumor. The presence or absence of blood is not related to the possible etiology and vomiting may or may not relieve the patients nausea.

23. A patient is being admitted to the neurologic ICU with suspected herpes simplex virus encephalitis. What nursing action best addresses the patient's complaints of headache? A) Initiating a patient-controlled analgesia (PCA) of morphine sulfate B) Administering hydromorphone (Dilaudid) IV as needed C) Dimming the lights and reducing stimulation D) Distracting the patient with activity

Ans: C Comfort measures to reduce headache include dimming the lights, limiting noise and visitors, grouping nursing interventions, and administering analgesic agents. Opioid analgesic medications may mask neurologic symptoms; therefore, they are used cautiously. Non-opioid analgesics may be preferred. Distraction is unlikely to be effective, and may exacerbate the patient's pain.

20. Following a traumatic brain injury, a patient has been in a coma for several days. Which of the following statements is true of this patients current LOC? A) The patient occasionally makes incomprehensible sounds. B) The patients current LOC will likely become a permanent state. C) The patient may occasionally make nonpurposeful movements. D) The patient is incapable of spontaneous respirations.

Ans: C Feedback: Coma is a clinical state of unarousable unresponsiveness in which no purposeful responses to internal or external stimuli occur, although nonpurposeful responses to painful stimuli and brain stem reflexes may be present. Verbal sounds, however, are atypical. Ventilator support may or may not be necessary. Comas are not permanent states.

11. A patient with increased ICP has a ventriculostomy for monitoring ICP. The nurses most recent assessment reveals that the patient is now exhibiting nuchal rigidity and photophobia. The nurse would be correct in suspecting the presence of what complication? A) Encephalitis B) CSF leak C) Meningitis D) Catheter occlusion

Ans: C Feedback: Complications of a ventriculostomy include ventricular infectious meningitis and problems with the monitoring system. Nuchal rigidity and photophobia are clinical manifestations of meningitis, but are not suggestive of encephalitis, a CSF leak, or an occluded catheter.

28. A patient has experienced a seizure in which she became rigid and then experienced alternating muscle relaxation and contraction. What type of seizure does the nurse recognize? A) Unclassified seizure B) Absence seizure C) Generalized seizure D) Focal seizure

Ans: C Feedback: Generalized seizures often involve both hemispheres of the brain, causing both sides of the body to react. Intense rigidity of the entire body may occur, followed by alternating muscle relaxation and contraction (generalized tonicclonic contraction). This pattern of rigidity does not occur in patients who experience unclassified, absence, or focal seizures.

6. A patient who has been on long-term phenytoin (Dilantin) therapy is admitted to the unit. In light of the adverse of effects of this medication, the nurse should prioritize which of the following in the patients plan of care? A) Monitoring of pulse oximetry B) Administration of a low-protein diet C) Administration of thorough oral hygiene D) Fluid restriction as ordered

Ans: C Feedback: Gingival hyperplasia (swollen and tender gums) can be associated with long-term phenytoin (Dilantin) use. Thorough oral hygiene should be provided consistently and encouraged after discharge. Fluid and protein restriction are contraindicated and there is no particular need for constant oxygen saturation monitoring.

25. The nurse is caring for a patient with a brain tumor. What drug would the nurse expect to be ordered to reduce the edema surrounding the tumor? A) Solumedrol B) Dextromethorphan C) Dexamethasone D) Furosemide

Ans: C Feedback: If a brain tumor is the cause of the increased ICP, corticosteroids (e.g., dexamethasone) help reduce the edema surrounding the tumor. Solumedrol, a steroid, and furosemide, a loop diuretic, are not the drugs of choice in this instance. Dextromethorphan is used in cough medicines.

3. The nurse is caring for a patient in the ICU who has a brain stem herniation and who is exhibiting an altered level of consciousness. Monitoring reveals that the patients mean arterial pressure (MAP) is 60 mm Hg with an intracranial pressure (ICP) reading of 55 mm Hg. What is the nurses most appropriate action? A) Position the patient in the high Fowlers position as tolerated. B) Administer osmotic diuretics as ordered. C) Participate in interventions to increase cerebral perfusion pressure. D) Prepare the patient for craniotomy.

Ans: C Feedback: The cerebral perfusion pressure (CPP) is 55 mm Hg, which is considered low. The normal CPP is 70 to 100 mm Hg. Patients with a CPP of less than 50 mm Hg experience irreversible neurologic damage. As a result, interventions are necessary. A craniotomy is not directly indicated. Diuretics and increased height of bed would exacerbate the patients condition.

1. A patient is being admitted to the neurologic ICU following an acute head injury that has resulted in cerebral edema. When planning this patients care, the nurse would expect to administer what priority medication? A) Hydrochlorothiazide (HydroDIURIL) B) Furosemide (Lasix) C) Mannitol (Osmitrol) D) Spirolactone (Aldactone)

Ans: C Feedback: The osmotic diuretic mannitol is given to dehydrate the brain tissue and reduce cerebral edema. This drug acts by reducing the volume of brain and extracellular fluid. Spirolactone, furosemide, and hydrochlorothiazide are diuretics that are not typically used in the treatment of increased ICP resulting from cerebral edema.

32. A patient is recovering from intracranial surgery that was performed using the transsphenoidal approach. The nurse should be aware that the patient may have required surgery on what neurologic structure? A) Cerebellum B) Hypothalamus C) Pituitary gland D) Pineal gland

Ans: C Feedback: The transsphenoidal approach (through the mouth and nasal sinuses) is often used to gain access to the pituitary gland. This surgical approach would not allow for access to the pineal gland, cerebellum, or hypothalamus.

35. A neurologic nurse is reviewing seizures with a group of staff nurses. How should this nurse best describe the cause of a seizure? A) Sudden electrolyte changes throughout the brain B) A dysrhythmia in the peripheral nervous system C) A dysrhythmia in the nerve cells in one section of the brain D) Sudden disruptions in the blood flow throughout the brain

Ans: C Feedback: The underlying cause of a seizure is an electrical disturbance (dysrhythmia) in the nerve cells in one section of the brain, these cells emit abnormal, recurring, uncontrolled electrical discharges. Seizures are not caused by changes in blood flow or electrolytes.

7. A nurse is admitting a patient with a severe migraine headache and a history of acute coronary syndrome. What migraine medication would the nurse question for this patient? A) Rizatriptan (Maxalt) B) Naratriptan (Amerge) C) Sumatriptan succinate (Imitrex) D) Zolmitriptan (Zomig)

Ans: C Feedback: Triptans can cause chest pain and are contraindicated in patients with ischemic heart disease. Maxalt, Amerge, and Zomig are triptans used in routine clinical use for the treatment of migraine headaches.

39. A nurse is collaborating with the interdisciplinary team to help manage a patients recurrent headaches. What aspect of the patients health history should the nurse identify as a potential contributor to the patients headaches? A) The patient leads a sedentary lifestyle. B) The patient takes vitamin D and calcium supplements. C) The patient takes vasodilators for the treatment of angina. D) The patient has a pattern of weight loss followed by weight gain.

Ans: C Feedback: Vasodilators are known to contribute to headaches. Weight fluctuations, sedentary lifestyle, and vitamin supplements are not known to have this effect.

4. A patient with Guillain-Barré syndrome has experienced a sharp decline in vital capacity. What is the nurse's most appropriate action? A) Administer bronchodilators as ordered. B) Remind the patient of the importance of deep breathing and coughing exercises. C) Prepare to assist with intubation. D) Administer supplementary oxygen by nasal cannula.

Ans: C For the patient with Guillain-Barré syndrome, mechanical ventilation is required if the vital capacity falls, making spontaneous breathing impossible and tissue oxygenation inadequate. Each of the other listed actions is likely insufficient to meet the patient's oxygenation needs.

31. The nurse is discharging a patient home after surgery for trigeminal neuralgia. What advice should the nurse provide to this patient in order to reduce the risk of injury? A) Avoid watching television or using a computer for more than 1 hour at a time. B) Use OTC antibiotic eye drops for at least 14 days. C) Avoid rubbing the eye on the affected side of the face. D) Rinse the eye on the affected side with normal saline daily for 1 week.

Ans: C If the surgery results in sensory deficits to the affected side of the face, the patient is instructed not to rub the eye because the pain of a resulting injury will not be detected. There is no need to limit TV viewing or to rinse the eye daily. Antibiotics may or may not be prescribed, and these would not reduce the risk of injury.

19. A 33-year-old patient presents at the clinic with complaints of weakness, incoordination, dizziness, and loss of balance. The patient is hospitalized and diagnosed with MS. What sign or symptom, revealed during the initial assessment, is typical of MS? A) Diplopia, history of increased fatigue, and decreased or absent deep tendon reflexes B) Flexor spasm, clonus, and negative Babinski's reflex C) Blurred vision, intention tremor, and urinary hesitancy D) Hyperactive abdominal reflexes and history of unsteady gait and episodic paresthesia in both legs

Ans: C Optic neuritis, leading to blurred vision, is a common early sign of MS, as is intention tremor (tremor when performing an activity). Nerve damage can cause urinary hesitancy. In MS, deep tendon reflexes are increased or hyperactive. A positive Babinski's reflex is found in MS. Abdominal reflexes are absent with MS.

39. A 48-year-old patient has been diagnosed with trigeminal neuralgia following recent episodes of unilateral face pain. The nurse should recognize what implication of this diagnosis? A) The patient will likely require lifelong treatment with anticholinergic medications. B) The patient has a disproportionate risk of developing myasthenia gravis later in life. C) The patient needs to be assessed for MS. D) The disease is self-limiting and the patient will achieve pain relief over time.

Ans: C Patients that develop trigeminal neuralgia before age 50 should be evaluated for the coexistent of MS because trigeminal neuralgia occurs in approximately 5% of patients with MS. Treatment does not include anticholinergics and the disease is not self-limiting. Trigeminal neuralgia is not associated with an increased risk of myasthenia gravis.

17. A nurse is planning the care of a 28-year-old woman hospitalized with a diagnosis of myasthenia gravis. What approach would be most appropriate for the care and scheduling of diagnostic procedures for this patient? A) All at one time, to provide a longer rest period B) Before meals, to stimulate her appetite C) In the morning, with frequent rest periods D) Before bedtime, to promote rest

Ans: C Procedures should be spaced to allow for rest in between. Procedures should be avoided before meals, or the patient may be too exhausted to eat. Procedures should be avoided near bedtime if possible.

27. The critical care nurse is admitting a patient in myasthenic crisis to the ICU. The nurse should prioritize what nursing action in the immediate care of this patient? A) Suctioning secretions B) Facilitating ABG analysis C) Providing ventilatory assistance D) Administering tube feedings

Ans: C Providing ventilatory assistance takes precedence in the immediate management of the patient with myasthenic crisis. It may be necessary to suction secretions and/or provide tube feedings, but they are not the priority for this patient. ABG analysis will be done, but this is not the priority.

22. The critical care nurse is caring for 25-year-old man admitted to the ICU with a brain abscess. What is a priority nursing responsibility in the care of this patient? A) Maintaining the patient's functional independence B) Providing health education C) Monitoring neurologic status closely D) Promoting mobility

Ans: C Vigilant neurologic monitoring is a key aspect of caring for a patient who has a brain abscess. This supersedes education, ADLs, and mobility, even though these are all valid and important aspects of nursing care.

40. A nurse who provides care on a burn unit is preparing to apply a patient's ordered topical antibiotic ointment. What action should the nurse perform when administering this medication? A) Apply the new ointment without disturbing the existing layer of ointment. B) Apply the ointment using a sterile tongue depressor. C) Apply a layer of ointment approximately 1/16 inch thick. D) Gently irrigate the wound bed after applying the antibiotic ointment.

Ans: C Feedback: After removing the old ointment from the wound bed, the nurse should apply a layer of ointment 1/16-inch thick using clean gloves. The wound would not be irrigated after application of new ointment.

14. A triage nurse in the emergency department (ED) receives a phone call from a frantic father who saw his 4-year-old child tip a pot of boiling water onto her chest. The father has called an ambulance. What would the nurse in the ED receiving the call instruct the father to do? A) Cover the burn with ice and secure with a towel. B) Apply butter to the area that is burned. C) Immerse the child in a cool bath. D) Avoid touching the burned area under any circumstances.

Ans: C Feedback: After the flames or heat source have been removed or extinguished, the burned area and adherent clothing are soaked with cool water briefly to cool the wound and halt the burning process. Cool water is the best first-aid measure. Ice and butter are contraindicated. Appropriate first aid necessitates touching the burn.

A school nurse is assessing a student who was kicked in the shin during a soccer game. The area of the injury has become swollen and discolored. The triage nurse recognizes that the patient has likely sustained what? A) Sprain B) Strain C) Contusion D) Dislocation

Ans: C Feedback: A contusion is a soft-tissue injury that results in bleeding into soft tissues, creating a hematoma and ecchymosis. A sprain is an injury to ligaments caused by wrenching or twisting. A strain is a "muscle pull" from overuse, overstretching, or excessive stress. A dislocation is a condition in which the articular surfaces of the bones forming a joint are no longer in anatomic contact. Because the injury is not at the site of a joint, the patient has not experienced a sprain, strain, or dislocation.

A nurse is planning the care of a patient with osteomyelitis that resulted from a diabetic foot ulcer. The patient requires a transmetatarsal amputation. When planning the patient's postoperative care, which of the following nursing diagnoses should the nurse most likely include in the plan of care? A) Ineffective Thermoregulation B) Risk-Prone Health Behavior C) Disturbed Body Image D) Deficient Diversion Activity

Ans: C Feedback: Amputations present a serious threat to any patient's body image. None of the other listed diagnoses is specifically associated with amputation.

A patient has presented to the emergency department with an injury to the wrist. The patient is diagnosed with a third-degree strain. Why would the physician order an x-ray of the wrist? A) Nerve damage is associated with third-degree strains. B) Compartment syndrome is associated with third-degree strains. C) Avulsion fractures are associated with third-degree strains. D) Greenstick fractures are associated with third-degree strains.

Ans: C Feedback: An x-ray should be obtained to rule out bone injury, because an avulsion fracture (in which a bone fragment is pulled away from the bone by a tendon) may be associated with a third-degree strain. Nerve damage, compartment syndrome, and greenstick fractures are not associated with third-degree strains.

A nurse is reviewing the pathophysiology that may underlie a patient's decreased bone density. What hormone should the nurse identify as inhibiting bone resorption and promoting bone formation? A) Estrogen B) Parathyroid hormone (PTH) C) Calcitonin D) Progesterone

Ans: C Feedback: Calcitonin inhibits bone resorption and promotes bone formation, estrogen inhibits bone breakdown, and parathyroid increases bone resorption. Estrogen, which inhibits bone breakdown, decreases with aging. Parathyroid hormone (PTH) increases with aging, increasing bone turnover and resorption. Progesterone is the major naturally occurring human progestogen and plays a role in the female menstrual cycle.

A nurse is discussing conservative management of tendonitis with a patient. Which of the following may be an effective approach to managing tendonitis? A) Weight reduction B) Use of oral opioid analgesics C) Intermittent application of ice and heat D) Passive range of motion exercises

Ans: C Feedback: Conservative management of tendonitis includes rest of the extremity, intermittent ice and heat to the joint, and NSAIDs. Weight reduction may prevent future injuries but will not relieve existing tendonitis. Range-of-motion exercises may exacerbate pain. Opioids would not be considered a conservative treatment measure.

A nurse is caring for a patient who has suffered a hip fracture and who will require an extended hospital stay. The nurse should ensure that the patient does which of the following in order to prevent common complications associated with a hip fracture? A) Avoid requesting analgesia unless pain becomes unbearable. B) Use supplementary oxygen when transferring or mobilizing. C) Increase fluid intake and perform prescribed foot exercises. D) Remain on bed rest for 14 days or until instructed by the orthopedic surgeon.

Ans: C Feedback: Deep vein thrombosis (DVT) is among the most common complications related to a hip fracture. To prevent DVT, the nurse encourages intake of fluids and ankle and foot exercises. The patient should not be told to endure pain; a proactive approach to pain control should be adopted. While respiratory complications commonly include atelectasis and pneumonia, the use of deep-breathing exercises, changes in position at least every 2 hours, and the use of incentive spirometry help prevent the respiratory complications more than using supplementary oxygen. Bed rest may be indicated in the short term, but is not normally required for 14 days.

A nurse is caring for a patient with a bone tumor. The nurse is providing education to help the patient reduce the risk for pathologic fractures. What should the nurse teach the patient? A) Strive to achieve maximum weight-bearing capabilities. B) Gradually strengthen the affected muscles through weight training. C) Support the affected extremity with external supports such as splints. D) Limit reliance on assistive devices in order to build strength.

Ans: C Feedback: During nursing care, the affected extremities must be supported and handled gently. External supports (splints) may be used for additional protection. Prescribed weight-bearing restrictions must be followed. Assistive devices should be used to strengthen the unaffected extremities.

A nurse is performing a shift assessment on an elderly patient who is recovering after surgery for a hip fracture. The nurse notes that the patient is complaining of chest pain, has an increased heart rate, and increased respiratory rate. The nurse further notes that the patient is febrile and hypoxic, coughing, and producing large amounts of thick, white sputum. The nurse recognizes that this is a medical emergency and calls for assistance, recognizing that this patient is likely demonstrating symptoms of what complication? A) Avascular necrosis of bone B) Compartment syndrome C) Fat embolism syndrome D) Complex regional pain syndrome

Ans: C Feedback: Fat embolism syndrome occurs most frequently in young adults and elderly patients who experience fractures of the proximal femur (i.e., hip fracture). Presenting features of fat embolism syndrome include hypoxia, tachypnea, tachycardia, and pyrexia. The respiratory distress response includes tachypnea, dyspnea, wheezes, precordial chest pain, cough, large amounts of thick, white sputum, and tachycardia. Avascular necrosis (AVN) occurs when the bone loses its blood supply and dies. This does not cause coughing. Complex regional pain syndrome does not have cardiopulmonary involvement.

A patient has come to the orthopedic clinic for a follow-up appointment 6 weeks after fracturing his ankle. Diagnostic imaging reveals that bone union is not taking place. What factor may have contributed to this complication? A) Inadequate vitamin D intake B) Bleeding at the injury site C) Inadequate immobilization D) Venous thromboembolism (VTE)

Ans: C Feedback: Inadequate fracture immobilization can delay or prevent union. A short-term vitamin D deficiency would not likely prevent bone union. VTE is a serious complication but would not be a cause of nonunion. Similarly, bleeding would not likely delay union.

A 20 year-old is brought in by ambulance to the emergency department after being involved in a motorcycle accident. The patient has an open fracture of his tibia. The wound is highly contaminated and there is extensive soft-tissue damage. How would this patient's fracture likely be graded? A) Grade I B) Grade II C) Grade III D) Grade IV

Ans: C Feedback: Open fractures are graded according to the following criteria. Grade I is a clean wound less than 1 cm long. Grade II is a larger wound without extensive soft-tissue damage. Grade III is highly contaminated, has extensive soft-tissue damage, and is the most severe. There is no grade IV fracture.

Six weeks after an above-the-knee amputation (AKA), a patient returns to the outpatient office for a routine postoperative checkup. During the nurse's assessment, the patient reports symptoms of phantom pain. What should the nurse tell the patient to do to reduce the discomfort of the phantom pain? A) Apply intermittent hot compresses to the area of the amputation. B) Avoid activity until the pain subsides. C) Take opioid analgesics as ordered. D) Elevate the level of the amputation site.

Ans: C Feedback: Opioid analgesics may be effective in relieving phantom pain. Heat, immobility, and elevation are not noted to relieve this form of pain.

A nurse is caring for a patient with Paget's disease and is reviewing the patient's most recent laboratory values. Which of the following values is most characteristic of Paget's disease? A) An elevated level of parathyroid hormone and low calcitonin levels B) A low serum alkaline phosphatase level and a low serum calcium level C) An elevated serum alkaline phosphatase level and a normal serum calcium level D) An elevated calcitonin level and low levels of parathyroid hormone

Ans: C Feedback: Patients with Paget's disease have normal blood calcium levels. Elevated serum alkaline phosphatase concentration and urinary hydroxyproline excretion reflect the increased osteoblastic activity associated with this condition. Alterations in PTH and calcitonin levels are atypical.

A patient presents at a clinic complaining of back pain that goes all the way down the back of the leg to the foot. The nurse should document the presence of what type of pain? A) Bursitis B) Radiculopathy C) Sciatica D) Tendonitis

Ans: C Feedback: Sciatica nerve pain travels down the back of the thigh to the foot of the affected leg. Bursitis is inflammation of a fluid-filled sac in a joint. Radiculopathy is disease of a nerve root. Tendonitis is inflammation of muscle tendons.

A nursing educator is reviewing the risk factors for osteoporosis with a group of recent graduates. What risk factor of the following should the educator describe? A) Recurrent infections and prolonged use of NSAIDs B) High alcohol intake and low body mass index C) Small frame, female gender, and Caucasian ethnicity D) Male gender, diabetes, and high protein intake

Ans: C Feedback: Small-framed, nonobese Caucasian women are at greatest risk for osteoporosis. Diabetes, high protein intake, alcohol use, and infections are not among the most salient risk factors for osteoporosis.

A young patient is being treated for a femoral fracture suffered in a snowboarding accident. The nurse's most recent assessment reveals that the patient is uncharacteristically confused. What diagnostic test should be performed on this patient? A) Electrolyte assessment B) Electrocardiogram C) Arterial blood gases D) Abdominal ultrasound

Ans: C Feedback: Subtle personality changes, restlessness, irritability, or confusion in a patient who has sustained a fracture are indications for immediate arterial blood gas studies due to the possibility of fat embolism syndrome. This assessment finding does not indicate an immediate need for electrolyte levels, an ECG, or abdominal ultrasound.

A patient is being treated for a fractured hip and the nurse is aware of the need to implement interventions to prevent muscle wasting and other complications of immobility. What intervention best addresses the patient's need for exercise? A) Performing gentle leg lifts with both legs B) Performing massage to stimulate circulation C) Encouraging frequent use of the overbed trapeze D) Encouraging the patient to log roll side to side once per hour

Ans: C Feedback: The patient is encouraged to exercise as much as possible by means of the overbed trapeze. This device helps strengthen the arms and shoulders in preparation for protected ambulation. Independent logrolling may result in injury due to the location of the fracture. Leg lifts would be contraindicated for the same reason. Massage by the nurse is not a substitute for exercise.

A nurse is providing care for a patient who has osteomalacia. What major goal will guide the choice of medical and nursing interventions? A) Maintenance of skin integrity B) Prevention of bone metastasis C) Maintenance of adequate levels of activated vitamin D D) Maintenance of adequate parathyroid hormone function

Ans: C Feedback: The primary defect in osteomalacia is a deficiency of activated vitamin D, which promotes calcium absorption from the gastrointestinal tract and facilitates mineralization of bone. Interventions are aimed at resolving the processes underlying this deficiency. Maintenance of skin integrity is important, but is not the primary goal in care. Osteomalacia is not a malignant process. Overproduction (not underproduction) of PTH can cause the disease.

The nurse is caring for a patient who underwent a total hip replacement yesterday. What should the nurse do to prevent dislocation of the new prosthesis? A) Keep the affected leg in a position of adduction. B) Have the patient reposition himself independently. C) Protect the affected leg from internal rotation. D) Keep the hip flexed by placing pillows under the patient's knee.

Ans: C Feedback: Abduction of the hip helps to prevent dislocation of a new hip joint. Rotation and adduction should be avoided. While the hip may be flexed slightly, it shouldn't exceed 90 degrees and maintenance of flexion isn't necessary. The patient may not be capable of safe independent repositioning at this early stage of recovery.

An elderly patient's hip joint is immobilized prior to surgery to correct a femoral head fracture. What is the nurse's priority assessment? A) The presence of leg shortening B) The patient's complaints of pain C) Signs of neurovascular compromise D) The presence of internal or external rotation

Ans: C Feedback: Because impaired circulation can cause permanent damage, neurovascular assessment of the affected leg is always a priority assessment. Leg shortening and internal or external rotation are common findings with a fractured hip. Pain, especially on movement, is also common after a hip fracture.

A patient is admitted to the unit in traction for a fractured proximal femur and requires traction prior to surgery. What is the most appropriate type of traction to apply to a fractured proximal femur? A) Russell's traction B) Dunlop's traction C) Buck's extension traction D) Cervical head halter

Ans: C Feedback: Buck's extension is used for fractures of the proximal femur. Russell's traction is used for lower leg fractures. Dunlop's traction is applied to the upper extremity for supracondylar fractures of the elbow and humerus. Cervical head halters are used to stabilize the neck.

A patient is complaining of pain in her casted leg. The nurse has administered analgesics and elevated the limb. Thirty minutes after administering the analgesics, the patient states the pain is unrelieved. The nurse should identify the warning signs of what complication? A) Subcutaneous emphysema B) Skin breakdown C) Compartment syndrome D) Disuse syndrome

Ans: C Feedback: Compartment syndrome may manifest as unrelenting, uncontrollable pain. This presentation of pain is not suggestive of disuse syndrome or skin breakdown. Subcutaneous emphysema is not a complication of casting.

A 91-year-old patient is slated for orthopedic surgery and the nurse is integrated gerontologic considerations into the patient's plan of care. What intervention is most justified in the care of this patient? A) Administration of prophylactic antibiotics B) Total parenteral nutrition (TPN) C) Use of a pressure-relieving mattress D) Use of a Foley catheter until discharge

Ans: C Feedback: Older adults have a heightened risk of skin breakdown; use of a pressure-reducing mattress addresses this risk. Older adults do not necessarily need TPN and the Foley catheter should be discontinued as soon as possible to prevent urinary tract infections. Prophylactic antibiotics are not a standard infection prevention measure.

A nurse is caring for a patient who has a leg cast. The nurse observes that the patient uses a pencil to scratch the skin under the edge of the cast. How should the nurse respond to this observation? A) Allow the patient to continue to scratch inside the cast with a pencil but encourage him to be cautious. B) Give the patient a sterile tongue depressor to use for scratching instead of the pencil. C) Encourage the patient to avoid scratching, and obtain an order for an antihistamine if severe itching persists. D) Obtain an order for a sedative, such as lorazepam (Ativan), to prevent the patient from scratching.

Ans: C Feedback: Scratching should be discouraged because of the risk for skin breakdown or damage to the cast. Most patients can be discouraged from scratching if given a mild antihistamine, such as diphenhydramine, to relieve itching. Benzodiazepines would not be given for this purpose.

The nurse has identified the diagnosis of Risk for Impaired Tissue Perfusion Related to Deep Vein Thrombosis in the care of a patient receiving skeletal traction. What nursing intervention best addresses this risk? A) Encourage independence with ADLs whenever possible. B) Monitor the patient's nutritional status closely. C) Teach the patient to perform ankle and foot exercises within the limitations of traction. D) Administer clopidogrel (Plavix) as ordered.

Ans: C Feedback: The nurse educates the patient how to perform ankle and foot exercises within the limits of the traction therapy every 1 to 2 hours when awake to prevent DVT. Nutrition is important, but does not directly prevent DVT. Similarly, independence with ADLs should be promoted, but this does not confer significant prevention of DVT, which often affects the lower limbs. Plavix is not normally used for DVT prophylaxis.

A nurse is caring for a patient in skeletal traction. In order to prevent bony fragments from moving against one another, the nurse should caution the patient against which of the following actions? A) Shifting one's weight in bed B) Bearing down while having a bowel movement C) Turning from side to side D) Coughing without splinting

Ans: C Feedback: To prevent bony fragments from moving against one another, the patient should not turn from side to side; however, the patient may shift position slightly with assistance. Bearing down and coughing do not pose a threat to bone union.

The nurse educator is discussing neoplasms with a group of recent graduates. The educator explains that the effects of neoplasms are caused by the compression and infiltration of normal tissue. The physiologic changes that result can cause what pathophysiologic events? Select all that apply. A. Intracranial hemorrhage B. Infection of cerebrospinal fluid C. Increased ICP D. Focal neurologic signs E. Altered pituitary function

Ans: C, D, E Feedback: The effects of neoplasms are caused by the compression and infiltration of tissue. A variety of physiologic changes result, causing any or all of the following pathophysiologic events: increased ICP and cerebral edema, seizure activity and focal neurologic signs, hydrocephalus, and altered pituitary function.

A patient is admitted to the orthopedic unit with a fractured femur after a motorcycle accident. The patient has been placed in traction until his femur can be rodded in surgery. For what early complications should the nurse monitor this patient? Select all that apply. A) Systemic infection B) Complex regional pain syndrome C) Deep vein thrombosis D) Compartment syndrome E) Fat embolism

Ans: C, D, E Feedback: Early complications include shock, fat embolism, compartment syndrome, and venous thromboemboli (deep vein thrombosis [DVT], pulmonary embolism [PE]). Infection and CRPS are later complications of fractures.

While assessing the patient at the beginning of the shift, the nurse inspects a surgical dressing covering the operative site after the patients cervical diskectomy. The nurse notes that the drainage is 75% saturated with serosanguineous discharge. What is the nurses most appropriate action? A. Page the physician and report this sign of infection. B. Reinforce the dressing and reassess in 1 to 2 hours. C. Reposition the patient to prevent further hemorrhage. D. Inform the surgeon of the possibility of a dural leak.

Ans: D Feedback: After a cervical diskectomy, the nurse will monitor the operative site and dressing covering this site. Serosanguineous drainage may indicate a dural leak. This constitutes a risk for meningitis, but is not a direct sign of infection. This should be reported to the surgeon, not just reinforced and observed.

A patient has just been diagnosed with Parkinsons disease and the nurse is planning the patients subsequent care for the home setting. What nursing diagnosis should the nurse address when educating the patients family? A. Risk for infection B. Impaired spontaneous ventilation C. Unilateral neglect D. Risk for injury

Ans: D Feedback: Individuals with Parkinsons disease face a significant risk for injury related to the effects of dyskinesia. Unilateral neglect is not characteristic of the disease, which affects both sides of the body. Parkinsons disease does not directly constitute a risk for infection or impaired respiration.

A patient, diagnosed with cancer of the lung, has just been told he has metastases to the brain. What change in health status would the nurse attribute to the patients metastatic brain disease? A. Chronic pain B. Respiratory distress C. Fixed pupils D. Personality changes

Ans: D Feedback: Neurologic signs and symptoms include headache, gait disturbances, visual impairment, personality changes, altered mentation (memory loss and confusion), focal weakness, paralysis, aphasia, and seizures. Pain, respiratory distress, and fixed pupils are not among the more common neurologic signs and symptoms of metastatic brain disease.

The nurse is caring for a patient with Huntington disease who has been admitted to the hospital for treatment of malnutrition. What independent nursing action should be implemented in the patients plan of care? A. Firmly redirect the patients head when feeding. B. Administer phenothiazines after each meal as ordered. C. Encourage the patient to keep his or her feeding area clean. D. Apply deep, gentle pressure around the patients mouth to aid swallowing.

Ans: D Feedback: Nursing interventions for a patient who has inadequate nutritional intake should include the following: Apply deep gentle pressure around the patients mouth to assist with swallowing, and administer phenothiazines prior to the patients meal as ordered. The nurse should disregard the mess of the feeding area and treat the person with dignity. Stiffness and turning away by the patient during feeding are uncontrollable choreiform movements and should not be interrupted.

A 37-year-old man is brought to the clinic by his wife because he is experiencing loss of motor function and sensation. The physician suspects the patient has a spinal cord tumor and hospitalizes him for diagnostic testing. In light of the need to diagnose spinal cord compression from a tumor, the nurse will most likely prepare the patient for what test? A. Anterior-posterior x-ray B. Ultrasound C. Lumbar puncture D. MRI

Ans: D Feedback: The MRI scan is the most commonly used diagnostic procedure. It is the most sensitive diagnostic tool that is particularly helpful in detecting epidural spinal cord compression and vertebral bone metastases.

A patient who was diagnosed with Parkinsons disease several months ago recently began treatment with levodopa-carbidopa. The patient and his family are excited that he has experienced significant symptom relief. The nurse should be aware of what implication of the patients medication regimen? A. The patient is in a honeymoon period when adverse effects of levodopa-carbidopa are not yet evident. B. Benefits of levodopa-carbidopa do not peak until 6 to 9 months after the initiation of treatment. C. The patients temporary improvement in status is likely unrelated to levodopa-carbidopa. D. Benefits of levodopa-carbidopa often diminish after 1 or 2 years of treatment.

Ans: D Feedback: The beneficial effects of levodopa therapy are most pronounced in the first year or two of treatment. Benefits begin to wane and adverse effects become more severe over time. However, a honeymoon period of treatment is not known.

A male patient with a metastatic brain tumor is having a generalized seizure and begins vomiting. What should the nurse do first? A. Perform oral suctioning. B. Page the physician. C. Insert a tongue depressor into the patients mouth. D. Turn the patient on his side.

Ans: D Feedback: The nurses first response should be to place the patient on his side to prevent him from aspirating emesis. Inserting something into the seizing patients mouth is no longer part of a seizure protocol. Obtaining supplies to suction the patient would be a delegated task. Paging or calling the physician would only be necessary if this is the patients first seizure.

7. The nurse is creating a plan of care for a patient who has a recent diagnosis of MS. Which of the following should the nurse include in the patient's care plan? A) Encourage patient to void every hour. B) Order a low-residue diet. C) Provide total assistance with all ADLs. D) Instruct the patient on daily muscle stretching.

Ans: D A patient diagnosed with MS should be encouraged to increase the fiber in his or her diet and void 30 minutes after drinking to help train the bladder. The patient should participate in daily muscle stretching to help alleviate and relax muscle spasms.

32. A patient diagnosed with Bell's palsy is having decreased sensitivity to touch of the involved nerve. What should the nurse recommend to prevent atrophy of the muscles? A) Blowing up balloons B) Deliberately frowning C) Smiling repeatedly D) Whistling

Ans: D Facial exercises, such as wrinkling the forehead, blowing out the cheeks, and whistling, may be performed with the aid of a mirror to prevent muscle atrophy. Blowing up balloons, frowning, and smiling are not considered facial exercises.

5. A patient with a documented history of seizure disorder experiences a generalized seizure. What nursing action is most appropriate? A) Restrain the patient to prevent injury. B) Open the patients jaws to insert an oral airway. C) Place patient in high Fowlers position. D) Loosen the patients restrictive clothing.

Ans: D Feedback: An appropriate nursing intervention would include loosening any restrictive clothing on the patient. No attempt should be made to restrain the patient during the seizure because muscular contractions are strong and restraint can produce injury. Do not attempt to pry open jaws that are clenched in a spasm to insert anything. Broken teeth and injury to the lips and tongue may result from such an action. If possible, place the patient on one side with head flexed forward, which allows the tongue to fall forward and facilitates drainage of saliva and mucus.

36. The nurse is caring for a patient who has undergone supratentorial removal of a pituitary mass. What medication would the nurse expect to administer prophylactically to prevent seizures in this patient? A) Prednisone B) Dexamethasone C) Cafergot D) Phenytoin

Ans: D Feedback: Antiseizure medication (phenytoin, diazepam) is often prescribed prophylactically for patients who have undergone supratentorial craniotomy because of the high risk of seizures after this procedure. Prednisone and dexamethasone are steroids and do not prevent seizures. Cafergot is used in the treatment of migraines.

22. The nurse is providing care for a patient who is withdrawing from heavy alcohol use. The nurse and other members of the care team are present at the bedside when the patient has a seizure. In preparation for documenting this clinical event, the nurse should note which of the following? A) The ability of the patient to follow instructions during the seizure. B) The success or failure of the care team to physically restrain the patient. C) The patients ability to explain his seizure during the postictal period. D) The patients activities immediately prior to the seizure.

Ans: D Feedback: Before and during a seizure, the nurse observes the circumstances before the seizure, including visual, auditory, or olfactory stimuli, tactile stimuli, emotional or psychological disturbances, sleep, and hyperventilation. Communication with the patient is not possible during a seizure and physical restraint is not attempted. The patients ability to explain the seizure is not clinically relevant.

18. What should the nurse suspect when hourly assessment of urine output on a patient postcraniotomy exhibits a urine output from a catheter of 1,500 mL for two consecutive hours? A) Cushing syndrome B) Syndrome of inappropriate antidiuretic hormone (SIADH) C) Adrenal crisis D) Diabetes insipidus

Ans: D Feedback: Diabetes insipidus is an abrupt onset of extreme polyuria that commonly occurs in patients after brain surgery. Cushing syndrome is excessive glucocorticoid secretion resulting in sodium and water retention. SIADH is the result of increased secretion of ADH, the patient becomes volume-overloaded, urine output diminishes, and serum sodium concentration becomes dilute. Adrenal crisis is undersecretion of glucocorticoids resulting in profound hypoglycemia, hypovolemia, and hypotension.

27. A patient has a poor prognosis after being involved in a motor vehicle accident resulting in a head injury. As the patients ICP increases and condition worsens, the nurse knows to assess for indications of approaching death. These indications include which of the following? A) Hemiplegia B) Dry mucous membranes C) Signs of internal bleeding D) Loss of brain stem reflexes

Ans: D Feedback: Loss of brain stem reflexes, including pupillary, corneal, gag, and swallowing reflexes, is an ominous sign of approaching death. Dry mucous membranes, hemiplegia, and bleeding must be promptly addressed, but none of these is a common sign of impending death.

19. During the examination of an unconscious patient, the nurse observes that the patients pupils are fixed and dilated. What is the most plausible clinical significance of the nurses finding? A) It suggests onset of metabolic problems. B) It indicates paralysis on the right side of the body. C) It indicates paralysis of cranial nerve X. D) It indicates an injury at the midbrain level.

Ans: D Feedback: Pupils that are fixed and dilated indicate injury at the midbrain level. This finding is not suggestive of unilateral paralysis, metabolic deficits, or damage to CN X.

11. The nurse caring for a patient diagnosed with Guillain-Barré syndrome is planning care with regard to the clinical manifestations associated this syndrome. The nurse's communication with the patient should reflect the possibility of what sign or symptom of the disease? A) Intermittent hearing loss B) Tinnitus C) Tongue enlargement D) Vocal paralysis

Ans: D Guillain-Barré syndrome is a disorder of the vagus nerve. Clinical manifestations include vocal paralysis, dysphagia, and voice changes (temporary or permanent hoarseness). Hearing deficits, tinnitus, and tongue enlargement are not associated with the disease.

12. The nurse is preparing to provide care for a patient diagnosed with myasthenia gravis. The nurse should know that the signs and symptoms of the disease are the result of what? A) Genetic dysfunction B) Upper and lower motor neuron lesions C) Decreased conduction of impulses in an upper motor neuron lesion D) A lower motor neuron lesion

Ans: D Myasthenia gravis is characterized by a weakness of muscles, especially in the face and throat, caused by a lower neuron lesion at the myoneural junction. It is not a genetic disorder. A combined upper and lower neuron lesion generally occurs as a result of spinal injuries. A lesion involving cranial nerves and their axons in the spinal cord would cause decreased conduction of impulses at an upper motor neuron.

13. A patient with suspected Creutzfeldt-Jakob disease (CJD) is being admitted to the unit. The nurse would expect what diagnostic test to be ordered for this patient? A) Cerebral angiography B) ABG analysis C) CT D) EEG

Ans: D The EEG reveals a characteristic pattern over the duration of CJD. A CT scan may be used to rule out disorders that may mimic the symptoms of CJD. ABGs would not be necessary until the later stages of CJD; they would not be utilized as a diagnostic test. Cerebral angiography is not used to diagnose CJD.

36. A 73-year-old man comes to the clinic complaining of weakness and loss of sensation in his feet and legs. Assessment of the patient shows decreased reflexes bilaterally. Why would it be a challenge to diagnose a peripheral neuropathy in this patient? A) Older adults are often vague historians. B) The elderly have fewer peripheral nerves than younger adults. C) Many older adults are hesitant to admit that their body is changing. D) Many symptoms can be the result of normal aging process..

Ans: D The diagnosis of peripheral neuropathy in the geriatric population is challenging because many symptoms, such as decreased reflexes, can be associated with the normal aging process. In this scenario, the patient has come to the clinic seeking help for his problem; this does not indicate a desire on the part of the patient to withhold information from the health care giver. The normal aging process does not include a diminishing number of peripheral nerves

40. A patient presents at the clinic complaining of pain and weakness in her hands. On assessment, the nurse notes diminished reflexes in the upper extremities bilaterally and bilateral loss of sensation. The nurse knows that these findings are indicative of what? A) Guillain-Barré syndrome B) Myasthenia gravis C) Trigeminal neuralgia D) Peripheral nerve disorder

Ans: D The major symptoms of peripheral nerve disorders are loss of sensation, muscle atrophy, weakness, diminished reflexes, pain, and paresthesia (numbness, tingling) of the extremities. Trigeminal neuralgia is a condition of the fifth cranial nerve that is characterized by paroxysms of pain in the area innervated by any of the three branches, but most commonly the second and third branches of the trigeminal nerve. Myasthenia gravis, an autoimmune disorder affecting the myoneural junction, is characterized by varying degrees of weakness of the voluntary muscles. Guillain-Barré syndrome is an autoimmune attack on the peripheral nerve myelin.

16. A middle-aged woman has sought care from her primary care provider and undergone diagnostic testing that has resulted in a diagnosis of MS. What sign or symptom is most likely to have prompted the woman to seek care? A) Cognitive declines B) Personality changes C) Contractures D) Difficulty in coordination

Ans: D The primary symptoms of MS most commonly reported are fatigue, depression, weakness, numbness, difficulty in coordination, loss of balance, spasticity, and pain. Cognitive changes and contractures usually occur later in the disease.

A nurse is preparing to discharge an emergency department patient who has been fitted with a sling to support her arm after a clavicle fracture. What should the nurse instruct the patient to do? A) Elevate the arm above the shoulder 3 to 4 times daily. B) Avoid moving the elbow, wrist, and fingers until bone remodeling is complete. C) Engage in active range of motion using the affected arm. D) Use the arm for light activities within the range of motion.

Ans: D Feedback: A patient with a clavicle fracture may use a sling to support the arm and relieve the pain. The patient may be permitted to use the arm for light activities within the range of comfort. The patient should not elevate the arm above the shoulder level until the ends of the bones have united, but the nurse should encourage the patient to exercise the elbow, wrist, and fingers.

Radiographs of a boy's upper arm show that the humerus appears to be fractured on one side and slightly bent on the other. This diagnostic result suggests what type of fracture? A) Impacted B) Compound C) Compression D) Greenstick

Ans: D Feedback: Greenstick fractures are an incomplete fracture that results in the bone being broken on one side, while the other side is bent. This is not characteristic of an impacted, compound, or compression fracture.

An older adult patient sought care for the treatment of a swollen, painful knee joint. Diagnostic imaging and culturing of synovial fluid resulted in a diagnosis of septic arthritis. The nurse should prioritize which of the following aspects of care? A) Administration of oral and IV corticosteroids as ordered B) Prevention of falls and pathologic fractures C) Maintenance of adequate serum levels of vitamin D D) Intravenous administration of antibiotics

Ans: D Feedback: IV antibiotics are the major treatment modality for septic arthritis; the nurse must ensure timely administration of these drugs. Corticosteroids are not used to treat septic arthritis and vitamin D levels are not necessarily affected. Falls prevention is important, but septic arthritis does not constitute the same fracture risk as diseases with decreased bone density.

An emergency department patient is diagnosed with a hip dislocation. The patient's family is relieved that the patient has not suffered a hip fracture, but the nurse explains that this is still considered to be a medical emergency. What is the rationale for the nurse's statement? A) The longer the joint is displaced, the more difficult it is to get it back in place. B) The patient's pain will increase until the joint is realigned. C) Dislocation can become permanent if the process of bone remodeling begins. D) Avascular necrosis may develop at the site of the dislocation if it is not promptly resolved.

Ans: D Feedback: If a dislocation or subluxation is not reduced immediately, avascular necrosis (AVN) may develop. Bone remodeling does not take place because a fracture has not occurred. Realignment does not become more difficult with time and pain would subside with time, not become worse.

A patient presents at the clinic with complaints of morning numbness, cramping, and stiffness in his fourth and fifth fingers. What disease process should the nurse suspect? A) Tendonitis B) A ganglion C) Carpal tunnel syndrome D) Dupuytren's disease

Ans: D Feedback: In cases of Dupuytren's disease, the patient may experience dull, aching discomfort, morning numbness, cramping, and stiffness in the affected fingers. This condition starts in one hand, but eventually both hands are affected. This clinical scenario does not describe tendonitis, a ganglion, or carpal tunnel syndrome.

A patient is undergoing diagnostic testing for osteomalacia. Which of the following laboratory results is most suggestive of this diagnosis? A) High chloride, calcium, and magnesium B) High parathyroid and calcitonin levels C) Low serum calcium and magnesium levels D) Low serum calcium and low phosphorus level

Ans: D Feedback: Laboratory studies will reveal a low serum calcium and low phosphorus level.

An orthopedic nurse is caring for a patient who is postoperative day one following foot surgery. What nursing intervention should be included in the patient's subsequent care? A) Dressing changes should not be performed unless there are clear signs of infection. B) The surgical site can be soaked in warm bath water for up to 5 minutes. C) The surgical site should be cleansed with hydrogen peroxide once daily. D) The foot should be elevated in order to prevent edema.

Ans: D Feedback: Pain experienced by patients who undergo foot surgery is related to inflammation and edema. To control the anticipated edema, the foot should be elevated on several pillows when the patient is sitting or lying. Regular dressing changes are performed and the wound should be kept dry. Hydrogen peroxide is not used to cleanse surgical wounds.

A nurse is reviewing the care of a patient who has a long history of lower back pain that has not responded to conservative treatment measures. The nurse should anticipate the administration of what drug? A) Calcitonin B) Prednisone C) Aspirin D) Cyclobenzaprine

Ans: D Feedback: Short-term prescription muscle relaxants (e.g., cyclobenzaprine [Flexeril]) are effective in relieving acute low back pain. ASA is not normally used for pain control, due to its antiplatelet action and associated risk for bleeding. Calcitonin and corticosteroids are not used in the treatment of lower back pain.

A patient is brought to the emergency department by ambulance after stepping in a hole and falling. While assessing him the nurse notes that his right leg is shorter than his left leg; his right hip is noticeably deformed and he is in acute pain. Imaging does not reveal a fracture. Which of the following is the most plausible explanation for this patient's signs and symptoms? A) Subluxated right hip B) Right hip contusion C) Hip strain D) Traumatic hip dislocation

Ans: D Feedback: Signs and symptoms of a traumatic dislocation include acute pain, change in positioning of the joint, shortening of the extremity, deformity, and decreased mobility. A subluxation would cause moderate deformity, or possibly no deformity. A contusion or strain would not cause obvious deformities.

A patient who has had an amputation is being cared for by a multidisciplinary rehabilitation team. What is the primary goal of this multidisciplinary team? A) Maximize the efficiency of care B) Ensure that the patient's health care is holistic C) Facilitate the patient's adjustment to a new body image D) Promote the patient's highest possible level of function

Ans: D Feedback: The multidisciplinary rehabilitation team helps the patient achieve the highest possible level of function and participation in life activities. The team is not primarily motivated by efficiency, the need for holistic care, or the need to foster the patient's body image, despite the fact that each of these are valid goals.

A nurse is preparing to discharge a patient from the emergency department after receiving treatment for an ankle sprain. While providing discharge education, the nurse should encourage which of the following? A) Apply heat for the first 24 to 48 hours after the injury. B) Maintain the ankle in a dependent position. C) Exercise hourly by performing rotation exercises of the ankle. D) Keep an elastic compression bandage on the ankle.

Ans: D Feedback: Treatment of a sprain consists of resting and elevating the affected part, applying cold, and using a compression bandage. After the acute inflammatory stage (usually 24 to 48 hours after injury), heat may be applied intermittently. Rotation exercises would likely be painful.

A nurse is providing a class on osteoporosis at the local seniors' center. Which of the following statements related to osteoporosis is most accurate? A) Osteoporosis is categorized as a disease of the elderly. B) A nonmodifiable risk factor for osteoporosis is a person's level of activity. C) Secondary osteoporosis occurs in women after menopause. D) Slow discontinuation of corticosteroid therapy can halt the progression of the osteoporosis.

Ans: D Feedback: When corticosteroid therapy is discontinued, the progression of osteoporosis is halted, but restoration of lost bone mass does not occur. Osteoporosis is not a disease of the elderly because its onset occurs earlier in life, when bone mass peaks and then begins to decline. A person's level of physical activity is a modifiable factor that influences peak bone mass. Lack of activity increases the risk for the development of osteoporosis. Primary osteoporosis occurs in women after menopause.

A nurse is caring for a patient who has had a plaster arm cast applied. Immediately postapplication, the nurse should provide what teaching to the patient? A) The cast will feel cool to touch for the first 30 minutes. B) The cast should be wrapped snuggly with a towel until the patient gets home. C) The cast should be supported on a board while drying. D) The cast will only have full strength when dry.

Ans: D Feedback: A cast requires approximately 24 to 72 hours to dry, and until dry, it does not have full strength. While drying, the cast should not be placed on a hard surface. The cast will exude heat while it dries and should not be wrapped.

A nurse is caring for a patient who is recovering in the hospital following orthopedic surgery. The nurse is performing frequent assessments for signs and symptoms of infection in the knowledge that the patient faces a high risk of what infectious complication? A) Cellulitis B) Septic arthritis C) Sepsis D) Osteomyelitis

Ans: D Feedback: Infection is a risk after any surgery, but it is of particular concern for the postoperative orthopedic patient because of the risk of osteomyelitis. Orthopedic patients do not have an exaggerated risk of cellulitis, sepsis, or septic arthritis when compared to other surgical patients.

A patient is scheduled for a total hip replacement and the surgeon has explained the risks of blood loss associated with orthopedic surgery. The risk of blood loss is the indication for which of the following actions? A) Use of a cardiopulmonary bypass machine B) Postoperative blood salvage C) Prophylactic blood transfusion D) Autologous blood donation

Ans: D Feedback: Many patients donate their own blood during the weeks preceding their surgery. Autologous blood donations are cost effective and eliminate many of the risks of transfusion therapy. Orthopedic surgery does not necessitate cardiopulmonary bypass and blood is not salvaged postoperatively. Transfusions are not given prophylactically.

A nurse is providing discharge education to a patient who is going home with a cast on his leg. What teaching point should the nurse emphasize in the teaching session? A) Using crutches efficiently B) Exercising joints above and below the cast, as ordered C) Removing the cast correctly at the end of the treatment period D) Reporting signs of impaired circulation

Ans: D Feedback: Reporting signs of impaired circulation is critical; signs of impaired circulation must be reported to the physician immediately to prevent permanent damage. For this reason, this education is a priority over exercise and crutch use. The patient does not independently remove the cast.

A nurse is assessing the neurovascular status of a patient who has had a leg cast recently applied. The nurse is unable to palpate the patient's dorsalis pedis or posterior tibial pulse and the patient's foot is pale. What is the nurse's most appropriate action? A) Warm the patient's foot and determine whether circulation improves. B) Reposition the patient with the affected foot dependent. C) Reassess the patient's neurovascular status in 15 minutes. D) Promptly inform the primary care provider.

Ans: D Feedback: Signs of neurovascular dysfunction warrant immediate medical follow-up. It would be unsafe to delay. Warming the foot or repositioning the patient may be of some benefit, but the care provider should be informed first.

A nurse is emptying an orthopedic surgery patient's closed suction drainage at the end of a shift. The nurse notes that the volume is within expected parameters but that the drainage has a foul odor. What is the nurse's best action? A) Aspirate a small amount of drainage for culturing. B) Advance the drain 1 to 1.5 cm. C) Irrigate the drain with normal saline. D) Inform the surgeon of this finding.

Ans: D Feedback: The nurse should promptly notify the surgeon of excessive or foul-smelling drainage. It would be inappropriate to advance the drain, irrigate the drain, or aspirate more drainage.

A nurse is caring for a patient who is in skeletal traction. To prevent the complication of skin breakdown in a patient with skeletal traction, what action should be included in the plan of care? A) Apply occlusive dressings to the pin sites. B) Encourage the patient to push up with the elbows when repositioning. C) Encourage the patient to perform isometric exercises once a shift. D) Assess the pin insertion site every 8 hours.

Ans: D Feedback: The pin insertion site should be assessed every 8 hours for inflammation and infection. Loose cover dressings should be applied to pin sites. The patient should be encouraged to use the overhead trapeze to shift weight for repositioning. Isometric exercises should be done 10 times an hour while awake.

A nurse is caring for a patient who has had a total hip replacement. The nurse is reviewing health education prior to discharge. Which of the patient's statements would indicate to the nurse that the patient requires further teaching? A) "I'll need to keep several pillows between my legs at night." B) "I need to remember not to cross my legs. It's such a habit." C) "The occupational therapist is showing me how to use a 'sock puller' to help me get dressed." D) "I will need my husband to assist me in getting off the low toilet seat at home."

Ans: D Feedback: To prevent hip dislocation after a total hip replacement, the patient must avoid bending the hips beyond 90 degrees. Assistive devices, such as a raised toilet seat, should be used to prevent severe hip flexion. Using an abduction pillow or placing several pillows between the legs reduces the risk of hip dislocation by preventing adduction and internal rotation of the legs. Likewise, teaching the patient to avoid crossing the legs also reduces the risk of hip dislocation. A sock puller helps a patient get dressed without flexing the hips beyond 90 degrees.

A patient with a fractured femur is in balanced suspension traction. The patient needs to be repositioned toward the head of the bed. During repositioning, what should the nurse do? A) Place slight additional tension on the traction cords. B) Release the weights and replace them immediately after positioning. C) Reposition the bed instead of repositioning the patient. D) Maintain consistent traction tension while repositioning.

Ans: D Feedback: Traction is used to reduce the fracture and must be maintained at all times, including during repositioning. It would be inappropriate to add tension or release the weights. Moving the bed instead of the patient is not feasible.

A nurse is providing an educational class to a group of older adults at a community senior center. In an effort to prevent osteoporosis, the nurse should encourage participants to ensure that they consume the recommended adequate intake of what nutrients? Select all that apply. A) Vitamin B12 B) Potassium C) Calcitonin D) Calcium E) Vitamin D

Ans: D, E Feedback: A diet rich in calcium and vitamin D protects against skeletal demineralization. Intake of vitamin B12and potassium does not directly influence the risk for osteoporosis. Calcitonin is not considered to be a dietary nutrient.

34. A patient diagnosed with MS has been admitted to the medical unit for treatment of an MS exacerbation. Included in the admission orders is baclofen (Lioresal). What should the nurse identify as an expected outcome of this treatment? A) Reduction in the appearance of new lesions on the MRI B) Decreased muscle spasms in the lower extremities C) Increased muscle strength in the upper extremities D) Decreased severity and duration of exacerbations

Ans:B Baclofen, a g-aminobutyric acid (GABA) agonist, is the medication of choice in treating spasms. It can be administered orally or by intrathecal injection. Avonex and Betaseron reduce the appearance of new lesions on the MRI. Corticosteroids limit the severity and duration of exacerbations. Anticholinesterase agents increase muscle strength in the upper extremities.

24. A patient is admitted through the ED with suspected St. Louis encephalitis. The unique clinical feature of St. Louis encephalitis will make what nursing action a priority? A) Serial assessments of hemoglobin levels B) Blood glucose monitoring C) Close monitoring of fluid balance D) Assessment of pain along dermatomes

Ans:C A unique clinical feature of St. Louis encephalitis is SIADH with hyponatremia. As such, it is important to monitor the patient's intake and output closely.

28. The nurse caring for a patient in ICU diagnosed with Guillain-Barré syndrome should prioritize monitoring for what potential complication? A) Impaired skin integrity B) Cognitive deficits C) Hemorrhage D) Autonomic dysfunction

Ans:D Based on the assessment data, potential complications that may develop include respiratory failure and autonomic dysfunction. Skin breakdown, decreased cognition, and hemorrhage are not complications of Guillain-Barré syndrome.

8. The nurse is caring for a patient with increased intracranial pressure (ICP). The patient has a nursing diagnosis of ineffective cerebral tissue perfusion. What would be an expected outcome that the nurse would document for this diagnosis? A) Copes with sensory deprivation. B) Registers normal body temperature. C) Pays attention to grooming. D) Obeys commands with appropriate motor responses.

Ans:D Feedback: An expected outcome of the diagnosis of ineffective cerebral tissue perfusion in a patient with increased intracranial pressure (ICP) would include obeying commands with appropriate motor responses. Vitals signs and neurologic status are assessed every 15 minutes to every hour. Coping with sensory deprivation would relate to the nursing diagnosis of disturbed sensory perception. The outcome of registers normal body temperature relates to the diagnosis of potential for ineffective thermoregulation. Body image disturbance would have a potential outcome of pays attention to grooming.

When caring for a patient who had a hemorrhagic stroke, close monitoring of vital signs and neurologic changes is imperative. What is the earliest sign of deterioration in a patient with a hemorrhagic stroke of which the nurse should be aware? A) Generalized pain B) Alteration in level of consciousness (LOC) C) Tonic-clonic seizures D) Shortness of breath

B (Feedback: Alteration in LOC is the earliest sign of deterioration in a patient after a hemorrhagic stroke, such as mild drowsiness, slight slurring of speech, and sluggish papillary reaction. Sudden headache may occur, but generalized pain is less common. Seizures and shortness of breath are not identified as early signs of hemorrhagic stroke.)

The nurse is caring for a patient recovering from an ischemic stroke. What intervention best addresses a potential complication after an ischemic stroke? A) Providing frequent small meals rather than three larger meals B) Teaching the patient to perform deep breathing and coughing exercises C) Keeping a urinary catheter in situ for the full duration of recovery D) Limiting intake of insoluble fiber

B (Feedback: Because pneumonia is a potential complication of stroke, deep breathing and coughing exercises should be encouraged unless contraindicated. No particular need exists to provide frequent meals and normally fiber intake should not be restricted. Urinary catheters should be discontinued as soon as possible.)

A preceptor is discussing stroke with a new nurse on the unit. The preceptor would tell the new nurse which cardiac dysrhythmia is associated with cardiogenic embolic strokes? A) Ventricular tachycardia B) Atrial fibrillation C) Supraventricular tachycardia D) Bundle branch block

B (Feedback: Cardiogenic embolic strokes are associated with cardiac dysrhythmias, usually atrial fibrillation. The other listed dysrhythmias are less commonly associated with this type of stroke.)

A patient has been admitted to the ICU after being recently diagnosed with an aneurysm and the patient's admission orders include specific aneurysm precautions. What nursing action will the nurse incorporate into the patient's plan of care? A) Elevate the head of the bed to 45 degrees. B) Maintain the patient on complete bed rest. C) Administer enemas when the patient is constipated. D) Avoid use of thigh-high elastic compression stockings.

B (Feedback: Cerebral aneurysm precautions are implemented for the patient with a diagnosis of aneurysm to provide a nonstimulating environment, prevent increases in ICP, and prevent further bleeding. The patient is placed on immediate and absolute bed rest in a quiet, nonstressful environment because activity, pain, and anxiety elevate BP, which increases the risk for bleeding. Visitors, except for family, are restricted. The head of the bed is elevated 15 to 30 degrees to promote venous drainage and decrease ICP. Some neurologists, however, prefer that the patient remains flat to increase cerebral perfusion. No enemas are permitted, but stool softeners and mild laxatives are prescribed. Thigh-high elastic compression stockings or sequential compression boots may be ordered to decrease the patient's risk for deep vein thrombosis (DVT).)

A patient has recently begun mobilizing during the recovery from an ischemic stroke. To protect the patient's safety during mobilization, the nurse should perform what action? A) Support the patient's full body weight with a waist belt during ambulation. B) Have a colleague follow the patient closely with a wheelchair. C) Avoid mobilizing the patient in the early morning or late evening. D) Ensure that the patient's family members do not participate in mobilization.

B (Feedback: During mobilization, a chair or wheelchair should be readily available in case the patient suddenly becomes fatigued or feels dizzy. The family should be encouraged to participate, as appropriate, and the nurse should not have to support the patient's full body weight. Morning and evening activity are not necessarily problematic.)

A nurse is caring for a patient diagnosed with a hemorrhagic stroke. When creating this patient's plan of care, what goal should be prioritized? A) Prevent complications of immobility. B) Maintain and improve cerebral tissue perfusion. C) Relieve anxiety and pain. D) Relieve sensory deprivation.

B (Feedback: Each of the listed goals is appropriate in the care of a patient recovering from a stroke. However, promoting cerebral perfusion is a priority physiologic need, on which the patient's survival depends.)

When caring for a patient who has had a stroke, a priority is reduction of ICP. What patient position is most consistent with this goal? A) Head turned slightly to the right side B) Elevation of the head of the bed C) Position changes every 15 minutes while awake D) Extension of the neck

B (Feedback: Elevation of the head of the bed promotes venous drainage and lowers ICP; the nurse should avoid flexing or extending the neck or turning the head side to side. The head should be in a neutral midline position. Excessively frequent position changes are unnecessary.)

A rehabilitation nurse caring for a patient who has had a stroke is approached by the patient's family and asked why the patient has to do so much for herself when she is obviously struggling. What would be the nurse's best answer? A) We are trying to help her be as useful as she possibly can. B) The focus on care in a rehabilitation facility is to help the patient to resume as much self-care as possible. C) We aren't here to care for her the way the hospital staff did; we are here to help her get better so she can go home. D) Rehabilitation means helping patients do exactly what they did before their stroke.

B (Feedback: In both acute care and rehabilitation facilities, the focus is on teaching the patient to resume as much self-care as possible. The goal of rehabilitation is not to be useful, nor is it to return patients to their prestroke level of functioning, which may be unrealistic.)

A family member brings the patient to the clinic for a follow-up visit after a stroke. The family member asks the nurse what he can do to decrease his chance of having another stroke. What would be the nurse's best answer? A) Have your heart checked regularly. B) Stop smoking as soon as possible. C) Get medication to bring down your sodium levels. D) Eat a nutritious diet.

B (Feedback: Smoking is a modifiable and highly significant risk factor for stroke. The significance of smoking, and the potential benefits of quitting, exceed the roles of sodium, diet, and regular medical assessments.)

The nurse is performing stroke risk screenings at a hospital open house. The nurse has identified four patients who might be at risk for a stroke. Which patient is likely at the highest risk for a hemorrhagic stroke? A) White female, age 60, with history of excessive alcohol intake B) White male, age 60, with history of uncontrolled hypertension C) Black male, age 60, with history of diabetes D) Black male, age 50, with history of smoking

B (Feedback: Uncontrolled hypertension is the primary cause of a hemorrhagic stroke. Control of hypertension, especially in individuals over 55 years of age, clearly reduces the risk for hemorrhagic stroke. Additional risk factors are increased age, male gender, and excessive alcohol intake. Another high-risk group includes African Americans, where the incidence of first stroke is almost twice that as in Caucasians.)

17. The nurse is planning the care of a patient with hyperthyroidism. What should the nurse specify in the patients meal plan? A) A clear liquid diet, high in nutrients B) Small, frequent meals, high in protein and calories C) Three large, bland meals a day D) A diet high in fiber and plant-sourced fat

B Feedback: A patient with hyperthyroidism has an increased appetite. The patient should be counseled to consume several small, well-balanced meals. High-calorie, high-protein foods are encouraged. A clear liquid diet would not satisfy the patients caloric or hunger needs. A diet rich in fiber and fat should be avoided because these foods may lead to GI upset or increase peristalsis.

5. The nurse is caring for a patient with Addisons disease who is scheduled for discharge. When teaching the patient about hormone replacement therapy, the nurse should address what topic? A) The possibility of precipitous weight gain B) The need for lifelong steroid replacement C) The need to match the daily steroid dose to immediate symptoms D) The importance of monitoring liver function

B Feedback: Because of the need for lifelong replacement of adrenal cortex hormones to prevent addisonian crises, the patient and family members receive explicit education about the rationale for replacement therapy and proper dosage. Doses are not adjusted on a short-term basis. Weight gain and hepatotoxicity are not common adverse effects.

15. While assisting with the surgical removal of an adrenal tumor, the OR nurse is aware that the patients vital signs may change upon manipulation of the tumor. What vital sign changes would the nurse expect to see? A) Hyperthermia and tachypnea B) Hypertension and heart rate changes C) Hypotension and hypothermia D) Hyperthermia and bradycardia

B Feedback: Manipulation of the tumor during surgical excision may cause release of stored epinephrine and norepinephrine, with marked increases in BP and changes in heart rate. The use of sodium nitroprusside and alpha-adrenergic blocking agents may be required during and after surgery. While other vital sign changes may occur related to surgical complications, the most common changes are related to hypertension and changes in the heart rate.

39. A patient who has been taking corticosteroids for several months has been experiencing muscle wasting. The patient has asked the nurse for suggestions to address this adverse effect. What should the nurse recommend? A) Activity limitation to conserve energy B) Consumption of a high-protein diet C) Use of OTC vitamin D and calcium supplements D) Passive range-of-motion exercises

B Feedback: Muscle wasting can be partly addressed through increased protein intake. Passive ROM exercises maintain flexibility, but do not build muscle mass. Vitamin D and calcium supplements do not decrease muscle wasting. Activity limitation would exacerbate the problem.

22. The physician has ordered a fluid deprivation test for a patient suspected of having diabetes insipidus. During the test, the nurse should prioritize what assessments? A) Temperature and oxygen saturation B) Heart rate and BP C) Breath sounds and bowel sounds D) Color, warmth, movement, and sensation of extremities

B Feedback: The fluid deprivation test is carried out by withholding fluids for 8 to 12 hours or until 3% to 5% of the body weight is lost. The patients condition needs to be monitored frequently during the test, and the test is terminated if tachycardia, excessive weight loss, or hypotension develops. Consequently, BP and heart rate monitoring are priorities over the other listed assessments.

36. What should the nurse teach a patient on corticosteroid therapy in order to reduce the patients risk of adrenal insufficiency? A) Take the medication late in the day to mimic the bodys natural rhythms. B) Always have enough medication on hand to avoid running out. C) Skip up to 2 doses in cases of illness involving nausea. D) Take up to 1 extra dose per day during times of stress.

B Feedback: The patient and family should be informed that acute adrenal insufficiency and underlying symptoms will recur if corticosteroid therapy is stopped abruptly without medical supervision. The patient should be instructed to have an adequate supply of the corticosteroid medication always available to avoid running out. Doses should not be skipped or added without explicit instructions to do so. Corticosteroids should normally be taken in the morning to mimic natural rhythms.

18. A patient with a diagnosis of syndrome of inappropriate antidiuretic hormone secretion (SIADH) is being cared for on the critical care unit. The priority nursing diagnosis for a patient with this condition is what? A) Risk for peripheral neurovascular dysfunction B) Excess fluid volume C) Hypothermia D) Ineffective airway clearance

B Feedback: The priority nursing diagnosis for a patient with SIADH is excess fluid volume, as the patient retains fluids and develops a sodium deficiency. Restricting fluid intake is a typical intervention for managing this syndrome. Temperature imbalances are not associated with SIADH. The patient is not at risk for neurovascular dysfunction or a compromised airway.

26. The nurses assessment of a patient with thyroidectomy suggests tetany and a review of the most recent blood work corroborate this finding. The nurse should prepare to administer what intervention? A) Oral calcium chloride and vitamin D B) IV calcium gluconate C) STAT levothyroxine D) Administration of parathyroid hormone (PTH)

B Feedback: When hypocalcemia and tetany occur after a thyroidectomy, the immediate treatment is administration of IV calcium gluconate. This has a much faster therapeutic effect than PO calcium or vitamin D supplements. PTH and levothyroxine are not used to treat this complication.

8. A nurse is doing a shift assessment on a group of patients after first taking report. An elderly patient is having her second dose of IV antibiotics for a diagnosis of pneumonia. The nurse notices a new rash on the patient's chest. The nurse should ask what priority question regarding the presence of a reddened rash? A) Is the rash worse at a particular time or season? B) Are you allergic to any foods or medication? C) Are you having any loss of sensation in that area? D) Is your rash painful?

B) Are you allergic to any foods or medication? Ans: B Feedback: The nurse should suspect an allergic reaction to the antibiotic therapy. Allergies can be a significant threat to the patient's immediate health, thus questions addressing this possibility would be prioritized over those addressing sensation. Asking about previous rashes is important, but this should likely be framed in the context of an allergy assessment.

31. A nurse practitioner working in a dermatology clinic finds an open lesion on a patient who is being assessed. What should the nurse do next? A) Obtain a swab for culture. B) Assess the characteristics of the lesion. C) Obtain a swab for pH testing. D) Apply a test dose of broad-spectrum topical antibiotic.

B) Assess the characteristics of the lesion. Ans: B Feedback: If acute open wounds or lesions are found on inspection of the skin, a comprehensive assessment should be made and documented. Testing for culture and pH are not necessarily required, and assessment should precede these actions. Antibiotics are not applied on an empiric basis.

1. A nurse practitioner is seeing a 16-year-old male patient who has come to the dermatology clinic for treatment of acne. The nurse practitioner would know that the treatment may consist of which of the following medications? A) Acyclovir (Zovirax) B) Benzoyl peroxide and erythromycin (Benzamycin) C) Diphenhydramine (Benadryl) D) Triamcinolone (Kenalog)

B) Benzoyl peroxide and erythromycin (Benzamycin) Ans: B Feedback: Benzamycin gel is among the topical treatments available for acne. Zovirax is used in the treatment of herpes zoster as an oral antiviral agent. Benadryl is an oral antihistamine used in the treatment of pruritus. Intralesional injections of Kenalog have been utilized in the treatment of psoriasis.

9. A gerontologic nurse is teaching a group of nursing students about integumentary changes that occur in older adults. How should these students best integrate these changes into care planning? A) By avoiding the use of moisturizing lotions on older adults' skin B) By protecting older adults against shearing injuries C) By avoiding the use of ice packs to treat muscle pain D) By protecting older adults against excessive sweat accumulation

B) By protecting older adults against shearing injuries Ans: B Feedback: Cellular changes associated with aging include thinning at the junction of the dermis and epidermis, which creates a risk for shearing injuries. Moisturizing lotions can be safely used to address the increased dryness of older adults' skin. Ice packs can be used, provided skin is assessed regularly and the patient possesses normal sensation. Older adults perspire much less than younger adults, thus sweat accumulation is rarely an issue.

13. A nurse is explaining the importance of sunlight on the skin to a woman with decreased mobility who rarely leaves her house. The nurse would emphasize that ultraviolet light helps to synthesize what vitamin? A) E B) D C) A D) C

B) D Ans: B Feedback: Skin exposed to ultraviolet light can convert substances necessary for synthesizing vitamin D (cholecalciferol). Vitamin D is essential for preventing rickets, a condition that causes bone deformities and results from a deficiency of vitamin D, calcium, and phosphorus.

3. A young student is brought to the school nurse after falling off a swing. The nurse is documenting that the child has bruising on the lateral aspect of the right arm. What term will the nurse use to describe bruising on the skin in documentation? A) Telangiectasias B) Ecchymoses C) Purpura D) Urticaria

B) Ecchymoses Ans: B Feedback: Telangiectasias consists of red marks on the skin caused by stretching of superficial blood vessels. Ecchymoses are bruises, and purpura consists of pinpoint hemorrhages into the skin. Urticariais wheals or hives.

10. A patient is diagnosed with atrial fibrillation and the physician orders Coumadin (warfarin). For what skin lesion should the nurse monitor this patient? A) Ulcer B) Ecchymosis C) Scar D) Erosion

B) Ecchymosis Ans: B Feedback: Ecchymosis refers to a round or irregular macular lesion, which is larger than petechiae. This occurs secondary to blood extravasation. It is important to watch for ecchymosis in a patient receiving any type of anticoagulant. An ulcer is an open lesion eroded into the patient's flesh. A scar is an area on the skin caused by the healing of an injury. Erosion is loss of superficial epidermis that does not extend to the dermisa depressed, moist area.

25. When caring for a patient with toxic epidermal necrolysis (TEN), the critical care nurse assesses frequently for high fever, tachycardia, and extreme weakness and fatigue. The nurse is aware that these findings are potential indicators of what? Select all that apply. A) Possible malignancy B) Epidermal necrosis C) Neurologic involvement D) Increased metabolic needs E) Possible gastrointestinal mucosal sloughing .

B) Epidermal necrosis D) Increased metabolic needs E) Possible gastrointestinal mucosal sloughing Ans: B, D, E Feedback: Assessment for high fever, tachycardia, and extreme weakness and fatigue is essential because these factors indicate the process of epidermal necrosis, increased metabolic needs, and possible gastrointestinal and respiratory mucosal sloughing. These factors are less likely to suggest malignancy or neurologic involvement, as these are not common complications of TEN

18. A patient has received a diagnosis of irritant contact dermatitis. What action should the nurse prioritize in the patient's subsequent care? A) Teaching the patient to safely and effectively administer immunosuppressants B) Helping the patient identify and avoid the offending agent C) Teaching the patient how to maintain meticulous skin hygiene D) Helping the patient perform wound care in the home environment

B) Helping the patient identify and avoid the offending agent Ans: B Feedback: A focus of care for patients with irritant contact dermatitis is identifying and avoiding the offending agent. Immunosuppressants are not used to treat eczema and wound care is not normally required, except in cases of open lesions. Poor hygiene has no correlation with contact dermatitis.

21. A nurse who is taking care of a patient with burns is asked by a family member why the patient is losing so much weight. The patient is currently in the intermediate phase of recovery. What would be the nurse's most appropriate response to the family member? A) He's on a calorie-restricted diet in order to divert energy to wound healing. B) His body has consumed his fat deposits for fuel because his calorie intake is lower than normal. C) He actually hasn't lost weight. Instead, there's been a change in the distribution of his body fat. D) He lost many fluids while he was being treated in the emergency phase of burn care.

B) His body has consumed his fat deposits for fuel because his calorie intake is lower than normal. Ans: B Feedback: Patients lose a great deal of weight during recovery from severe burns. Reserve fat deposits are catabolized as a result of hypermetabolism. Patients are not placed on a calorie restriction during recovery and fluid losses would not account for weight loss later in the recovery period. Changes in the overall distribution of body fat do not occur.

20. A patient's burns have required a homograft. During the nurse's most recent assessment, the nurse observes that the graft is newly covered with purulent exudate. What is the nurse's most appropriate response? A) Perform mechanical débridement to remove the exudate and prevent further infection. B) Inform the primary care provider promptly because the graft may need to be removed. C) Perform range of motion exercises to increase perfusion to the graft site and facilitate healing. D) Document this finding as an expected phase of graft healing.

B) Inform the primary care provider promptly because the graft may need to be removed. Ans: B Feedback: An infected graft may need to be removed, thus the care provider should be promptly informed. ROM exercises will not resolve this problem and the nurse would not independently perform débridement.

35. A patient comes to the dermatology clinic requesting the removal of a port-wine stain on his right cheek. The nurse knows that the procedure especially useful in treating cutaneous vascular lesions such as port-wine stains is what? A) Skin graft B) Laser treatment C) Chemical face peeling D) Free flap

B) Laser treatment Ans: B Feedback: Argon lasers are useful in treating cutaneous vascular lesions such as port-wine stains. Skin grafts, chemical face peels, and free flaps would not be used to remove a port-wine stain.

4. A patient has experienced an electrical burn and has developed thick eschar over the burn site. Which of the following topical antibacterial agents will the nurse expect the physician to order for the wound? A) Silver sulfadiazine 1% (Silvadene) water-soluble cream B) Mafenide acetate 10% (Sulfamylon) hydrophilic-based cream C) Silver nitrate 0.5% aqueous solution D) Acticoat

B) Mafenide acetate 10% (Sulfamylon) hydrophilic-based cream Ans: B Feedback: Mafenide acetate 10% hydrophilic-based cream is the agent of choice when there is a need to penetrate thick eschar. Silver products do not penetrate eschar; Acticoat is a type of silver dressing.

38. A patient is suspected of developing an allergy to an environmental substance and has been given a patch test. During the test, the patient develops fine blisters, papules, and severe itching. The nurse knows that this is indicative of what strength reaction? A) Weak positive B) Moderately positive C) Strong positive D) Severely positive

B) Moderately positive Ans: B Feedback: The development of redness, fine elevations, or itching is considered a weak positive reaction; fine blisters, papules, and severe itching indicate a moderately positive reaction; and blisters, pain, and ulceration indicate a strong positive reaction.

1. A patient with possible bacterial meningitis is admitted to the ICU. What assessment finding would the nurse expect for a patient with this diagnosis? A) Pain upon ankle dorsiflexion of the foot B) Neck flexion produces flexion of knees and hips C) Inability to stand with eyes closed and arms extended without swaying D) Numbness and tingling in the lower extremities

B) Neck flexion produces flexion of knees and hips Clinical manifestations of bacterial meningitis include a positive Brudzinski's sign. Neck flexion producing flexion of knees and hips correlates with a positive Brudzinski's sign. Positive Homan's sign (pain upon dorsiflexion of the foot) and negative Romberg's sign (inability to stand with eyes closed and arms extended) are not expected assessment findings for the patient with bacterial meningitis. Peripheral neuropathy manifests as numbness and tingling in the lower extremities. Again, this would not be an initial assessment to rule out bacterial meningitis.

20. A school nurse has sent home four children who show evidence of pediculosis capitis. What is an important instruction the nurse should include in the note being sent home to parents? A) The child's scalp should be monitored for 48 to 72 hours before starting treatment. B) Nits may have to be manually removed from the child's hair shafts. C) The disease is self-limiting and symptoms will abate within 1 week. D) Efforts should be made to improve the child's level of hygiene.

B) Nits may have to be manually removed from the child's hair shafts. Ans: B Feedback: Treatment for head lice should begin promptly and may require manual removal of nits following medicating shampoo. Head lice are not related to a lack of hygiene. Treatment is necessary because the condition will not likely resolve spontaneously within 1 week.

39. A home care nurse is performing a visit to a patient's home to perform wound care following the patient's hospital treatment for severe burns. While interacting with the patient, the nurse should assess for evidence of what complication? A) Psychosis B) Post-traumatic stress disorder C) Delirium D) Vascular dementia

B) Post-traumatic stress disorder Ans: B Feedback: Post-traumatic stress disorder (PTSD) is the most common psychiatric disorder in burn survivors, with a prevalence that may be as high as 45%. As a result, it is important for the nurse to assess for this complication of burn injuries. Psychosis, delirium, and dementia are not among the noted psychiatric and psychosocial complications of burns.

14. A patient with a chronic diabetic wound is being discharged after receiving a skin graft to aid wound healing. What direction should the nurse include in home care instructions? A) Gently massage the graft site daily to promote perfusion. B) Protect the graft from direct sunlight and temperature extremes. C) Protect the graft site from any form of moisture for at least 12 weeks. D) Apply antibiotic ointment to the graft site and donor site daily.

B) Protect the graft from direct sunlight and temperature extremes. Ans: B Feedback: Both the donor site and the grafted area must be protected from exposure to extremes in temperature, external trauma, and sunlight because these areas are sensitive, especially to thermal injuries. Antibiotic ointments are not typically prescribed and massage may damage these fragile sites. There is no need to protect the sites from all forms of moisture for the long term.

A patient who was burned in a workplace accident has completed the acute phase of treatment and the plan of care has been altered to prioritize rehabilitation. What nursing action should be prioritized during this phase of treatment? A) Monitoring fluid and electrolyte imbalances B) Providing education to the patient and family C) Treating infection D) Promoting thermoregulation

B) Providing education to the patient and family Ans: B Feedback: Patient and family education is a priority during rehabilitation. There should be no fluid and electrolyte imbalances in the rehabilitation phase. The presence of impaired thermoregulation or infection would suggest that the patient is still in the acute phase of burn recovery.

10. The nurse is preparing the patient for mechanical débridement and informs the patient that this will involve which of the following procedures? A) A spontaneous separation of dead tissue from the viable tissue B) Removal of eschar until the point of pain and bleeding occurs C) Shaving of burned skin layers until bleeding, viable tissue is revealed D) Early closure of the wound

B) Removal of eschar until the point of pain and bleeding occurs Ans: B Feedback: Mechanical débridementcan be achieved through the use of surgical scissors, scalpels, or forceps to remove the eschar until the point of pain and bleeding occurs. Mechanical débridement can also be accomplished through the use of topical enzymatic débridement agents. The spontaneous separation of dead tissue from the viable tissue is an example of natural débridement. Shaving the burned skin layers and early wound closure are examples of surgical débridement.

12. A patient with a suspected malignant melanoma is referred to the dermatology clinic. The nurse knows to facilitate what diagnostic test to rule out a skin malignancy? A) Tzanck smear B) Skin biopsy C) Patch testing D) Skin scrapings

B) Skin biopsy Ans: B Feedback: A skin biopsy is done to rule out malignancies of skin lesions. A Tzanck smear is used to examine cells from blistering skin conditions, such as herpes zoster. Patch testing is performed to identify substances to which the patient has developed an allergy. Skin scrapings are done for suspected fungal infections.

19. A wound care nurse is reviewing skin anatomy with a group of medical nurses. Which area of the skin would the nurse identify as providing a cushion between the skin layers, muscles, and bones? A) Dermis B) Subcutaneous tissue C) Epidermis D) Stratum corneum

B) Subcutaneous tissue Ans: B Feedback: The subcutaneous tissue, or hypodermis, is the innermost layer of the skin that is responsible for providing a cushion between the skin layers, muscles, and bones. The dermis is the largest portion of the skin, providing strength and structure. The epidermis is the outermost layer of stratified epithelial cells and composed of keratinocytes. The stratum corneum is the outermost layer of the epidermis, which provides a barrier to prevent epidermal water loss.

28. A nurse is caring for a patient whose skin cancer will soon be removed by excision. Which of the following actions should the nurse perform? A) Teach the patient about early signs of secondary blistering diseases. B) Teach the patient about self-care after treatment. C) Assess the patient's risk for recurrent malignancy. D) Assess the patient for adverse effects of radiotherapy.

B) Teach the patient about self-care after treatment. Ans: B Feedback: Because many skin cancers are removed by excision, patients are usually treated in outpatient surgical units. The role of the nurse is to teach the patient about prevention of skin cancer and about self-care after treatment. Assessing the patient's risk for recurrent malignancy is primarily the role of the physician. Blistering diseases do not result from cancer or subsequent excision. Excision is not accompanied by radiotherapy.

35. Assessment of a patient's leg reveals the presence of a 1.5-cm circular region of necrotic tissue that is deeper than the epidermis. The nurse should document the presence of what type of skin lesion? A) Keloid B) Ulcer C) Fissure D) Erosion

B) Ulcer Ans: B Feedback: An ulcer is skin loss extending past the epidermis with the involvement of necrotic tissue. Keloids lack necrosis and consist of scar tissue. A fissure is linear and erosions do not extend to the dermis.

39. A dermatologist has asked the nurse to assist with examination of a patient's skin using a Wood's light. This test will allow the physician to assess for which of the following? A) The presence of minute regions of keloid scarring B) Unusual patterns of pigmentation on the patient's skin C) Vascular lesions that are not visible to the naked eye D) The presence of parasites on the epidermis

B) Unusual patterns of pigmentation on the patient's skin Ans: B Feedback: Wood's light makes it possible to differentiate epidermal from dermal lesions and hypopigmented and hyperpigmented lesions from normal skin.

37. The nurse is caring for a patient at risk for an addisonian crisis. For what associated signs and symptoms should the nurse monitor the patient? Select all that apply. A) Epistaxis B) Pallor C) Rapid respiratory rate D) Bounding pulse E) Hypotension

B, C, E Feedback: The patient at risk is monitored for signs and symptoms indicative of addisonian crisis, which can include shock; hypotension; rapid, weak pulse; rapid respiratory rate; pallor; and extreme weakness. Epistaxis and a bounding pulse are not symptoms or signs of an addisonian crisis.

25. A patient has been admitted to the critical care unit with a diagnosis of thyroid storm. What interventions should the nurse include in this patients immediate care? Select all that apply. A) Administering diuretics to prevent fluid overload B) Administering beta blockers to reduce heart rate C) Administering insulin to reduce blood glucose levels D) Applying interventions to reduce the patients temperature E) Administering corticosteroids

B, D Feedback: Thyroid storm necessitates interventions to reduce heart rate and temperature. Diuretics, insulin, and steroids are not indicated to address the manifestations of this health problem.

As a member of the stroke team, the nurse knows that thrombolytic therapy carries the potential for benefit and for harm. The nurse should be cognizant of what contraindications for thrombolytic therapy? Select all that apply. A) INR above 1.0 B) Recent intracranial pathology C) Sudden symptom onset D) Current anticoagulation therapy E) Symptom onset greater than 3 hours prior to admission

B, D, E (Feedback: Some of the absolute contraindications for thrombolytic therapy include symptom onset greater than 3 hours before admission, a patient who is anticoagulated (with an INR above 1.7), or a patient who has recently had any type of intracranial pathology (e.g., previous stroke, head injury, trauma).)

A patient has been admitted to the critical care unit with a diagnosis of toxic hepatitis. When planning the patient's care, the nurse should be aware of what potential clinical course of this health problem? Place the following events in the correct sequence. 1. Fever rises 2. Hematemesis 3. Clotting abnormalities 4. Vascular collapse 5. Coma A. 1,2,5,4,3 B. 1,2,3,4,5 C. 2,3,1,4,5 D. 3,1,2,5,4

B. 1,2,3,4,5 Rationale: Recovery from acute toxic hepatitis is rapid if the hepatotoxin is identified early and removed or if exposure to the agent has been limited. Recovery is unlikely if there is a prolonged period between exposure and onset of symptoms. There are no effective antidotes. The fever rises; the patient becomes toxic and prostrated. Vomiting may be persistent, with the emesis containing blood. Clotting abnormalities may be severe, and hemorrhages may appear under the skin. The severe GI symptoms may lead to vascular collapse. Delirium, coma, and seizures develop, and within a few days the patient may die of fulminant hepatic failure unless he or she receives a liver transplant.

A patient with a history of injection drug use has been diagnosed with Hepatitis C. When collaborating with the care team to plan this patient's treatment, the nurse should anticipate what intervention? A. Administration of immune globulins B. A regimen of antiviral medications C. Rest and watchful waiting D. Administration of fresh-frozen plasma (FFP)

B. A regimen of antiviral medications Rationale: There is no benefit from rest, diet, or vitamin supplements in HCV treatment. Studies have demonstrated that a combination of two antiviral agents, Peg-interferon and ribavirin (Rebetol), is effective in producing improvement in patients with hepatitis C and in treating relapses. Immune globulins and FFP are not indicated.

A patient has been diagnosed with advanced stage breast cancer and will soon begin aggressive treatment. What assessment findings would most strongly suggest that the patient may have developed liver metastases? A. Persistent fever and cognitive changes B. Abdominal pain and hepatomegaly C. Peripheral edema unresponsive to diuresis D. Spontaneous bleeding and jaundice

B. Abdominal pain and hepatomegaly Rationale: The early manifestations of malignancy of the liver include pain, a continuous dull ache in the right upper quadrant, epigastrium, or back. Weight loss, loss of strength, anorexia, and anemia may also occur. The liver may be enlarged and irregular on palpation. Jaundice is present only if the larger bile ducts are occluded by the pressure of malignant nodules in the hilum of the liver. Fever, cognitive changes, peripheral edema, and bleeding are atypical signs.

A participant in a health fair has asked the nurse about the role of drugs in liver disease. What health promotion teaching has the most potential to prevent drug-induced hepatitis? A. Finish all prescribed courses of antibiotics, regardless of symptoms resolution. B. Adhere to dosing recommendations of OTC analgesics. C. Ensure that expired medications are disposed of safely. D. Ensure that pharmacists regularly review drug regimens for potential interactions.

B. Adhere to dosing recommendations of OTC analgesics. Rationale: Although any medication can affect liver function, use of acetaminophen (found in many OTC medications used to treat fever and pain) has been identified as the leading cause of acute liver failure. Finishing prescribed antibiotics and avoiding expired medications are unrelated to this disease. Drug interactions are rarely the cause of drug-induced hepatitis.

A nurse is caring for a patient who has been admitted for the treatment of advanced cirrhosis. What assessment should the nurse prioritize in this patient's plan of care? A. Measurement of abdominal girth and body weight B. Assessment for variceal bleeding C. Assessment for signs and symptoms of jaundice D. Monitoring of results of liver function testing

B. Assessment of variceal bleeding Rationale: Esophageal varices are a major cause of mortality in patients with uncompensated cirrhosis. Consequently, this should be a focus of the nurse's assessments and should be prioritized over the other listed assessments, even though each should be performed.

A patient with cirrhosis has experienced a progressive decline in his health; and liver transplantation is being considered by the interdisciplinary team. How will the patient's prioritization for receiving a donor liver be determined? A. By considering that patient's age and prognosis B. By objectively determining the patient's medical need C. By objectively assessing the patient's willingness to adhere to post-transplantation care D. By systematically ruling out alternative treatment options

B. By objectively determining the patient's medical need Rationale: The patient would undergo a classification of the degree of medical need through an objective determination known as the Model of End-Stage Liver Disease (MELD) classification, which stratifies the level of illness of those awaiting a liver transplant. This algorithm considers multiple variables, not solely age, prognosis, potential for adherence and the rejection of alternative options.

A patient with a diagnosis of esophageal varices has undergone endoscopy to gauge the progression of this complication of liver disease. Following the completion of this diagnostic test, what nursing intervention should the nurse perform? A. Keep patient NPO until the results of test are known B. Keep patient NPO until the patient's gag reflex returns C. Administer analgesia until post-procedure tenderness is relieved D. Give the patient a cold beverage to promote swallowing ability

B. Keep patient NPO until the patient's gag reflex returns Rationale: After the examination, fluids are not given until the patient's gag reflex returns. Lozenges and gargles may be used to relieve throat discomfort if the patient's physical condition and mental status permit. The result of the test is known immediately. Food and fluids are contraindicated until the gag reflex returns.

A nurse is caring for a patient with a blocked bile duct from a tumor. What manifestation of obstructive jaundice should the nurse anticipate? A. Watery, blood-streaked diarrhea B. Orange and foamy urine C. Increased abdominal girth D. Decreased cognition

B. Orange and foamy urine Rationale: If the bile duct is obstructed, the bile will be reabsorbed into the blood and carried throughout the entire body. It is excreted in the urine, which becomes deep orange and foamy. Bloody diarrhea, ascites, and cognitive changes are not associated with obstructive jaundice.

A nurse is caring for a patient with cirrhosis secondary to heavy alcohol use. The nurse's most recent assessment reveals subtle changes in the patient's cognition and behavior. What is the nurse's most appropriate response? A. Ensure that the patient's sodium intake does not exceed recommended levels B. Report this finding to the primary care provider due to the possibility of hepatic encephalopathy C. Inform the primary care provider that the patient should be assessed for alcoholic hepatitis. D. Implement interventions aimed at ensuring a calm and therapeutic care environment.

B. Report this finding to the primary care provider due to the possibility of hepatic encephalopathy Rationale: Monitoring is an essential nursing function to identify early deterioration in mental status. The nurse monitors the patient's mental status closely and reports changes so that treatment of encephalopathy can be initiated promptly. This change in status is likely unrelated to sodium intake and would not signal the onset of hepatitis. A supportive care environment is beneficial, but does not address the patient's physiologic deterioration.

A patient has had an ischemic stroke and has been admitted to the medical unit. What action should the nurse perform to best prevent joint deformities? A) Place the patient in the prone position for 30 minutes/day. B) Assist the patient in acutely flexing the thigh to promote movement. C) Place a pillow in the axilla when there is limited external rotation. D) Place patient's hand in pronation.

C (Feedback: A pillow in the axilla prevents adduction of the affected shoulder and keeps the arm away from the chest. The prone position with a pillow under the pelvis, not flat, promotes hyperextension of the hip joints, essential for normal gait. To promote venous return and prevent edema, the upper thigh should not be flexed acutely. The hand is placed in slight supination, not pronation, which is its most functional position.)

A community health nurse is giving an educational presentation about stroke and heart disease at the local senior citizens center. What nonmodifiable risk factor for stroke should the nurse cite? A) Female gender B) Asian American race C) Advanced age D) Smoking

C (Feedback: Advanced age, male gender, and race are well-known nonmodifiable risk factors for stroke. High-risk groups include people older than 55 years of age; the incidence of stroke more than doubles in each successive decade. Men have a higher rate of stroke than that of women. Another high-risk group is African Americans; the incidence of first stroke in African Americans is almost twice that as in Caucasian Americans; Asian American race is not a risk factor. Smoking is a modifiable risk.)

A patient with a new diagnosis of ischemic stroke is deemed to be a candidate for treatment with tissue plasminogen activator (t-PA) and has been admitted to the ICU. In addition to closely monitoring the patient's cardiac and neurologic status, the nurse monitors the patient for signs of what complication? A) Acute pain B) Septicemia C) Bleeding D) Seizures

C (Feedback: Bleeding is the most common side effect of t-PA administration, and the patient is closely monitored for any bleeding. Septicemia, pain, and seizures are much less likely to result from thrombolytic therapy.)

The nurse is discharging home a patient who suffered a stroke. He has a flaccid right arm and leg and is experiencing problems with urinary incontinence. The nurse makes a referral to a home health nurse because of an awareness of what common patient response to a change in body image? A) Denial B) Fear C) Depression D) Disassociation

C (Feedback: Depression is a common and serious problem in the patient who has had a stroke. It can result from a profound disruption in his or her life and changes in total function, leaving the patient with a loss of independence. The nurse needs to encourage the patient to verbalize feelings to assess the effect of the stroke on self-esteem. Denial, fear, and disassociation are not the most common patient response to a change in body image, although each can occur in some patients.)

A patient diagnosed with a hemorrhagic stroke has been admitted to the neurologic ICU. The nurse knows that teaching for the patient and family needs to begin as soon as the patient is settled on the unit and will continue until the patient is discharged. What will family education need to include? A) How to differentiate between hemorrhagic and ischemic stroke B) Risk factors for ischemic stroke C) How to correctly modify the home environment D) Techniques for adjusting the patient's medication dosages at home

C (Feedback: For a patient with a hemorrhagic stroke, teaching addresses the use of assistive devices or modification of the home environment to help the patient live with the disability. This is more important to the patient's needs than knowing about risk factors for ischemic stroke. It is not necessary for the family to differentiate between different types of strokes. Medication regimens should never be altered without consultation.)

The nurse is caring for a patient diagnosed with an ischemic stroke and knows that effective positioning of the patient is important. Which of the following should be integrated into the patient's plan of care? A) The patient's hip joint should be maintained in a flexed position. B) The patient should be in a supine position unless ambulating. C) The patient should be placed in a prone position for 15 to 30 minutes several times a day. D) The patient should be placed in a Trendelenberg position two to three times daily to promote cerebral perfusion.

C (Feedback: If possible, the patient is placed in a prone position for 15 to 30 minutes several times a day. A small pillow or a support is placed under the pelvis, extending from the level of the umbilicus to the upper third of the thigh. This helps to promote hyperextension of the hip joints, which is essential for normal gait, and helps prevent knee and hip flexion contractures. The hip joints should not be maintained in flexion and the Trendelenberg position is not indicated.)

The nurse is reviewing the medication administration record of a female patient who possesses numerous risk factors for stroke. Which of the woman's medications carries the greatest potential for reducing her risk of stroke? A) Naproxen 250 PO b.i.d. B) Calcium carbonate 1,000 mg PO b.i.d. C) Aspirin 81 mg PO o.d. D) Lorazepam 1 mg SL b.i.d. PRN

C (Feedback: Research findings suggest that low-dose aspirin may lower the risk of stroke in women who are at risk. Naproxen, lorazepam, and calcium supplements do not have this effect.)

The pathophysiology of an ischemic stroke involves the ischemic cascade, which includes the following steps: 1. Change in pH 2. Blood flow decreases 3. A switch to anaerobic respiration 4. Membrane pumps fail 5. Cells cease to function 6. Lactic acid is generated Put these steps in order in which they occur. A) 635241 B) 352416 C) 236145 D) 162534

C (Feedback: The ischemic cascade begins when cerebral blood flow decreases to less than 25 mL per 100 g of blood per minute. At this point, neurons are no longer able to maintain aerobic respiration. The mitochondria must then switch to anaerobic respiration, which generates large amounts of lactic acid, causing a change in the pH. This switch to the less efficient anaerobic respiration also renders the neuron incapable of producing sufficient quantities of adenosine triphosphate (ATP) to fuel the depolarization processes. The membrane pumps that maintain electrolyte balances begin to fail, and the cells cease to function.)

A patient diagnosed with transient ischemic attacks (TIAs) is scheduled for a carotid endarterectomy. The nurse explains that this procedure will be done for what purpose? A) To decrease cerebral edema B) To prevent seizure activity that is common following a TIA C) To remove atherosclerotic plaques blocking cerebral flow D) To determine the cause of the TIA

C (Feedback: The main surgical procedure for select patients with TIAs is carotid endarterectomy, the removal of an atherosclerotic plaque or thrombus from the carotid artery to prevent stroke in patients with occlusive disease of the extracranial arteries. An endarterectomy does not decrease cerebral edema, prevent seizure activity, or determine the cause of a TIA.)

The nurse is preparing health education for a patient who is being discharged after hospitalization for a hemorrhagic stroke. What content should the nurse include in this education? A) Mild, intermittent seizures can be expected. B) Take ibuprofen for complaints of a serious headache. C) Take antihypertensive medication as ordered. D) Drowsiness is normal for the first week after discharge.

C (Feedback: The patient and family are provided with information that will enable them to cooperate with the care and restrictions required during the acute phase of hemorrhagic stroke and to prepare the patient to return home. Patient and family teaching includes information about the causes of hemorrhagic stroke and its possible consequences. Symptoms of hydrocephalus include gradual onset of drowsiness and behavioral changes. Hypertension is the most serious risk factor, suggesting that appropriate antihypertensive treatment is essential for a patient being discharged. Seizure activity is not normal; complaints of a serious headache should be reported to the physician before any medication is taken. Drowsiness is not normal or expected.)

1. The ED nurse is caring for a patient who has been brought in by ambulance after sustaining a fall at home. What physical assessment finding is suggestive of a basilar skull fracture? A) Epistaxis B) Periorbital edema C) Bruising over the mastoid D) Unilateral facial numbness

C Feedback: An area of ecchymosis (bruising) may be seen over the mastoid (Battle's sign) in a basilar skull fracture. Numbness, edema, and epistaxis are not directly associated with a basilar skull fracture.

20. The nurse is assessing a patient diagnosed with Graves disease. What physical characteristics of Graves disease would the nurse expect to find? A) Hair loss B) Moon face C) Bulging eyes D) Fatigue

C Feedback: Clinical manifestations of the endocrine disorder Graves disease include exophthalmos (bulging eyes) and fine tremor in the hands. Graves disease is not associated with hair loss, a moon face, or fatigue.

29. A 30 year-old female patient has been diagnosed with Cushing syndrome. What psychosocial nursing diagnosis should the nurse most likely prioritize when planning the patients care? A) Decisional conflict related to treatment options B) Spiritual distress related to changes in cognitive function C) Disturbed body image related to changes in physical appearance D) Powerlessness related to disease progression

C Feedback: Cushing syndrome causes characteristic physical changes that are likely to result in disturbed body image. Decisional conflict and powerless may exist, but disturbed body image is more likely to be present. Cognitive changes take place in patients with Cushing syndrome, but these may or may not cause spiritual distress.

9. The nurse caring for a patient with Cushing syndrome is describing the dexamethasone suppression test scheduled for tomorrow. What does the nurse explain that this test will involve? A) Administration of dexamethasone orally, followed by a plasma cortisol level every hour for 3 hours B) Administration of dexamethasone IV, followed by an x-ray of the adrenal glands C) Administration of dexamethasone orally at 11 PM, and a plasma cortisol level at 8 AM the next morning D) Administration of dexamethasone intravenously, followed by a plasma cortisol level 3 hours after the drug is administered

C Feedback: Dexamethasone (1 mg) is administered orally at 11 PM, and a plasma cortisol level is obtained at 8 AM the next morning. This test can be performed on an outpatient basis and is the most widely used and sensitive screening test for diagnosis of pituitary and adrenal causes of Cushing syndrome.

16. A patient has returned to the floor after having a thyroidectomy for thyroid cancer. The nurse knows that sometimes during thyroid surgery the parathyroid glands can be injured or removed. What laboratory finding may be an early indication of parathyroid gland injury or removal? A) Hyponatremia B) Hypophosphatemia C) Hypocalcemia D) Hypokalemia

C Feedback: Injury or removal of the parathyroid glands may produce a disturbance in calcium metabolism and result in a decline of calcium levels (hypocalcemia). As the blood calcium levels fall, hyperirritability of the nerves occurs, with spasms of the hands and feet and muscle twitching. This group of symptoms is known as tetany and must be reported to the physician immediately, because laryngospasm may occur and obstruct the airway. Hypophosphatemia, hyponatremia, and hypokalemia are not expected responses to parathyroid injury or removal. In fact, parathyroid removal or injury that results in hypocalcemia may lead to hyperphosphatemia.

4. The nurse is caring for a patient with a diagnosis of Addisons disease. What sign or symptom is most closely associated with this health problem? A) Truncal obesity B) Hypertension C) Muscle weakness D) Moon face

C Feedback: Patients with Addisons disease demonstrate muscular weakness, anorexia, gastrointestinal symptoms, fatigue, emaciation, dark pigmentation of the skin, and hypotension. Patients with Cushing syndrome demonstrate truncal obesity, moon face, acne, abdominal striae, and hypertension.

8. A nurse caring for a patient with diabetes insipidus is reviewing laboratory results. What is an expected urinalysis finding? A) Glucose in the urine B) Albumin in the urine C) Highly dilute urine D) Leukocytes in the urine

C Feedback: Patients with diabetes insipidus produce an enormous daily output of very dilute, water-like urine with a specific gravity of 1.001 to 1.005. The urine contains no abnormal substances such as glucose or albumin. Leukocytes in the urine are not related to the condition of diabetes insipidus, but would indicate a urinary tract infection, if present in the urine.

6. The nurse is teaching a patient that the body needs iodine for the thyroid to function. What food would be the best source of iodine for the body? A) Eggs B) Shellfish C) Table salt D) Red meat

C Feedback: The major use of iodine in the body is by the thyroid. Iodized table salt is the best source of iodine.

7. A patient is prescribed corticosteroid therapy. What would be priority information for the nurse to give the patient who is prescribed long-term corticosteroid therapy? A) The patients diet should be low protein with ample fat. B) The patient may experience short-term changes in cognition. C) The patient is at an increased risk for developing infection. D) The patient is at a decreased risk for development of thrombophlebitis and thromboembolism.

C Feedback: The patient is at increased risk of infection and masking of signs of infection. The cardiovascular effects of corticosteroid therapy may result in development of thrombophlebitis or thromboembolism. Diet should be high in protein with limited fat. Changes in appearance usually disappear when therapy is no longer necessary. Cognitive changes are not common adverse effects.

2. A patient has been admitted to the post-surgical unit following a thyroidectomy. To promote comfort and safety, how should the nurse best position the patient? A) Side-lying (lateral) with one pillow under the head B) Head of the bed elevated 30 degrees and no pillows placed under the head C) Semi-Fowlers with the head supported on two pillows D) Supine, with a small roll supporting the neck

C Feedback: When moving and turning the patient, the nurse carefully supports the patients head and avoids tension on the sutures. The most comfortable position is the semi-Fowlers position, with the head elevated and supported by pillows.

21. While assessing a 25-year-old female, the nurse notes that the patient has hair on her lower abdomen. Earlier in the health interview, the patient stated that her menses are irregular. The nurse should suspect what type of health problem? A) A metabolic disorder B) A malignancy C) A hormonal imbalance D) An infectious process

C) A hormonal imbalance Ans: C Feedback: Some women with higher levels of testosterone have hair in the areas generally thought of as masculine, such as the face, chest, and lower abdomen. This is often a normal genetic variation, but if it appears along with irregular menses and weight changes, it may indicate a hormonal imbalance. This combination of irregular menses and hair distribution is inconsistent with metabolic disorders, malignancy, or infection.

2. The current phase of a patient's treatment for a burn injury prioritizes wound care, nutritional support, and prevention of complications such as infection. Based on these care priorities, the patient is in what phase of burn care? A) Emergent B) Immediate resuscitative C) Acute D) Rehabilitation

C) Acute Ans: C Feedback: The acute or intermediate phase of burn care follows the emergent/resuscitative phase and begins 48 to 72 hours after the burn injury. During this phase, attention is directed toward continued assessment and maintenance of respiratory and circulatory status, fluid and electrolyte balance, and gastrointestinal function. Infection prevention, burn wound care (i.e., wound cleaning, topical antibacterial therapy, wound dressing, dressing changes, wound débridement, and wound grafting), pain management, and nutritional support are priorities at this stage. Priorities during the emergent or immediate resuscitative phase include first aid, prevention of shock and respiratory distress, detection and treatment of concomitant injuries, and initial wound assessment and care. The priorities during the rehabilitation phase include prevention of scars and contractures, rehabilitation, functional and cosmetic reconstruction, and psychosocial counseling.

9. A patient requires a full-thickness graft to cover a chronic wound. How is the donor site selected? A) The largest area of the body without hair is selected. B) Any area that is not normally visible can be used. C) An area matching the color and texture of the skin at the surgical site is selected. D) An area matching the sensory capability of the skin at the surgical site is selected.

C) An area matching the color and texture of the skin at the surgical site is selected. Ans: C Feedback: The site where the intact skin is harvested is called the donor site. Selection of the donor site is made to match the color and texture of skin at the surgical site and to leave as little scarring as possible.

22. A nurse is caring for a patient who has a diagnosis of bullous pemphigoid and who is being treated on the medical unit. When providing hygiene for this patient, the nurse should perform which of the following actions? A) After washing, wipe lesions with sterile gauze to remove cellular debris. B) Apply antibiotic ointment to lesions after washing. C) Apply cornstarch to the patient's skin after bathing to facilitate mobility. D) Avoid using water to cleanse the patient's skin in order to maintain skin integrity.

C) Apply cornstarch to the patient's skin after bathing to facilitate mobility. Ans: C Feedback: After the patient's skin is bathed, it is dried carefully and dusted liberally with nonirritating powder (e.g., cornstarch), which enables the patient to move about freely in bed. Open blisters should not normally be wiped and antibiotics are not applied to wound beds in the absence of a secondary infection. Water can safely be used to provide hygiene.

31. A patient has experienced burns to his upper thighs and knees. Following the application of new wound dressings, the nurse should perform what nursing action? A) Instruct the patient to keep the wound site in a dependent position. B) Administer PRN analgesia as ordered. C) Assess the patient's peripheral pulses distal to the dressing. D) Assist with passive range of motion exercises to set the new dressing.

C) Assess the patient's peripheral pulses distal to the dressing. Ans: C Feedback: Dressings can impede circulation if they are wrapped too tightly. The peripheral pulses must be checked frequently and burned extremities elevated. Dependent positioning does not need to be maintained. PRN analgesics should be administered prior to the dressing change. ROM exercises do not normally follow a dressing change.

16. A nurse is providing care for a patient who has psoriasis. The nurse is aware of the sequelae that can result from this health problem. Following the appearance of skin lesions, the nurse should prioritize what assessment? A) Assessment of the patient's stool for evidence of intestinal sloughing B) Assessment of the patient's apical heart rate for dysrhythmias C) Assessment of the patient's joints for pain and decreased range of motion D) Assessment for cognitive changes resulting from neurologic lesions

C) Assessment of the patient's joints for pain and decreased range of motion Ans: C Feedback: Asymmetric rheumatoid factornegative arthritis of multiple joints occurs in up to 30% of people with psoriasis, most typically after the skin lesions appear. The most typical joints affected include those in the hands or feet, although sometimes larger joints such as the elbow, knees, or hips may be affected. As such, the nurse should assess for this musculoskeletal complication. GI, cardiovascular, and neurologic function are not affected by psoriasis.

38. An older adult resident of a long-term care facility has been experiencing generalized pruritus that has become more severe in recent weeks. What intervention should the nurse add to this resident's plan of care? A) Avoid the application of skin emollients. B) Apply antibiotic ointment as ordered following baths. C) Avoid using hot water during the patient's baths. D) Administer acetaminophen 4 times daily as ordered.

C) Avoid using hot water during the patient's baths. Ans: C Feedback: If baths have been prescribed, the patient is reminded to use tepid (not hot) water and to shake off the excess water and blot between intertriginous areas (body folds) with a towel. Skin emollients should be applied to reduce pruritus. Acetaminophen and antibiotics do not reduce pruritus.

32. The nurse is performing a comprehensive assessment of a patient's skin surfaces and intends to assess moisture, temperature, and texture. The nurse should perform this component of assessment in what way? A) By examining the patient under a Wood's light B) By inspecting the patient's skin in direct sunlight C) By palpating the patient's skin D) By performing percussion of major skin surfaces

C) By palpating the patient's skin Ans: C Feedback: Inspection and palpation are techniques commonly used in examining the skin. A patient would only be examined under a Wood's light if there were indications it could be diagnostic. The patient is examined in a well-lit room, not in direct sunlight. Percussion is not a technique used in assessing the skin.

15. A nurse is teaching a patient with a partial-thickness wound how to wear his elastic pressure garment. How would the nurse instruct the patient to wear this garment? A) 4 to 6 hours a day for 6 months B) During waking hours for 2 to 3 months after the injury C) Continuously D) At night while sleeping for a year after the injury

C) Continuously Ans: C Feedback: Elastic pressure garments are worn continuously (i.e., 23 hours a day).

24. A patient with severe burns is admitted to the intensive care unit to stabilize and begin fluid resuscitation before transport to the burn center. The nurse should monitor the patient closely for what signs of the onset of burn shock? A) Confusion B) High fever C) Decreased blood pressure D) Sudden agitation

C) Decreased blood pressure Ans: C Feedback: As fluid loss continues and vascular volume decreases, cardiac output continues to decrease and the blood pressure drops, marking the onset of burn shock. Shock and the accompanying hemodynamic changes are not normally accompanied by confusion, fever, or agitation.

7. While performing an initial assessment of a patient admitted with appendicitis, the nurse observes an elevated blue-black lesion on the patient's ear. The nurse knows that this lesion is consistent with what type of skin cancer? A) Basal cell carcinoma B) Squamous cell carcinoma C) Dermatofibroma D) Malignant melanoma

C) Dermatofibroma Ans: D Feedback: A malignant melanoma presents itself as a superficial spreading melanoma which may appear in a combination of colors, with hues of tan, brown, and black mixed with gray, blue-black, or white. The lesion tends to be circular, with irregular outer portions. BCC usually begins as a small, waxy nodule with rolled, translucent, pearly borders; telangiectatic vessels may be present. SCC appears as a rough, thickened, scaly tumor that may be asymptomatic or may involve bleeding. A dermatofibroma presents as a firm, dome-shaped papule or nodule that may be skin colored or pinkish-brown.

25. A nurse is reviewing gerontologic considerations relating to the care of patients with dermatologic problems. What vulnerability results from the age-related loss of subcutaneous tissue? A) Decreased resistance to ultraviolet radiation B) Increased vulnerability to infection C) Diminished protection of tissues and organs D) Increased risk of skin malignancies

C) Diminished protection of tissues and organs Ans: C Feedback: Loss of the subcutaneous tissue substances of elastin, collagen, and fat diminishes the protection and cushioning of underlying tissues and organs, decreases muscle tone, and results in the loss of the insulating properties of fat. This age-related change does not correlate to an increased vulnerability to sun damage, infection, or cancer.

39. A patient has a diagnosis of seborrhea and has been referred to the dermatology clinic, where the nurse contributes to care. When planning this patient's care, the nurse should include which of the following nursing diagnoses? A) Risk for Deficient Fluid Volume Related to Excess Sebum Synthesis B) Ineffective Thermoregulation Related to Occlusion of Sebaceous Glands C) Disturbed Body Image Related to Excess Sebum Production D) Ineffective Tissue Perfusion Related to Occlusion of Sebaceous Glands

C) Disturbed Body Image Related to Excess Sebum Production Ans: C Feedback: Seborrhea causes highly visible manifestations that are likely to have a negative effect on the patient's body image. Seborrhea does not normally affect fluid balance, thermoregulation, or tissue perfusion.

28. The nurse is performing an initial assessment of a patient who has a raised, pruritic rash. The patient denies taking any prescription medication and denies any allergies. What would be an appropriate question to ask this patient at this time? A) Is anyone in your family allergic to anything? B) How long have you had this abrasion? C) Do you take any over-the-counter drugs or herbal preparations? D) What do you do for a living?

C) Do you take any over-the-counter drugs or herbal preparations? Ans: C Feedback: If suspicious areas are noted, the patient is questioned about nonprescription or herbal preparations that might be in use. Ascertaining a family history of allergies would not give helpful information at this time. The patient's lesion is not described as an abrasion. The patient's occupation may or may not be relevant; it is more important to assess for herb or drug reactions.

37. A public health nurse is participating in a health promotion campaign that has the goal of improving outcomes related to skin cancer in the community. What action has the greatest potential to achieve this goal? A) Educating participants about the relationship between general health and the risk of skin cancer B) Educating participants about treatment options for skin cancer C) Educating participants about the early signs and symptoms of skin cancer D) Educating participants about the health risks associated with smoking and assisting with smoking cessation

C) Educating participants about the early signs and symptoms of skin cancer Ans: C Feedback: The best hope of decreasing the incidence of skin cancer lies in educating patients about the early signs. There is a relationship between general health and skin cancer, but teaching individuals to identify the early signs and symptoms is more likely to benefit overall outcomes related to skin cancer. Teaching about treatment options is not likely to have a major effect on outcomes of the disease. Smoking is not among the major risk factors for skin cancer.

32. A 35-year-old kidney transplant patient comes to the clinic exhibiting new skin lesions. The diagnosis is Kaposi's sarcoma. The nurse caring for this patient recognizes that this is what type of Kaposi's sarcoma? A) Classic B) AIDS-related C) Immunosuppression-related D) Endemic

C) Immunosuppression-related Ans: C Feedback: Immunosuppression-associated Kaposi's sarcoma occurs in transplant recipients and people with AIDS. This form of KS is characterized by local skin lesions and disseminated visceral and mucocutaneous diseases. Classic Kaposi's sarcoma occurs predominantly in men of Mediterranean or Jewish ancestry between 40 and 70 years of age. Endemic KS affects people predominantly in the eastern half of Africa. AIDS-related KS is seen in people with AIDS.

23. A nurse is preparing to perform the physical assessment of a newly admitted patient. During which of the following components of the assessment should the nurse wear gloves? Select all that apply. A) Palpation of the patient's scalp B) Palpation of the patient's upper extremities C) Palpation of a rash on the patient's trunk D) Palpation of a lesion on the patient's upper back E) Palpation of the patient's fingers

C) Palpation of a rash on the patient's trunk D) Palpation of a lesion on the patient's upper back Ans: C, D Feedback: Gloves are worn during skin examination if a rash or lesions are to be palpated. It is not normally necessary to wear gloves to palpate a patient's scalp, extremities, or fingers unless contact with body fluids is reasonably foreseeable.

11. A new patient has come to the dermatology clinic to be assessed for a reddened rash on his abdomen. What diagnostic test would most likely be ordered to identify the causative allergen? A) Skin scrapings B) Skin biopsy C) Patch testing D) Tzanck smear

C) Patch testing Ans: C Feedback: Patch testing is performed to identify substances to which the patient has developed an allergy. Skin scrapings are done for suspected fungal lesions. A skin biopsy is completed to rule out malignancy and to establish an exact diagnosis of skin lesions. A Tzanck smear is used to examine cells from blistering skin conditions, such as herpes zoster.

12. A patient diagnosed with a stasis ulcer has been hospitalized. There is an order to change the dressing and provide wound care. Which activity should the nurse first perform when providing wound care? A) Assess the drainage in the dressing. B) Slowly remove the soiled dressing. C) Perform hand hygiene. D) Don non-latex gloves.

C) Perform hand hygiene. Ans: C Feedback: The nurse and physician must adhere to standard precautions and wear gloves when inspecting the skin or changing a dressing. Use of standard precautions and proper disposal of any contaminated dressing is carried out according to Occupational Safety and Health Administration (OSHA) regulations. Hand hygiene must precede other aspects of wound care.

36. A new patient presents at the clinic and the nurse performs a comprehensive health assessment. The nurse notes that the patient's fingernail surfaces are pitted. The nurse should suspect the presence of what health problem? A) Eczema B) Systemic lupus erythematosus (SLE) C) Psoriasis D) Chronic obstructive pulmonary disease (COPD)

C) Psoriasis Ans: C Feedback: Pitted surface of the nails is a definite indication of psoriasis. Pitting of the nails does not indicate eczema, SLE, or COPD.

16. A nurse in a dermatology clinic is reading the electronic health record of a new patient. The nurse notes that the patient has a history of a primary skin lesion. What is an example of a primary skin lesion? A) Crust B) Keloid C) Pustule D) Ulcer

C) Pustule Ans: C Feedback: A pustule is an example of a primary skin lesion. Primary skin lesions are original lesions arising from previously normal skin. Crusts, keloids and ulcers are secondary lesions.

17. A patient is admitted to the intensive care unit with what is thought to be toxic epidermal necrolysis (TEN). When assessing the health history of the patient, the nurse would be alert to what precipitating factor? A) Recent heavy ultraviolet exposure B) Substandard hygienic conditions C) Recent administration of new medications D) Recent varicella infection

C) Recent administration of new medications Ans: C Feedback: In adults, TEN is usually triggered by a reaction to medications. Antibiotics, antiseizure agents, butazones, and sulfonamides are the most frequent medications implicated. TEN is unrelated to UV exposure, hygiene, or varicella infection.

28. A patient experienced a 33% TBSA burn 72 hours ago. The nurse observes that the patient's hourly urine output has been steadily increasing over the past 24 hours. How should the nurse best respond to this finding? A) Obtain an order to reduce the rate of the patient's IV fluid infusion. B) Report the patient's early signs of acute kidney injury (AKI). C) Recognize that the patient is experiencing an expected onset of diuresis. D) Administer sodium chloride as ordered to compensate for this fluid loss.

C) Recognize that the patient is experiencing an expected onset of diuresis. Ans: C Feedback: As capillaries regain integrity, 48 or more hours after the burn, fluid moves from the interstitial to the intravascular compartment and diuresis begins. This is an expected development and does not require a reduction in the IV infusion rate or the administration of NaCl. Diuresis is not suggestive of AKI.

4. A patient with squamous cell carcinoma has been scheduled for treatment of this malignancy. The nurse should anticipate that treatment for this type of cancer will primarily consist of what intervention? A) Chemotherapy B) Radiation therapy C) Surgical excision D) Biopsy of sample tissue

C) Surgical excision Ans: C Feedback: The primary goal of surgical management of squamous cell carcinoma is to remove the tumor entirely. Radiation therapy is reserved for older patients, because x-ray changes may be seen after 5 to 10 years, and malignant changes in scars may be induced by irradiation 15 to 30 years later. Obtaining a biopsy would not be a goal of treatment; it may be an assessment. Chemotherapy and radiation therapy are generally reserved for patients who are not surgical candidates.

40. A nurse is working with a family whose 5 year-old daughter has been diagnosed with impetigo. What educational intervention should the nurse include in this family's care? A) Ensuring that the family knows that impetigo is not contagious B) Teaching about the safe and effective use of topical corticosteroids C) Teaching about the importance of maintaining high standards of hygiene D) Ensuring that the family knows how to safely burst the child's vesicles

C) Teaching about the importance of maintaining high standards of hygiene Ans: C Feedback: Impetigo is associated with unhygienic conditions; educational interventions to address this are appropriate. The disease is contagious, thus vesicles should not be manually burst. Because of the bacterial etiology, corticosteroids are ineffective.

11. A nurse is leading a health promotion workshop that is focusing on cancer prevention. What action is most likely to reduce participants' risks of basal cell carcinoma (BCC)? A) Teaching participants to improve their overall health through nutrition B) Encouraging participants to identify their family history of cancer C) Teaching participants to limit their sun exposure D) Teaching participants to control exposure to environmental and occupational radiation

C) Teaching participants to limit their sun exposure Ans: C Feedback: Sun exposure is the best known and most common cause of BCC. BCC is not commonly linked to general health debilitation, family history, or radiation exposure.

2. When planning the skin care of a patient with decreased mobility, the nurse is aware of the varying thickness of the epidermis. At what location is the epidermal layer thickest? A) The scalp B) The elbows C) The palms of the hands D) The knees

C) The palms of the hands Ans: C Feedback: The epidermis is the thickest over the palms of the hands and the soles of the feet.

19. A nurse is developing a care plan for a patient with a partial-thickness burn, and determines that an appropriate goal is to maintain position of joints in alignment. What is the best rationale for this intervention? A) To prevent neuropathies B) To prevent wound breakdown C) To prevent contractures D) To prevent heterotopic ossification

C) To prevent contractures Ans: C Feedback: To prevent the complication of contractures, the nurse will establish a goal to maintain position of joints in alignment. Gentle range of motion exercises and a consult to PT and OT for exercises and positioning recommendations are also appropriate interventions for the prevention of contractures. Joint alignment is not maintained specifically for preventing neuropathy, wound breakdown, or heterotopic ossification.

11. A patient with a partial-thickness burn injury had Biobrane applied 2 weeks ago. The nurse notices that the Biobrane is separating from the burn wound. What is the nurse's most appropriate intervention? A) Reinforce the Biobrane dressing with another piece of Biobrane. B) Remove the Biobrane dressing and apply a new dressing. C) Trim away the separated Biobrane. D) Notify the physician for further emergency-related orders.

C) Trim away the separated Biobrane. Ans: C Feedback: As the Biobrane gradually separates, it is trimmed, leaving a healed wound. When the Biobrane dressing adheres to the wound, the wound remains stable and the Biobrane can remain in place for 3 to 4 weeks. There is no need to reinforce the Biobrane nor to remove it and apply a new dressing. There is not likely any need to notify the physician for further orders.

40. A patient presents at the dermatology clinic with suspected herpes simplex. The nurse knows to prepare what diagnostic test for this condition? A) Skin biopsy B) Patch test C) Tzanck smear D) Examination with a Wood's light

C) Tzanck smear Ans: C Feedback: The Tzanck smear is a test used to examine cells from blistering skin conditions, such as herpes zoster, varicella, herpes simplex, and all forms of pemphigus. The secretions from a suspected lesion are applied to a glass slide, stained, and examined. This is not accomplished by biopsy, patch test, or Wood's light.

10. A patient has just been told that he has malignant melanoma. The nurse caring for this patient should anticipate that the patient will undergo what treatment? A) Chemotherapy B) Immunotherapy C) Wide excision D) Radiation therapy

C) Wide excision Ans: C Feedback: Wide excision is the primary treatment for malignant melanoma, which removes the entire lesion and determines the level and staging. Chemotherapy may be used after the melanoma is excised. Immunotherapy is experimental and radiation therapy is palliative.

38. A patient has been assessed for aldosteronism and has recently begun treatment. What are priority areas for assessment that the nurse should frequently address? Select all that apply. A) Pupillary response B) Creatinine and BUN levels C) Potassium level D) Peripheral pulses E) BP

C, E Feedback: Patients with aldosteronism exhibit a profound decline in the serum levels of potassium, and hypertension is the most prominent and almost universal sign of aldosteronism. Pupillary response, peripheral pulses, and renal function are not directly affected.

A nurse is caring for a patient with severe hemolytic jaundice. Laboratory tests show free bilirubin to be 24 mg/dL. For what complication is this patient at risk? A. Chronic jaundice B. Pigment stones in portal circulation C. Central nervous system damage D. Hepatomegaly

C. Central nervous system damage Rationale: Prolonged jaundice, even if mild, predisposes to the formation of pigment stones in the gallbladder, and extremely severe jaundice (levels of free bilirubin exceeding 20 to 25 mg/dL) poses a risk for CNS damage. There are not specific risks of hepatomegaly or chronic jaundice resulting from high bilirubin

A patient with portal hypertension has been admitted to the medical floor. The nurse should prioritize which of the following assessments related to the manifestations of this health problem? A. Assessment of blood pressure and assessment for headaches and visual changes. B. Assessments for signs and symptoms of venous thromboembolism. C. Daily weights and abdominal girth measurement. D. Blood glucose monitoring q4h.

C. Daily weights and abdominal girth measurement Rationale: Obstruction to blood flow through the damaged liver results in increased blood pressure (portal hypertension) throughout the portal venous system. This can result in varices and ascites in the abdominal cavity. Assessments related to ascites are daily weights and abdominal girths. Portal hypertension is not synonymous with cardiovascular hypertension and does not create a risk for unstable blood glucose of VTE.

A patient with a diagnosis of cirrhosis has developed variceal bleeding and will imminently undergo variceal banding. What psychosocial nursing diagnosis should the nurse most likely prioritize during this phase of the patient's treatment? A. Decisional Conflict B. Deficient Knowledge C. Death Anxiety D. Disturbed Thought Processes

C. Death Anxiety Rationale: The sudden hemorrhage that accompanies variceal bleeding is intensely anxiety-provoking. The nurse must address the patient's likely fear of death, which is a realistic possibility. For most patients, anxiety is likely to be a more acute concern than lack of knowledge or decisional conflict. The patient may or may not experience disturbances in thought processes.

A nurse has entered the room of a patient with cirrhosis and found the patient on the floor. The patient stated that she fell when transferring to the commode. The patient's vital signs are within reference ranges and the nurse observes no apparent injuries. What is the nurse's most appropriate action? A. Remove the patient's commode and supply a bedpan B. Complete an incident report and submit it to the unit supervisor C. Have the patient assessed by the physician due to the risk of internal bleeding D. Perform a focused abdominal assessment in order to rule out injury.

C. Have the patient assessed by the physician due to the risk of internal bleeding Rationale: A fall would necessitate thorough medical assessment due to the patient's risk of bleeding. The nurse's abdominal assessment is an appropriate action, but is not wholly sufficient to rule out internal injury. Medical assessment is a priority over removing the commode or filling out an incident report, even though these actions are appropriate.

A previously healthy adult's sudden and precipitous decline in health has been attributed to fulminant hepatic failure, and the patient has been admitted to the intensive care unit. The nurse should be aware that the treatment of choice for this patient is what? A. IV administration of immune globulins B. Transfusion of packed red blood cells and fresh-frozen plasma (FFP) C. Liver transplantation D. Lobectomy

C. Liver transplantation Rationale: Liver transplantation carries the highest potential for the resolution of fulminant hepatic failure. This is preferred over other interventions, such as pharmacologic treatments, transfusions, and surgery

Diagnostic testing has revealed that a patient's hepatocellular carcinoma (HCC) is limited to one lobe. The nurse should anticipate that this patient's plan of care will focus on what intervention? A. Cryosurgery B. Liver transplantation C. Lobectomy D. Laser hyperthermia

C. Lobectomy Rationale: Surgical resection is the treatment of choice when HCC is confined to one lobe of the liver and the function of the remaining liver is considered adequate for postoperative recovery. Removal of a lobe of the liver (lobectomy) is the most common surgical procedure for excising a liver tumor. While cryosurgery and liver transplantation are other surgical options for management of liver cancer, these procedures are not performed at the same frequency as a lobectomy. Laser hyperthermia is a nonsurgical treatment for liver cancer.

A nurse is performing an admission assessment of a patient with a diagnosis of cirrhosis. What technique should the nurse use to palpate the patient's liver? A. Place hand under the right lower abdominal quadrant and press down lightly with the other hand. B. Place the left hand over the abdomen and behind the left side at the 11th rib. C. Place hand under right lower rib cage and press down lightly with the other hand. D. Hold hand 90 degree to right side of the abdomen and push down firmly.

C. Place hand under right lower rib cage and press down lightly with the other hand. Rationale: To palpate the liver, the examiner places one hand under the right lower rib cage and presses downward with light pressure with the other hand. The liver is not on the left side or in the right lower abdominal quadrant.

A patient with liver disease has developed jaundice; the nurse is collaborating with the patient to develop a nutritional plan. The nurse should prioritize which of the following in the patient's plan? A. Increased potassium intake B. Fluid restriction to 2L per day C. Reduction in sodium intake D. High-protein, low-fat diet

C. Reduction in sodium intake Rationale: Patient with ascites require a sharp reduction in sodium intake. Potassium intake should not be correspondingly increased. There is no need for fluid restriction or increased protein intake.

A nurse is caring for a patient with hepatic encephalopathy. The nurse's assessment reveals that the patient exhibits episodes of confusion, is difficult to arouse from sleep and has rigid extremities. Based on these clinical findings, the nurse should document what stage of hepatic encephalopathy? A. Stage 1 B. Stage 2 C. Stage 3 D. Stage 4

C. Stage 3 Rationale: Patients in the third stage of hepatic encephalopathy exhibit the following symptoms: stuporous, difficult to arouse, sleeps most of the time, exhibits marked confusion, incoherent in speech, asterixis, increased deep tendon reflexes, rigidity of extremities, marked EEG abnormalities. Patients in stages 1 and 2 exhibit clinical symptoms that are not as advanced as found in stage 3, and patients in stage 4 are comatose. In stage 4, there is an absence of asterixis, absence of deep tendon reflexes, flaccidity of extremities, and EEG abnormalities.

A patient who has undergone liver transplantation is ready to be discharged home. Which outcome of health education should the nurse prioritize? A. The patient will obtain measurement of drainage from the T-tube. B. The patient will exercise three times a week. C. The patient will take immunosuppressive agents as required. D. The patient will monitor for signs of liver dysfunction.

C. The patient will take immunosuppressive agents as required. Rationale: The patient is given written and verbal instructions about immunosuppressive agent doses and dosing schedules. The patient is also instructed on steps to follow to ensure that an adequate supply of medication is available so that there is no change of running out of the medication or skipping a dose. Failure to take medications as instructed may precipitate rejection. The nurse would not teach the patient to measure drainage from a T-tube as the patient would;t go home with a T-tube. The nurse may teach the patient about the need to exercise or what the signs of liver dysfunction are, but the nurse would not stress these topics over the immunosuppressive drug regimen.

A patient diagnosed with a cerebral aneurysm reports a severe headache to the nurse. What action is a priority for the nurse? A) Sit with the patient for a few minutes. B) Administer an analgesic. C) Inform the nurse-manager. D) Call the physician immediately.

D (Feedback: A headache may be an indication that the aneurysm is leaking. The nurse should notify the physician immediately. The physician will decide whether administration of an analgesic is indicated. Informing the nurse-manager is not necessary. Sitting with the patient is appropriate, once the physician has been notified of the change in the patient's condition.)

A nurse in the ICU is providing care for a patient who has been admitted with a hemorrhagic stroke. The nurse is performing frequent neurologic assessments and observes that the patient is becoming progressively more drowsy over the course of the day. What is the nurse's best response to this assessment finding? A) Report this finding to the physician as an indication of decreased metabolism. B) Provide more stimulation to the patient and monitor the patient closely. C) Recognize this as the expected clinical course of a hemorrhagic stroke. D) Report this to the physician as a possible sign of clinical deterioration.

D (Feedback: Alteration in LOC often is the earliest sign of deterioration in a patient with a hemorrhagic stroke. Drowsiness and slight slurring of speech may be early signs that the LOC is deteriorating. This finding is unlikely to be the result of metabolic changes and it is not expected. Stimulating a patient with an acute stroke is usually contraindicated.)

The public health nurse is planning a health promotion campaign that reflects current epidemiologic trends. The nurse should know that hemorrhagic stroke currently accounts for what percentage of total strokes in the United States? A) 43% B) 33% C) 23% D) 13%

D (Feedback: Strokes can be divided into two major categories: ischemic (87%), in which vascular occlusion and significant hypoperfusion occur, and hemorrhagic (13%), in which there is extravasation of blood into the brain or subarachnoid space.)

What should be included in the patient's care plan when establishing an exercise program for a patient affected by a stroke? A) Schedule passive range of motion every other day. B) Keep activity limited, as the patient may be over stimulated. C) Have the patient perform active range-of-motion (ROM) exercises once a day. D) Exercise the affected extremities passively four or five times a day.

D (Feedback: The affected extremities are exercised passively and put through a full ROM four or five times a day to maintain joint mobility, regain motor control, prevent development of a contracture in the paralyzed extremity, prevent further deterioration of the neuromuscular system, and enhance circulation. Active ROM exercises should ideally be performed more than once per day.)

A patient with a cerebral aneurysm exhibits signs and symptoms of an increase in intracranial pressure (ICP). What nursing intervention would be most appropriate for this patient? A) Range-of-motion exercises to prevent contractures B) Encouraging independence with ADLs to promote recovery C) Early initiation of physical therapy D) Absolute bed rest in a quiet, nonstimulating environment

D (Feedback: The patient is placed on immediate and absolute bed rest in a quiet, nonstressful environment because activity, pain, and anxiety elevate BP, which increases the risk for bleeding. Visitors are restricted. The nurse administers all personal care. The patient is fed and bathed to prevent any exertion that might raise BP.)

After a subarachnoid hemorrhage, the patient's laboratory results indicate a serum sodium level of less than 126 mEq/L. What is the nurse's most appropriate action? A) Administer a bolus of normal saline as ordered. B) Prepare the patient for thrombolytic therapy as ordered. C) Facilitate testing for hypothalamic dysfunction. D) Prepare to administer 3% NaCl by IV as ordered.

D (Feedback: The patient may be experiencing syndrome of inappropriate antidiuretic hormone (SIADH) or cerebral salt-wasting syndrome. The treatment most often is the use of IV hypertonic 3% saline. A normal saline bolus would exacerbate the problem and there is no indication for tests of hypothalamic function or thrombolytic therapy.)

A nursing student is writing a care plan for a newly admitted patient who has been diagnosed with a stroke. What major nursing diagnosis should most likely be included in the patient's plan of care? A) Adult failure to thrive B) Post-trauma syndrome C) Hyperthermia D) Disturbed sensory perception

D (Feedback: The patient who has experienced a stroke is at a high risk for disturbed sensory perception. Stroke is associated with multiple other nursing diagnoses, but hyperthermia, adult failure to thrive, and post-trauma syndrome are not among these.)

A female patient is diagnosed with a right-sided stroke. The patient is now experiencing hemianopsia. How might the nurse help the patient manage her potential sensory and perceptional difficulties? A) Keep the lighting in the patient's room low. B) Place the patient's clock on the affected side. C) Approach the patient on the side where vision is impaired. D) Place the patient's extremities where she can see them.

D (Feedback: The patient with homonymous hemianopsia (loss of half of the visual field) turns away from the affected side of the body and tends to neglect that side and the space on that side; this is called amorphosynthesis. In such instances, the patient cannot see food on half of the tray, and only half of the room is visible. It is important for the nurse to remind the patient constantly of the other side of the body, to maintain alignment of the extremities, and if possible, to place the extremities where the patient can see them. Patients with a decreased field of vision should be approached on the side where visual perception is intact. All visual stimuli (clock, calendar, and television) should be placed on this side. The patient can be taught to turn the head in the direction of the defective visual field to compensate for this loss. Increasing the natural or artificial lighting in the room and providing eyeglasses are important in increasing vision. There is no reason to keep the lights dim.)

A patient recovering from a stroke has severe shoulder pain from subluxation of the shoulder and is being cared for on the unit. To prevent further injury and pain, the nurse caring for this patient is aware of what principle of care? A) The patient should be fitted with a cast because use of a sling should be avoided due to adduction of the affected shoulder. B) Elevation of the arm and hand can lead to further complications associated with edema. C) Passively exercising the affected extremity is avoided in order to minimize pain. D) The patient should be taught to interlace fingers, place palms together, and slowly bring scapulae forward to avoid excessive force to shoulder.

D (Feedback: To prevent shoulder pain, the nurse should never lift a patient by the flaccid shoulder or pull on the affected arm or shoulder. The patient is taught how to move and exercise the affected arm/shoulder through proper movement and positioning. The patient is instructed to interlace the fingers, place the palms together, and push the clasped hands slowly forward to bring the scapulae forward; he or she then raises both hands above the head. This is repeated throughout the day. The use of a properly worn sling when the patient is out of bed prevents the paralyzed upper extremity from dangling without support. Range-of-motion exercises are still vitally important in preventing a frozen shoulder and ultimately atrophy of subcutaneous tissues, which can cause more pain. Elevation of the arm and hand is also important in preventing dependent edema of the hand.)

27. A patient has been taking prednisone for several weeks after experiencing a hypersensitivity reaction. To prevent adrenal insufficiency, the nurse should ensure that the patient knows to do which of the following? A) Take the drug concurrent with levothyroxine (Synthroid). B) Take each dose of prednisone with a dose of calcium chloride. C) Gradually replace the prednisone with an OTC alternative. D) Slowly taper down the dose of prednisone, as ordered.

D Feedback: Corticosteroid dosages are reduced gradually (tapered) to allow normal adrenal function to return and to prevent steroid-induced adrenal insufficiency. There are no OTC substitutes for prednisone and neither calcium chloride nor levothyroxine addresses the risk of adrenal insufficiency.

19. A patient with hypofunction of the adrenal cortex has been admitted to the medical unit. What would the nurse most likely find when assessing this patient? A) Increased body temperature B) Jaundice C) Copious urine output D) Decreased BP

D Feedback: Decreased BP may occur with hypofunction of the adrenal cortex. Decreased function of the adrenal cortex does not affect the patients body temperature, urine output, or skin tone.

30. A patient with pheochromocytoma has been admitted for an adrenalectomy to be performed the following day. To prevent complications, the nurse should anticipate preoperative administration of which of the following? A) IV antibiotics B) Oral antihypertensives C) Parenteral nutrition D) IV corticosteroids

D Feedback: IV administration of corticosteroids (methylprednisolone sodium succinate [Solu-Medrol]) may begin on the evening before surgery and continue during the early postoperative period to prevent adrenal insufficiency. Antibiotics, antihypertensives, and parenteral nutrition do not prevent adrenal insufficiency or other common complications of adrenalectomy.

24. A patient with a recent diagnosis of hypothyroidism is being treated for an unrelated injury. When administering medications to the patient, the nurse should know that the patients diminished thyroid function may have what effect? A) Anaphylaxis B) Nausea and vomiting C) Increased risk of drug interactions D) Prolonged duration of effect

D Feedback: In all patients with hypothyroidism, the effects of analgesic agents, sedatives, and anesthetic agents are prolonged. There is no direct increase in the risk of anaphylaxis, nausea, or drug interactions, although these may potentially result from the prolonged half-life of drugs.

12. The home care nurse is conducting patient teaching with a patient on corticosteroid therapy. To achieve consistency with the bodys natural secretion of cortisol, when would the home care nurse instruct the patient to take his or her corticosteroids? A) In the evening between 4 PM and 6 PM B) Prior to going to sleep at night C) At noon every day D) In the morning between 7 AM and 8 AM

D Feedback: In keeping with the natural secretion of cortisol, the best time of day for the total corticosteroid dose is in the morning from 7 to 8 AM. Large-dose therapy at 8 AM, when the adrenal gland is most active, produces maximal suppression of the gland. Also, a large 8 AM dose is more physiologic because it allows the body to escape effects of the steroids from 4 PM to 6 AM, when serum levels are normally low, thus minimizing cushingoid effects.

14. The nurse is caring for a patient with hyperparathyroidism. What level of activity would the nurse expect to promote? A) Complete bed rest B) Bed rest with bathroom privileges C) Out of bed (OOB) to the chair twice a day D) Ambulation and activity as tolerated

D Feedback: Mobility, with walking or use of a rocking chair for those with limited mobility, is encouraged as much as possible because bones subjected to normal stress give up less calcium. Best rest should be discouraged because it increases calcium excretion and the risk of renal calculi. Limiting the patient to getting out of bed only a few times a day also increases calcium excretion and the associated risks.

40. The nurse is providing care for an older adult patient whose current medication regimen includes levothyroxine (Synthroid). As a result, the nurse should be aware of the heightened risk of adverse effects when administering an IV dose of what medication? A) A fluoroquinalone antibiotic B) A loop diuretic C) A proton pump inhibitor (PPI) D) A benzodiazepine

D Feedback: Oral thyroid hormones interact with many other medications.Even in small IV doses, hypnotic and sedative agents may induce profound somnolence, lasting far longer than anticipated and leading to narcosis (stupor like condition). Furthermore, they are likely to cause respiratory depression, which can easily be fatal because of decreased respiratory reserve and alveolar hypoventilation. Antibiotics, PPIs and diuretics do not cause the same risk.

34. A patient on corticosteroid therapy needs to be taught that a course of corticosteroids of 2 weeks duration can suppress the adrenal cortex for how long? A) Up to 4 weeks B) Up to 3 months C) Up to 9 months D) Up to 1 year

D Feedback: Suppression of the adrenal cortex may persist up to 1 year after a course of corticosteroids of only 2 weeks duration.

36. A 30-year-old male patient has just returned from the operating room after having a "flap" done following a motorcycle accident. The patient's wife asks the nurse about the major complications following this type of surgery. What would be the nurse's best response? A) "The major complication is when the patient develops chronic pain." B) "The major complication is when the patient loses sensation in the flap." C) "The major complication is when the pedicle tears loose and the flap dies." D) "The major complication is when the blood supply fails and the tissue in the flap dies."

D) "The major complication is when the blood supply fails and the tissue in the flap dies." Ans: D Feedback: The major complication of a flap is necrosis of the pedicle or base as a result of failure of the blood supply. This is more likely than tearing of the pedicle and chronic pain and is more serious than loss of sensation.

17. A patient arrives in the emergency department after being burned in a house fire. The patient's burns cover the face and the left forearm. What extent of burns does the patient most likely have? A) 13% B) 25% C) 9% D) 18%

D) 18% Ans: D Feedback: When estimating the percentage of body area or burn surface area that has been burned, the Rule of Nines is used: the face is 9%, and the forearm is 9% for a total of 18% in this patient.

13. A nurse is caring for a patient who has sustained a deep partial-thickness burn injury. In prioritizing the nursing diagnoses for the plan of care, the nurse will give the highest priority to what nursing diagnosis? A) Activity Intolerance B) Anxiety C) Ineffective Coping D) Acute Pain

D) Acute Pain Ans: D Feedback: Pain is inevitable during recovery from any burn injury. Pain in the burn patient has been described as one of the most severe causes of acute pain. Management of the often-severe pain is one of the most difficult challenges facing the burn team. While the other nursing diagnoses listed are valid, the presence of pain may contribute to these diagnoses. Management of the patient's pain is the priority, as it may have a direct correlation to the other listed nursing diagnoses.

3. A nurse is planning the care of a patient with herpes zoster. What medication, if administered within the first 24 hours of the initial eruption, can arrest herpes zoster? A) Prednisone (Deltasone) B) Azanthioprine (Imuran) C) Triamcinolone (Kenalog) D) Acyclovir (Zovirax)

D) Acyclovir (Zovirax) Ans: D Feedback: Acyclovir, if started early, is effective in significantly reducing the pain and halting the progression of the disease. There is evidence that infection is arrested if oral antiviral agents are administered within the first 24 hours. Prednisone is an anti-inflammatory agent used in a variety of skin disorders, but not in the treatment of herpes. Azanthioprine is an immunosuppressive agent used in the treatment of pemphigus. Triamcinolone is utilized in the treatment of psoriasis.

16. A patient is brought to the ED by paramedics, who report that the patient has partial-thickness burns on the chest and legs. The patient has also suffered smoke inhalation. What is the priority in the care of a patient who has been burned and suffered smoke inhalation? A) Pain B) Fluid balance C) Anxiety and fear D) Airway management

D) Airway management Ans: D Feedback: Systemic threats from a burn are the greatest threat to life. The ABCs of all trauma care apply during the early postburn period. While all options should be addressed, pain, fluid balance, and anxiety and fear do not take precedence over airway management.

29. A patient has just undergone surgery for malignant melanoma. Which of the following nursing actions should be prioritized? A) Maintain the patient on bed rest for the first 24 hours postoperative. B) Apply distraction techniques to relieve pain. C) Provide soft or liquid diet that is high in protein to assist with healing. D) Anticipate the need for, and administer, appropriate analgesic medications.

D) Anticipate the need for, and administer, appropriate analgesic medications. Ans: D Feedback: Nursing interventions after surgery for a malignant melanoma center on promoting comfort, because wide excision surgery may be necessary. Anticipating the need for and administering appropriate analgesic medications are important. Distraction techniques may be appropriate for some patients, but these are not a substitute for analgesia. Bed rest and a modified diet are not necessary.

30. A nurse is performing a home visit to a patient who is recovering following a long course of inpatient treatment for burn injuries. When performing this home visit, the nurse should do which of the following? A) Assess the patient for signs of electrolyte imbalances. B) Administer fluids as ordered. C) Assess the risk for injury recurrence. D) Assess the patient's psychosocial state.

D) Assess the patient's psychosocial state. Ans: D Feedback: Recovery from burns can be psychologically challenging; the nurse's assessments must address this reality. Fluid and electrolyte imbalances are infrequent during the rehabilitation phase of recovery. Burns are not typically a health problem that tends to recur; the experience of being burned tends to foster vigilance.

33. A 65-year-old man presents at the clinic complaining of nodules on both legs. The man tells the nurse that his son, who is in medical school, encouraged him to seek prompt care and told him that the nodules are related to the fact that he is Jewish. What health problem should the nurse suspect? A) Stasis ulcers B) Bullous pemphigoid C) Psoriasis D) Classic Kaposi's sarcoma

D) Classic Kaposi's sarcoma Ans: D Feedback: Classic Kaposi's sarcoma occurs predominantly in men of Mediterranean or Jewish ancestry between 40 and 70 years of age. Most patients have nodules or plaques on the lower extremities that rarely metastasize beyond this area. Classic KS is chronic, relatively benign, and rarely fatal. Stasis ulcers do not create nodules. Bullous pemphigoid is characterized by blistering. Psoriasis characteristically presents with silvery plaques.

33. A nurse is assessing the skin of a patient who has been diagnosed with bacterial cellulitis on the dorsal portion of the great toe. When reviewing the patient's health history, the nurse should identify what comorbidity as increasing the patient's vulnerability to skin infections? A) Chronic obstructive pulmonary disease B) Rheumatoid arthritis C) Gout D) Diabetes

D) Diabetes Ans: D Feedback: Patients with diabetes are particularly susceptible to skin infections. COPD, RA, and gout are less commonly associated with integumentary manifestations.

26. A nurse is assessing a teenage patient with acne vulgaris. The patient's mother states, I keep telling him that this is what happens when you eat as much chocolate as he does. What aspect of the pathophysiology of acne should inform the nurse's response? A) A sudden change in patient's diet may exacerbate, rather than alleviate, the patient's symptoms. B) Chocolate is not among the foods that are known to cause acne. C) Elimination of chocolate from the patient's diet will likely lead to resolution within several months. D) Diet is thought to play a minimal role in the development of acne.

D) Diet is thought to play a minimal role in the development of acne. Ans: D Feedback: Diet is not believed to play a major role in acne therapy. A change in diet is not known to exacerbate symptoms.

37. A patient's health assessment has resulted in a diagnosis of alopecia areata. What nursing diagnosis should the nurse most likely associate with this health problem? A) Chronic Pain B) Impaired Skin Integrity C) Impaired Tissue Integrity D) Disturbed Body Image

D) Disturbed Body Image Ans: D Feedback: Alopecia areata causes hair loss in smaller defined areas. As such, it is common for the patient to experience a disturbed body image. Hair loss does not cause pain and does not affect skin or tissue integrity.

37. The nurse caring for a patient who is recovering from full-thickness burns is aware of the patient's risk for contracture and hypertrophic scarring. How can the nurse best mitigate this risk? A) Apply skin emollients as ordered after granulation has occurred. B) Keep injured areas immobilized whenever possible to promote healing. C) Administer oral or IV corticosteroids as ordered. D) Encourage physical activity and range of motion exercises.

D) Encourage physical activity and range of motion exercises. Ans: D Feedback: Exercise and the promotion of mobility can reduce the risk of contracture and hypertrophic scarring. Skin emollients are not normally used in the treatment of burns, and these do not prevent scarring. Steroids are not used to reduce scarring, as they also slow the healing process.

8. A nurse is providing care for a patient who has developed Kaposi's sarcoma secondary to HIV infection. The nurse should be aware that this form of malignancy originates in what part of the body? A) Connective tissue cells in diffuse locations B) Smooth muscle cells of the gastrointestinal and respiratory tract C) Neural tissue of the brain and spinal cord D) Endothelial cells lining small blood vessels

D) Endothelial cells lining small blood vessels Ans: D Feedback: Kaposi's sarcoma (KS) is a malignancy of endothelial cells that line the small blood vessels. It does not originate in connective tissue, smooth muscle cells of the GI and respiratory tract, or in neural tissue.

1. A patient is brought to the emergency department from the site of a chemical fire, where he suffered a burn that involves the epidermis, dermis, and the muscle and bone of the right arm. On inspection, the skin appears charred. Based on these assessment findings, what is the depth of the burn on the patient's arm? A) Superficial partial-thickness B) Deep partial-thickness C) Full partial-thickness D) Full-thickness

D) Full-thickness Ans: D Feedback: A full-thickness burn involves total destruction of the epidermis and dermis and, in some cases, underlying tissue as well. Wound color ranges widely from white to red, brown, or black. The burned area is painless because the nerve fibers are destroyed. The wound can appear leathery; hair follicles and sweat glands are destroyed. Edema may also be present. Superficial partial-thickness burns involve the epidermis and possibly a portion of the dermis; the patient will experience pain that is soothed by cooling. Deep partial-thickness burns involve the epidermis, upper dermis, and portion of the deeper dermis; the patient will complain of pain and sensitivity to cold air. Full partial thickness is not a depth of burn.

13. A patient comes to the clinic complaining of a red rash of small, fluid-filled blisters and is suspected of having herpes zoster. What presentation is most consistent with herpes zoster? A) Grouped vesicles occurring on lips and oral mucous membranes B) Grouped vesicles occurring on the genitalia C) Rough, fresh, or gray skin protrusions D) Grouped vesicles in linear patches along a dermatome

D) Grouped vesicles in linear patches along a dermatome Ans: D Feedback: Herpes zoster, or shingles, is an acute inflammation of the dorsal root ganglia, causing localized, vesicular skin lesions following a dermatome. Herpes simplex type 1 is a viral infection affecting the skin and mucous membranes, usually producing cold sores or fever blisters. Herpes simplex type 2 primarily affects the genital area, causing painful clusters of small ulcerations. Warts appear as rough, fresh, or gray skin protrusions.

14. A nurse is working with a patient who has a diagnosis of Cushing syndrome. When completing a physical assessment, the nurse should specifically observe for what integumentary manifestation? A) Alopecia B) Yellowish skin tone C) Patchy, bronze pigmentation D) Hirsutism

D) Hirsutism Ans: D Feedback: Cushing syndrome causes excessive hair growth, especially in women. Alopecia is hair loss from the scalp and other parts of the body. Jaundice causes a yellow discoloration in light-skinned patients, but this does not accompany Cushing syndrome. Patients that have Addison's disease exhibit a bronze discoloration to their skin due to increased melanin production.

22. An 82-year-old patient is being treated in the hospital for a sacral pressure ulcer. What age-related change is most likely to affect the patient's course of treatment? A) Increased thickness of the subcutaneous skin layer B) Increased vascular supply to superficial skin layers C) Changes in the character and quantity of bacterial skin flora D) Increased time required for wound healing

D) Increased time required for wound healing Ans: D Feedback: Wound healing becomes slower with age, requiring more time for older adults to recover from surgical and traumatic wounds. There are no changes in skin flora with increased age. Vascular supply and skin thickness both decrease with age.

26. A patient's burns are estimated at 36% of total body surface area; fluid resuscitation has been ordered in the emergency department. After establishing intravenous access, the nurse should anticipate the administration of what fluid? A) 0.45% NaCl with 20 mEq/L KCl B) 0.45% NaCl with 40 mEq/L KCl C) Normal saline D) Lactated Ringer's

D) Lactated Ringer's Ans: D Feedback: Fluid resuscitation with lactated Ringers (LR) should be initiated using the American Burn Association's (ABA) fluid resuscitation formulas. LR is the crystalloid of choice because its composition and osmolality most closely resemble plasma and because use of normal saline is associated with hyperchloremic acidosis. Potassium chloride solutions would exacerbate the hyperkalemia that occurs following burn injuries.

18. A nurse is providing an educational presentation addressing the topic of Protecting Your Skin. When discussing the anatomy of the skin with this group, the nurse should know that what cells are responsible for producing the pigmentation of the skin? A) Islets of Langerhans B) Squamous cells C) T cells D) Melanocytes

D) Melanocytes Ans: D Feedback: Melanocytes are the special cells of the epidermis that are primarily responsible for producing the pigment melanin. Islets of Langerhans are clusters of cells in the pancreas. Squamous cells are flat, scaly epithelial cells. T cells function in the immune response.

1. A nurse is aware that the outer layer of the skin consists of dead cells that contain large amounts of keratin. The physiologic functions of keratin include which of the following? Select all that apply. A) Producing antibodies B) Absorbing electrolytes C) Maintaining acidbase balance D) Physically repelling pathogens E) Preventing fluid loss

D) Physically repelling pathogens E) Preventing fluid loss Ans: D, E Feedback: The dead cells of the epidermis contain large amounts of keratin, an insoluble, fibrous protein that forms the outer barrier of the skin. Keratin has the capacity to repel pathogens and prevent excessive fluid loss from the body. It does not contribute directly to antibody production, acidbase balance, or electrolyte levels.

32. A nurse is caring for a patient with burns who is in the later stages of the acute phase of recovery. The plan of nursing care should include which of the following nursing actions? A) Maintenance of bed rest to aid healing B) Choosing appropriate splints and functional devices C) Administration of beta adrenergic blockers D) Prevention of venous thromboembolism

D) Prevention of venous thromboembolism Ans: D Feedback: Prevention of deep vein thrombosis (DVT) is an important factor in care. Early mobilization of the patient is important. The nurse monitors the splints and functional devices, but these are selected by occupational and physical therapists. The hemodynamic changes accompanying burns do not normally require the use of beta blockers.

19. A nurse is caring for a patient whose chemical injury has necessitated a skin graft to his left hand. The nurse enters the room and observes that the patient is performing active range of motion (ROM) exercises with the affected hand. How should the nurse best respond? A) Liaise with the physical therapist to ensure that the patient is performing exercises safely. B) Validate the patient's efforts to increase blood perfusion to the graft site. C) Remind the patient that ROM exercises should be passive, not active. D) Remind the patient of the need to immobilize the graft to facilitate healing.

D) Remind the patient of the need to immobilize the graft to facilitate healing. Ans: D Feedback: The nurse should instruct the patient to keep the affected part immobilized as much as possible in order to facilitate healing. Passive ROM exercises can be equally as damaging as active ROM.

7. An African American is admitted to the medical unit with liver disease. To correctly assess this patient for jaundice, on what body area should the nurse look for yellow discoloration? A) Elbows B) Lips C) Nail beds D) Sclerae

D) Sclerae Ans: D Feedback: Jaundice, a yellowing of the skin, is directly related to elevations in serum bilirubin and is often first observed in the sclerae and mucous membranes.

24. A patient with an exceptionally low body mass index has been admitted to the emergency department with signs and symptoms of hypothermia. The nurse should know that this patient's susceptibility to heat loss is related to atrophy of what skin component? A) Epidermis B) Merkel cells C) Dermis D) Subcutaneous tissue

D) Subcutaneous tissue Ans: D Feedback: The subcutaneous tissues and the amount of fat deposits are important factors in body temperature regulation. The epidermis is an outermost layer of stratified epithelial cells. Merkel cells are receptors that transmit stimuli to the axon through a chemical synapse. The dermis makes up the largest portion of the skin, providing strength and structure. It is composed of two layers: papillary and reticular.

6. A patient who has sustained third-degree facial burns and a facial fracture is undergoing reconstructive surgery and implantation of a prosthesis. The nurse has identified a nursing diagnosis of Disturbed Body Image Related to Disfigurement. What would be an appropriate nursing intervention related to this diagnosis? A) Referring the patient to a speech therapist B) Gradually adding soft foods to diet C) Administering analgesics as prescribed D) Teaching the patient how to use and care for the prosthesis

D) Teaching the patient how to use and care for the prosthesis Ans: D Feedback: The process of facial reconstruction is often slow and tedious. Because a person's facial appearance affects self-esteem so greatly, this type of reconstruction is often a very emotional experience for the patient. Reinforcement of the patient's successful coping strategies improves self-esteem. If prosthetic devices are used, the patient is taught how to use and care for them to gain a sense of greater independence. This is an intervention that relates to Disturbed Body Image in these patients. None of the other listed interventions relates directly to the diagnosis of Disturbed Body Image.

38. While performing a patient's ordered wound care for the treatment of a burn, the patient has made a series of sarcastic remarks to the nurse and criticized her technique. How should the nurse best interpret this patient's behavior? A) The patient may be experiencing an adverse drug reaction that is affecting his cognition and behavior. B) The patient may be experiencing neurologic or psychiatric complications of his injuries. C) The patient may be experiencing inconsistencies in the care that he is being provided. D) The patient may be experiencing anger about his circumstances that he is deflecting toward the nurse.

D) The patient may be experiencing anger about his circumstances that he is deflecting toward the nurse. Ans: D Feedback: The patient may experience feelings of anger. The anger may be directed outward toward those who escaped unharmed or toward those who are now providing care. While drug reactions, complications, and frustrating inconsistencies in care cannot be automatically ruled out, it is not uncommon for anger to be directed at caregivers.

6. An emergency department nurse has just admitted a patient with a burn. What characteristic of the burn will primarily determine whether the patient experiences a systemic response to this injury? A) The length of time since the burn B) The location of burned skin surfaces C) The source of the burn D) The total body surface area (TBSA) affected by the burn

D) The total body surface area (TBSA) affected by the burn Ans: D Feedback: Systemic effects are a result of several variables. However, TBSA and wound severity are considered the major factors that affect the presence or absence of systemic effects.

5. While assessing a dark-skinned patient at the clinic, the nurse notes the presence of patchy, milky white spots. The nurse knows that this finding is characteristic of what diagnosis? A) Cyanosis B) Addison's disease C) Polycythemia D) Vitiligo

D) Vitiligo Ans: D Feedback: With cyanosis, nail beds are dusky. With polycythemia, the nurse notes ruddy blue face, oral mucosa, and conjunctiva. A bronzed appearance, or external tan, is associated with Addison's disease. Vitiligo is a condition characterized by destruction of the melanocytes in circumscribed areas of skin and appears in light or dark skin as patchy, milky white spots, often symmetric bilaterally.

20. A young student comes to the school nurse and shows the nurse a mosquito bite. As the nurse expects, the bite is elevated and has serous fluid contained in the dermis. How would the nurse classify this lesion? A) Vesicle B) Macule C) Nodule D) Wheal

D) Wheal Ans: D Feedback: A wheal is a primary skin lesion that is elevated and has fluid contained in the dermis. An example of a wheal would be an insect bite or hives. Vesicles, macules, and nodules are not characterized by elevation and the presence of serous fluid.

5. An occupational health nurse is called to the floor of a factory where a worker has sustained a flash burn to the right arm. The nurse arrives and the flames have been extinguished. The next step is to cool the burn. How should the nurse cool the burn? A) Apply ice to the site of the burn for 5 to 10 minutes. B) Wrap the patient's affected extremity in ice until help arrives. C) Apply an oil-based substance or butter to the burned area until help arrives. D) Wrap cool towels around the affected extremity intermittently.

D) Wrap cool towels around the affected extremity intermittently. Ans: D Feedback: Once the burn has been sustained, the application of cool water is the best first-aid measure. Soaking the burn area intermittently in cool water or applying cool towels gives immediate and striking relief from pain, and limits local tissue edema and damage. However, never apply ice directly to the burn, never wrap the person in ice, and never use cold soaks or dressings for longer than several minutes; such procedures may worsen the tissue damage and lead to hypothermia in people with large burns. Butter is contraindicated.

A nurse is performing an admission assessment for an 81-year-old patient who generally enjoys good health. When considering normal, age-related changes to hepatic function, the nurse should anticipate what finding? A. Similar liver size and texture as in younger adults B. A nonpalpable liver C. A slightly enlarged liver with palpably hard edges D. A slightly decreased size of the liver

D. A slightly decreased size of the liver Rationale: The most common age-related change in the liver is a decrease in size and weight. The liver is usually still palpable, however, it is not expected to have hardened edges.

A nurse is amending a patient's plan of care in light of the fact that the patient has recently developed ascites. What should the nurse include in this patient's care plan? A. Mobilization with assistance at least 4 times daily B. Administration of beta-adrenergic blockers as ordered C. Vitamin B12 injections as ordered D. Administration of diuretics as ordered

D. Administration of diuretics as ordered Rationale: Use of diuretics along with sodium restriction is successful in 90% of patients with ascites. Beta-blockers are not used to treat ascites and bed rest is often more beneficial than increased mobility. Vitamin B23 injections are not necessary.

A local public health nurse is informed that a cook in a local restaurant has been diagnosed with hepatitis A. What should the nurse advise individuals to obtain who ate at this restaurant and have never received the hepatitis A vaccine? A. The hepatitis A vaccine B. Albumin infusion C. The hepatitis A and B vaccines D. An immune globulin injection

D. An immune globulin injection Rationale: For people who have not been previously vaccinated, hepatitis A can be prevented by the intramuscular administration of immune globulin during the incubation period, if given within 2 weeks of exposure. Administration of the hepatitis A vaccine will not protect the patient exposed to hepatitis A, as protection will take a few weeks to develop after the first dose of the vaccine. The hepatitis B vaccine provides protection against the hepatitis B virus, but plays no role in protection for the patient exposed to hepatitis A. Albumin confers no therapeutic benefit.

A patient with liver cancer is being discharged home with a biliary drainage system in place. The nurse should teach the patient's family how to safely perform which of the following actions? A. Aspirating bile from the catheter using a syringe B. Removing the catheter when output is 15 mL in 24 hours C. Instilling antibiotics into the catheter D. Assessing the patency of the drainage catheter

D. Assessing the patency of the drainage catheter Rationale: Families should be taught to provide basic catheter care, including assessment of patency. Antibiotics are not instilled into the catheter and aspiration using a syringe is contraindicated. The family would not independently remove the catheter; this would be done by a member of the care team when deemed necessary.

A patient with liver cancer is being discharged home with a hepatic artery catheter in place. The nurse should be aware that this catheter will facilitate which of the following? A. Continuous monitoring for portal hypertension B. Administration of immunosuppressive drugs during the first weeks after transplantation C. Real-time monitoring of vascular changes in the hepatic system D. Delivery of a continuous chemotherapeutic dose

D. Delivery of a continuous chemotherapeutic dose Rationale: In most cases, the hepatic artery catheter has been inserted surgically and has a prefilled infusion pump that delivers a continuous chemotherapeutic dose until completed. The hepatic artery catheter dose not monitor portal hypertension, deliver immunosuppressive drugs, or monitor vascular changes in the hepatic system

A nurse is participating in the emergency care of a patient who has just developed variceal bleeding. What intervention should the nurse anticipate? A. Infusion of intravenous heparin B. IV administration of albumin C. STAT administration of vitamin K by the intramuscular route D. IV administration of octreotide (Sandostatin)

D. IV administration of octreotide (Sandostation) Rationale: Octreotide (Sandostatin) a synthetic analog of the hormone somatostatin is effective in decreasing bleeding from esophageal varices, and lacks the vasoconstrictive effects of vasopressin. Because of this safety and efficacy profile, octreotide is considered the preferred treatment regimen for immediate control of variceal bleeding. Vitamin K and albumin are not administered and heparin would exacerbate, not alleviate, bleeding.

A nurse is caring for a patient with liver failure and is performing an assessment in the knowledge of the patient's increased risk of bleeding. The nurse recognizes that this risk is related to the patient's inability to synthesize prothrombin in the liver. What factor most likely contributes to this loss of function? A. Alterations in glucose metabolism B. Retention of bile salts C. Inadequate production of albumin by hepatocytes D. Inability of the liver to use vitamin K.

D. Inability of the liver to use vitamin K. Rationale: Decreased production of several clotting factors may be partially due to deficient absorption of vitamin K from the GI tract. This probably is caused by the inability of liver cells to use vitamin K to make prothrombin. This bleeding risk is unrelated to the roles of glucose, bile salts, or albumin.

A patient with esophageal varices is being cared for in the ICU. The varices have begun to bleed and the patient is at risk for hypovolemia. The patient has Ringer's Lactate at 150 cc/hr infusing. What else might the nurse expect to have ordered to maintain volume for this patient? A. Arterial line B. Diuretics C. Foley catheter D. Volume expanders

D. Volume expanders Rationale: Because patients with bleeding esophageal varices have intravascular volume depletion and are subject to electrolyte imbalance, IV fluids with electrolytes and volume expanders are provided to restore fluid volume and replace electrolytes. Diuretics would reduce vascular volume. An arterial line and Foley catheter are likely to be ordered, but neither actively maintains the patient's volume.

Ans: "For many people, lack of nutrition can cause a loss of bone density." Feedback: Nutrition has a profound effect on bone density, especially later life. Genetics are also an important factor, but nutrition has a more pronounced effect. The pathophysiology of bone density is well understood and psychosocial stress has a minimal effect.

Diagnostic tests show that a patient's bone density has decreased over the past several years. The patient asks the nurse what factors contribute to bone density decreasing. What would be the nurse's best response?

Ans: Flat bones Feedback: Flat bones, such as the sternum, provide vital organ protection. Fractures of the flat bones may lead to puncturing of the vital organs or may interfere with the protection of the vital organs. Long, short, and irregular bones do not usually have this physiologic function.

The human body is designed to protect its vital parts. A fracture of what type of bone may interfere with the protection of vital organs?

Ans: Calcium, Vitamin D Feedback: A patient's risk for osteoporosis is strongly influenced by vitamin D and calcium intake. Carbohydrate, protein, and fiber intake do not have direct effect on the development of osteoporosis.

The nurse is assessing a patient for dietary factors that may influence her risk for osteoporosis. The nurse should question the patient about her intake of what nutrients? Select all that apply.

Ans: Compare parts of the body symmetrically. Feedback: When assessing bone integrity, symmetric parts of the body, such as extremities, are compared. Analgesia should not be necessary and percussion is not a clinically useful assessment technique. Bone integrity is best assessed when the patient is not moving.

The nurse is performing an assessment of a patient's musculoskeletal system and is appraising the patient's bone integrity. What action should the nurse perform during this phase of assessment?

Ans: Instruct the patient to walk away from the nurse for a short distance and then toward the nurse. Feedback: Gait is assessed by having the patient walk away from the examiner for a short distance. The examiner observes the patient's gait for smoothness and rhythm. Looking at the floor is not disallowed and gait is not assessed by observing balance on one leg. Heel-to-toe walking ability is not gauged during an assessment of normal gait.

The nurse's comprehensive assessment of an older adult involves the assessment of the patient's gait. How should the nurse best perform this assessment?

Ans: Fasciculations Feedback: Fasciculation is involuntary twitching of muscle fiber groups. Clonus is a series of involuntary, rhythmic, muscular contractions and tetany is involuntary muscle contraction, but neither is characterized as "twitching." Atony is a loss of muscle strength.

The nurse's musculoskeletal assessment of a patient reveals involuntary twitching of muscle groups. How would the nurse document this observation in the patient's chart?

Ans: Lordosis Feedback: The nurse documents the spinal abnormality as lordosis. Lordosis is an increase in lumbar curvature of the spine. Kyphosis is an increase in the convex curvature of the spine. Scoliosis is a lateral curvature of the spine. Osteoporosis is the significant loss of bone mass and strength with an increased risk for fracture.

The results of a nurse's musculoskeletal examination show an increase in the lumbar curvature of the spine. The nurse should recognize the presence of what health problem?

Ans: Ulnar Feedback: The ulnar nerve is assessed for sensation by pricking the fat pad at the top of the small finger. The radial, median, and tibial nerves are not assessed in this manner.

When assessing a patient's peripheral nerve function, the nurse uses an instrument to prick the fat pad at the top of the patient's small finger. This action will assess which of the following nerves?

Ans: Clonus Feedback: Clonus may occur when the ankle is dorsiflexed or the wrist is extended. It is characterized as rhythmic contractions of the muscle. Fasciculation is involuntary twitching of muscle fiber groups. Contractures are prolonged tightening of muscle groups and an effusion is the pathologic escape of body fluid.

While assessing a patient, the patient tells the nurse that she is experiencing rhythmic muscle contractions when the nurse performs passive extension of her wrist. What is this pattern of muscle contraction referred to as?


Related study sets

General Appraiser Market Analysis Highest and Best Use

View Set

Property Chapter 4 ExamFx: NC Statutes & Regulations For Property

View Set

Macroeconomics: Ch. 9 / Economic Growth and Rising Living Standards

View Set

RN 2.0 Caregiving - Custom Adaptive

View Set

Special Topic: Relativity and Spacetime

View Set